You are on page 1of 280

Introduction to Law

Course Pack

Compiled by:

The Academic Resource Center

Thomas M. Cooley Law School

Rev. 11/2009

1
2
Introduction to Law Course Pack
Table of Contents

Student Information Sheet……………………………………………………………..5


Week One……………………………………………………………………………..7
Week 1 Pre-Reading Questions………………………………………………………..9
Listening and Note-Taking in Class………………………………………………… 11
Week Two…............................................................................................................... 17
Week 2 Pre-Reading Questions………………………………………………………19
Getting the Most Out of What You Read……………………………………………..21
Reading Cases Efficiently: Two Examples…………………………………………. .27
Reading and Analyzing Notecases, Problems, Etc. in the Casebook…………………43
Contracts Table of Contents…………………………………………………………..46
Intent to Contract, Lucy v. Zehmer………………………………..…………………..50
Trespass to Land, Dougherty v. Stepp………………………………………………...59
Sample Briefs, Dougherty v. Stepp……………………………………………………61
Sample In-Class Essay, “The Hills Have Eyes”………………………………………63
Sample Answer, “The Hills Have Eyes”……………………………………………...65
Week Three…………………………………………………………………………..67
Week 3 Pre-Reading Questions……………………………………………………….69
Divergent Versus Convergent Thinking………………………………………………71
Multiple-Choice Questions: Understanding Transition Words……………………….77
Multiple Choice: Focused Practice……………………………………………………83
Intro to Law Multiple-Choice Questions……………………………………………...85
Template for Practicing Multiple-Choice Questions………………………………....111
Maximizing Memory…………………………………………………………………119
Week Four…………………………………………………………………………...129
Week 4 Pre-Reading Questions………………………………………………………131
Concept Development and Data-Organization Devices……………………………...133
Creating an Outline…………………………………………………………………...149
Week Five……………………………………………………………………………157
Week 5 Pre-Reading Questions………………………………………………………159
Reading Code…………………………………………………………………………161
UCC § 2-205………………………………………………………………………….168
Sample Essay………………………………………………………………………….169
UCC § 2-207………………………………………………………………………….170
Problem 31…………………………………………………………………………….171
UCC§ 2-314…………………………………………………………………………...172
Week Seven…………………………………………………………………………...173
Week 7 Pre-Reading Questions………………………………………………………..175
Creating Good Reasoning Statements…………………………………………………177
Endnote Practice……………………………………………………………………….183
Week Eight……………………………………………………………………………191
Week 8 Pre-Reading Questions………………………………………………………..193
Reading the “Call of the Question”……………………………………………………195
Planning the Answer to a Long Essay Question……………………………………….197

3
Question One…………………………………………………………………………...199
“Oh Nuts”……………………………………………………………………………….201
Week Nine……………………………………………………………………………...203
Planning the Answer to a Long Essay Question: Contracts…………………………….205
Planning the Answer to a Long Essay Question: Intentional Torts……………………..207
Planning the Answer to a Long Essay Question: Contracts…………………………….209
Planning the Answer to a Long Essay Question: Intentional Torts……………………..210
Writing Practice………………………………………………………………………..211
“Jimmy and the Petunias”……………………………………………………………….213
“Little Red Corvette”……………………………………………………………………219
“Psychics on Drugs”…………………………………………………………………….223
“David Ate a Sandwich – So What?”…………………………………………………...229
“Should She Pay the Ticket?”…………………………………………………………...233
“The Case of the Smoking Car”…………………………………………………………235
“Pete Hates Football”……………………………………………………………………237
“Here’s a Little Ditty `Bout Paula and Diane”…………………………………………..241
“And the Food’s Bad, Too!”…………………………………………………………….245
“Good Sports Get Cheated”……………………………………………………………..247
“A Swing and a Miss”…………………………………………………………………...250
Answer Guides…………………………………………………………………………255
Planning the Answer to a Long Essay Question: Contracts………………………….….256
Issue Guide: Intentional Torts…………………………………………………………...257
Issue Guide: Contracts…………………………………………………………………..258
Issue Guide: Intentional Torts…………………………………………………………...259
Sample Answer: Problem 2……………………………………………………………...260
Answers to Multiple-Choice Questions………………………………………………….261

4
Welcome to Introduction to Law!
Student Information Sheet

Please fill this out and give it to your adjunct professor in the second hour of class.

Your name (Please print) ___________________________ Student Number ________________

Your telephone number(s) __________________________ _____________________________

Your e-mail address (please give the e-mail address that you check most frequently)

_______________________________________________

Your class schedule

Mon Tues Wed Thurs Fri Sat Sun


8-9

9-10

10-11

11-12

12-1

1-2

2-3

3-4

4-5

5-6

6-7

7-8

8-9

5
6
Week One:

Listening and Taking Notes in Class

7
8
Week 1 Pre-Reading Questions

(1) Why is effective listening in the classroom important?

(2) What does it mean to “be an active listener”?

(3) What are some specific techniques for active listening?

(4) Why is it important to take notes, even if you follow and understand the discussion?

(5) What are the five traditional patterns used by speakers, and what is the nature of those patterns?

(6) What is the pattern that is unique to law school? Why is it important to recognize and take notes
when the discussion is in this pattern?

(7) What is the Cornell System of note-taking?

9
10
Listening and Note-Taking in Class
A. Listening
Purpose of this section: Some students have been taught effective listening techniques at some point
during their education. Most have not. This means that many students have learned, on their own, some
techniques of effective listening – but not others. And effective listening is a crucial skill in law school.

Why it’s important: The most important reason to develop effective listening skills is that reading the
book is not enough by itself. During class, professors
• add important information that the book does not explain,
• clarify points that were not clear before, and
• teach students how to apply what they have read and to write essay answers.

However, be aware that unlike in college classes, most law school professors DO
NOT attempt to teach you everything you need to know. You must learn most of
the material yourself, outside of class, through your reading.

Why you should have strategies for listening: Using strategies for listening will help you to organize and
remember the information that you are listening to.

Some strategies for listening:

For purely Socratic presentations . . .

1. DON’T let your attention wander. Be an active listener. When the professor asks a
question of another student, try to come up with the answer before the student does. (You can
think faster than anybody can talk.) If the student’s answer is wrong, see if you can figure out
why it’s wrong. If your answer was wrong, listen for the right answer.

2. For each question the professor asks, DO try to figure out what kind of information the
professor is talking about. Is the professor asking about the law – an element or rule? Is the
professor asking about the facts of the case, about the court’s reasoning, about an issue, or
about one of the other parts of your brief? If the professor is asking about one of these, it
should already be written in your brief. Find it and see whether what you wrote is complete
and accurate. Is the professor giving a hypothetical? If so, write down the hypothetical and
the professor’s analysis. (Today’s hypotheticals are tomorrow’s exam questions.)

3. DO take notes. It’s easy to get caught up when the discussion is lively (and it’s easy to shut
down if you let yourself get bored). Don’t forget to take notes! Even if you’re following and
understanding the discussion, short-term memory lasts for only a short time. After class, even
vivid memories fade quickly. If you haven’t taken notes, you lose the opportunity to transfer
the information into your long-term memory.

11
For both Socratic and non-Socratic presentations . . .

1. DO categorize the pattern of information the speaker is giving. Traditionally, speakers


use five main patterns when giving information: (1) a description, clarification, or main
idea/detail pattern; (2) a problem/solution pattern; (3) a cause-and-effect pattern; (4) a
sequence or chronological order pattern; and (5) a comparison/contrast pattern.

To use the pattern so that you can organize and retain the information, you must first
recognize the pattern. Professors use verbal clues to tell you what organizational pattern they
are using. They may also give visual clues to their pattern by writing the information on the
board.

Name of the Pattern Cues for the Pattern Nature of the Pattern
Now let’s discuss ________.
There are three parts of _____. The speaker names a topic,
Description, Classification, or The first part is . . . .The explains the parts of the topic,
Main Idea/Detail second part is . . . . The third and then gives details about
part the topic and the sub-parts.
is . . ..
The speaker suggests a
problem and then offers
The problem is _________.
solutions, perhaps also
Problem/Solution One solution is . . . . Another
offering advantages and
solution is . . . .
disadvantages for each
solution.

The speaker describes an


event (effect) and explains
The solution was . . . . This led
Cause and Effect what led to (caused) the event.
to . . . .
There may be multiple causes
and effects.

Sequence or Chronological First . . . . Then, . . . . The speaker lists events or


Order Later . . . . directions or dates.

_____ and _____ are similar The speaker explains how two
Comparison/Contrast in that . . . . However, they things are alike and/or how
differ in that . . . . they are different.

12
The sixth pattern, the pattern unique to law school: the legal-analysis pattern
In law school, there is a sixth pattern that you must listen for and take notes over: the legal-
analysis pattern. Professors in all classes routinely introduce the name of a concept or part of the
concept; describe what it means; describe facts, either from a case, a problem, or a hypothetical;
explain the reasoning or counter-arguments; and conclude. Essentially, the professor is writing
an exam answer in the air. The professor is demonstrating how he or she would expect you to
write an answer using that law and those facts.

Example: Here is what a Contracts professor, Professor Mara Kent, said in class when
discussing Problem 6 from the Contracts book.

Was this ad an offer that you could accept by paying for the tires? An offer must
be definite in its terms, which means that there were no terms left open for
negotiation. In this problem, the ad in the newspaper was for “major brand”
radial snow tires, and the ad gave the price. When you pay, what do you get?
Tires. But the ad doesn’t specify what kind, what type, or what size. A buyer
might be concerned about the type: the buyer might have an SUV, and these tires
might be for a car. This ad doesn’t give any details, so it isn’t definite. It is an
invitation to make an offer – an invitation to negotiate.

This professor was using this problem as an example of how to write an exam answer.

Issue: Was this ad an offer that you could accept by paying for the tires?

Element: An offer must be definite in its terms,

Rule: which means that there were no terms left open for negotiation.

Facts: In this problem, the ad in the newspaper was for “major brand” radial snow tires,
and the ad gave the price.

Reasoning: When you pay, what do you get? Tires. But the ad doesn’t specify what
kind, what type, or what size. A buyer might be concerned about the type: the buyer
might have an SUV, and these tires might be for a car. This ad doesn’t give any details,

Conclusion: so it isn’t definite. It is an invitation to make an offer – an invitation to


negotiate.

Listen for the legal-analysis pattern. You will hear it frequently. When you do, write down as
much of it as you can. Your professor is teaching you how to write an exam.

13
2. DO monitor your listening and see whether you have lost the speaker’s train of thought.
Asking yourself questions will help you to monitor your listening.

What is the speaker’s point?


Do I know what _______ means?
Does the information make sense to me?
How does this information relate to what was just said?

3. DO ask questions to clarify meaning. As you are listening, ask the professor questions
when you need to clarify meaning, eliminate confusion, or increase your understanding of
what is being said.

4. DO look for clues from the speaker. Speakers use both visual and verbal clues to convey
their message, direct their listeners’ attention, and emphasize certain ideas. Accomplished
speakers use these clues intentionally so that they can help their listeners, but the clues only
work if the listeners know to watch for them.

Visual Clues Verbal Clues


Using hand gestures Pausing
Writing information on the board Raising or lowering the voice
Changing facial expressions Slowing down to stress key points
Acting out events Repeating information
Moving closer to or further away from Using organizing words, such as first,
the audience second, third, then, next, another, in
contrast, etc.

5. DO take notes. This was listed above, under Socratic method, but it bears repeating. Note-
taking helps students become more active listeners because note-taking is a kind of
individual “responding-with-pen-in-hand.” Because you can’t store all the information in
your memory, note-taking is crucial. Note-taking does not have to be in an outline format. In
fact, everyone develops an individual note-taking style. But it is important to understand the
purpose of the speaker’s message (to compare, to list, to describe, etc.) to take good notes. If
your professor speaks very quickly so that you cannot keep up, try recording the class (with
permission from the professor). After class, you can go back and fill in the gaps in your
notes.

Questions like these will help you to monitor your listening effectively:

Before listening:
• What is the speaker’s purpose? (In a law school class, the topic of the assigned
reading will help you answer this one.)
• What is my purpose for listening? (To add to what I have read, to clarify what I have
read, etc.)
• What am I going to do with what I’m listening to?
• Do I need to take notes? (Usually, the answer is “yes.”)

14
• Which strategy should I use?
• Which strategy will I select?

While listening:
• Is my strategy working?
• Am I grouping information into categories?
• Is the speaker giving me visual clues about the organization of the message?
• Is the speaker giving me verbal clues about the organization of the message?
• Is the speaker’s voice, through pace or variation in tone, giving me clues?

After listening:
• Do I have questions?
• Are my notes complete?
• Did I make a good choice of strategies? Why or why not?

Credit to:
Tompkins, G. (1999). Language Arts. Englewood Cliffs, NJ: Merrill.

B. Note-taking: The Cornell System

1. Leave enough room on your brief so that you can take class notes onto the brief. This means
that you will want to separate the parts with as much white space as possible.

2. If the professor is lecturing in class, listen for the main ideas, sub-points, and hypotheticals.
If the professor is asking questions, listen to the questions. Most of the lecture will be about
the facts, the law, and the reasoning. Write your notes in a different color than the color you
used to write the brief. Identify where each of the professor’s points are in your brief. Write
your notes right onto the place in your brief where the information appears.

3. After class, go back through your notes and make the important information stand out by
underlining it, by numbering any lists, and by starring key points. If you have recorded the
class, listen to the recording and fill in what you missed.

4. Make an outline of the information from the class. You can use a chart form for your outline,
with headings like “Law,” “Cases,” and “Reasoning and Examples.” Or you can use a
traditional outline format that incorporates the same information.

5. Develop questions over the points in your outline. For example, “What are the elements of
battery?” “What are the three ways to prove intent for battery?” “What is the rule for
minors?”

6. Put that information into your Big Picture outline.

7. Keep your Big Picture outline current. Frequently recite and review the information in the
outline until you are certain that you understand and know all of it. Recite the information
out loud by stating the question out loud and then, from memory, reciting the information
that answers the question.

15
16
Week Two

Reading and Briefing Cases

17
18
Week 2 Pre-Reading Questions

(1) Beyond decoding words, what should reading involve?

(2) What are some active reading techniques? Why might you want to use them?

(3) What is “purpose setting”? How can you use the table of contents as a purpose-setting tool?

(4) What are some questions that you might want to ask yourself as you read a case?

(5) What are “recitation techniques”? Why might you want to use them? What kind of recitation
techniques could you use?

(6) What is a T-Chart? What does a T-Chart do that a traditional brief does not do?

(7) How can you use the notecases and problems in your book to further your understanding?

(8) Why should you translate what you read into your own words?

19
20
Getting the Most Out of What You Read
As you have noticed, law students have to read a lot. But the amount of reading is only one of the
problems law students face. Here are some others:

I. The language is often difficult, especially in older cases. The English language changes
constantly, and older cases often use familiar words in unfamiliar ways (for example, old
cases may use the word “occasioned” to mean “caused” – a meaning that many people have
never encountered before.) Old cases also use words that you may never have heard before,
or they may use words from other languages. But even newer sources are difficult: cases,
statutes, or court rules may contain long, complex sentences that are difficult to decode.

II. The law itself is like a language. It has its own vocabulary, and it’s not an easy vocabulary to
learn.

III. The law is full of concepts that are complex and completely abstract. Negligence, collateral
estoppel, intent – none of these concepts can be seen, heard, or touched. They exist only in
our minds. Yet you have to become as familiar with these concepts as you are with the layout
of your own home or the face of a loved one.

IV. Often, the topic that you’re reading about is something that you have no prior knowledge of,
or experience with. For example, if you’ve never been involved in a lawsuit, you may never
have encountered the concept of joinder of parties or venue. So when you’re reading about
unfamiliar concepts, you may have trouble deciding which information is important and how
the pieces fit together. It’s more difficult to learn about something completely new that it is
to learn more about something you already know about.

V. Many professors expect you to teach yourself the basics before you come to class. In class,
those professors may start out assuming that you have this basic information already; they
may start the discussion at the point where they assume you are, and then direct the
discussion upward from there. That works fine if you do, in fact, have that basic information.
But if you don’t, then class discussion may make no sense at all.

VI. Some law students have low reading comprehension and don’t know it. Some have
undiagnosed learning disabilities; some process information unusually slowly; and some
have poor reading strategies because they have never received proper reading instruction.
Some students have low reading comprehension because of emotional upset. Anxiety, anger,
or strong emotional states interfere with concentration, and a lack of concentration translates
into a lack of learning.

Strategies for Reading: How to Overcome Obstacles to Understanding

Reading is more than simply decoding words. “Reading is an interactive process in which a reader’s
prior knowledge of the subject and purposes for reading operate to influence what is learned from

21
text.”1 People who are active readers get much more from what they read than people who are
passive readers; active readers also build long-term memory more effectively. How can you tell
whether you are an active reader? Read the following active-reading strategies and see how many of
them you use.

Preparing to Read

Set your purposes for reading – what are you supposed to learn?

Look at the table of contents – your map of this course. Where does today’s assignment fall in this
map? How does today’s assignment fit with what has gone before? What specific topics will you
cover in today’s reading? What are you supposed to learn?

Get the big picture

Read about today’s topics in a hornbook, Nutshell, or other secondary source.

Ask questions

Scan the assignment and look for headings in the text. Turn those headings into questions. To form
the questions, you could think of the six question words: who, what, where, when, why, and how.
Then apply the appropriate words to the heading. For example, if the heading says, “Development of
Liability Based Upon Fault,” you could ask yourself, “What is fault? Whose fault are we concerned
with? How was liability decided before fault became important? What kinds of fault do courts look
for now?”

Reading

Read more than once

Very, very few people can read a case once and fully understand it. Instead, read the case at least
twice. There may be some parts that you have to read more than twice.

Use active reading techniques

The first time you read a case, try to get the basic idea of what’s going on in the case. The second
time, try to refine what you’ve read. As you read, use these techniques:

A. Translate what the court is saying – explain it in language that a twelve-year-old would
understand. To do this, “clump” the words in the sentence. That is, figure out which words fit
together to form a logical unit – a clump; then figure out what each clump means; then figure out
how those clumps fit together. Here’s an example from Garratt v. Dailey:

1
MICHAEL MCKENNA & RICHARD ROBINSON, Literacy Processes, TEACHING THROUGH TEXT: A
CONTENT LITERACY APPROACH TO CONTENT AREA READING, 22 (Longman Pubs. USA, 1997 ed.)

22
“A definition (not all inclusive but sufficient for our purpose) of a battery is the intentional
infliction of a harmful bodily contact upon another.”

If you “clump” these words, the sentence might look like this:

“A definition / (not all-inclusive / but sufficient for our purpose) / of a battery is / the
intentional / infliction / of a harmful / bodily contact upon another.”

Now translate the clumps.

A definition / (not all-inclusive / but sufficient for our purpose) / of a battery is / the

The court there are more but what the court is the legal
is giving rules that go giving is enough to concept is
law here. with battery decide this case. “battery.”

intentional / infliction / of a harmful / bodily contact upon another.

I have D has to D has to touching of P’s


to find “inflict” cause an body.
out what or actually injury
“intent” do
means something

So the point of this sentence is this: the court is giving some, but not all, elements or rules for
battery. But from this case we learn that to be liable for battery, D must (1) have intent, and (2)
actually do something that (3) causes an injury by (4) touching P’s body.

B. Underline or highlight any law that you see in the case. As soon as you find law, whether it is
from a case, a statute, a court rule, a constitutional provision, etc, asking yourself four questions
will help you develop the concepts within the law:

1. How many different ideas are contained in this explanation of the law?

2. How do these ideas fit together? (For example, do they form a list of elements? Is one
idea a definition of another part? Is there an exception?)

3. What does each idea mean?


Can you translate each idea into your own words?
Can you find a definition?

4. What would be an example of each idea? What would be a non-example of each idea?

The reason it is important to ask these questions about the law is that, unlike in most college
classes, you are not just reading for information. Instead, you are reading to learn concepts that

23
you must then be able to apply to new situations. Applying concepts to solve problems requires
you to understand the concepts more deeply than ever before.

C. Look for “word clues” – words or phrases that will enable you to predict what the issues are,
what the parties’ arguments will be, what the analysis will consist of, and what the outcome will
be. Be especially sensitive to connecting words like “and,” “or,” “however,” “unless,” etc. These
connecting words signal the relationships among ideas.

D. Revise your predictions when you get more clues about where the case is going.

E. Ask questions about what’s going on in the case and see whether you can find the answers. Here
are some questions you could ask:

What pieces of law did the court talk about? Which pieces of law did it actually use to make its
decision? Why did it use that law?

What are the important facts? Why were these facts important? What elements of the law did
these facts pertain to?

Why did the court decide the way it did? What steps did the court go through to make its
decision? Why were these steps necessary?

Why is this case here? What am I supposed to get out of it? What did it add to what I know?

F. Look up words that you don’t understand. Also, look up words that seem familiar, but are being
used in unfamiliar ways. Those words may have meanings that you’re not aware of. To establish
these words in your vocabulary, write unfamiliar words on one side of an index card. On the
other side, write the definition of the word, and use the word in a sentence or two. By the way – a
legal dictionary is not enough. Get a good regular dictionary, such as Webster’s.

G. Write notes in the margin as you read. Label the paragraphs with a word, phrase, or sentence
explaining what the point of each paragraph is. For example, next to a paragraph of facts, write
out the most important facts. Next to a paragraph explaining the defendant’s argument,
summarize that argument in a sentence or two. Next to a paragraph where the court is applying
the law to the facts, write a note summarizing what facts went with what elements (“no intent
because [fact], [fact], and [fact]”).

H. T-chart the case so that you have notes on all the essential information in the case. (See the
information on T-charting later in this course pack.)

I. Read the notecases (the short descriptions of cases that many casebook authors put after the
principal case). Sort out the information in the notecases and add it to your T-chart. If your
casebook includes questions or problems for you to solve, answer the questions and solve the
problems as best as you can.

24
Reciting

Reconstruct the whole

When you’ve finished the case for the first or second time, reconstruct the whole. That is, read your
marginal notes and, in your head or out loud, recite all the major steps that the court went through to
reach its decision. This will solidify the information in your memory and enable you to see the point of
that case.

Explain it to a layperson

Look away from the case and recite it. Pretend that you’re telling the story to someone you know well –
someone who is intelligent, but knows nothing about the law. As you tell the story, think about that
person’s reaction. Will that person understand what you’re saying? Can that person follow every step? If
you skip a step or you’re unclear, try again. If you can explain it so that an intelligent layperson would
understand it, then you know that you understand it.

Make up a story

Change the facts of the case and ask yourself these questions: “If I change this fact, how will it change
the analysis? What element does this new fact relate to? Will the new fact change the outcome? Why?”
Think of this as exam preparation that goes on all term. You know that, on the exam, the professor will
give you facts that you’ve never seen before. So get ready by making up your own facts and analyzing
them. If you do this throughout the term, then by the end of the term you will have taken scores of
“practice exams” that you made up yourself.

Relate the material to something you already know

Think how the material relates to an experience that you’ve had, an experience that a friend or family
member has had, or to something you’ve read about or seen on television. If you’re taking Contracts and
you’re reading about implied duty to perform a job in a workmanlike way, think about the story your
second cousin told you. You know, the story where the contractor who built his house forgot to hook up
the drain on the bathtub, so the first time your cousin drained the tub, the dining room ceiling caved in!
How would that implied duty apply to your cousin’s situation? What kind of remedy would he have
had? If you’re studying assault in Criminal Law or Torts, think about the guy you knew in college who
got drunk and got into a bar fight. How would these principles apply to him? If you relate material to
something you know, it is concrete instead of abstract. And the more associations you have between the
material and information you have, the more material will enter into your long-term memory.

Review

Go back to the table of contents

After you’ve finished reading the entire assignment, take a moment to reflect on what you’ve learned.
What new concepts did you learn about? How do they fit together? Then go back to the table of contents
and reconstruct what you’ve learned up until now in the course. How do these pieces fit together? Can
you see the big picture?

25
Conclusion

You won’t use every single technique listed here for every single case you read and every single
assignment. However, these techniques will maximize what you get from reading. And though you may
be thinking that these techniques will take forever, you will find that you will actually become more
efficient. Furthermore, the more you use these techniques, the easier it will become to do the deep
thinking and deep processing that true learning requires.

(By the way, many of these techniques are good to use in a study group.)

If you suspect that you’re having reading difficulties, please contact the ARC as soon as possible. We can
give you a diagnostic reading test that can detect basic reading deficiencies. If the test indicates a
problem, we can refer you for further testing.

26
Reading Cases Efficiently:
Two Examples

Objectives:

(1) To help you become an independent learner. Once you have finished law school, you will
have learned a lot of law. However, law school cannot teach you all the law you need to
know. In fact, nobody knows all the law. Therefore, lawyers must learn to teach themselves
the law. When a client come into a lawyer’s office with a problem, very often the lawyer must
go to the law books and find and learn the law that governs the client’s case. Very often the
law is found in judges’ opinions – court cases. So you must learn to learn by yourself – to read
cases, find the law, and understand how the law applies to different fact situations.

(2) To help you become more efficient. Reading the cases requires a whole series of skills, and
the cases themselves are written in a pattern that is likely to be unfamiliar to you. These things
will slow down your reading. Furthermore, you will be required to read and brief a huge
number of cases in law school, which takes a lot of time. But you can’t spend all your time on
reading and preparing for class; you also need to find time to create outlines, to recite and
review the material, and to practice essay writing and multiple choice. Therefore, you will
need to learn to read and brief cases efficiently so that you have time for the other things that
you need to do.

Be aware that at the beginning of your first semester, you will spend a lot of time reading and
briefing your cases. That is normal. However, over the course of the semester, you should
notice that you gradually become more efficient. If, toward the end of the semester, you notice
that you are still spending all your time reading and briefing, contact your Intro instructor or
the Academic Resource Center for help.

Overview of the Steps:

(1) Create the Big Picture. Use the table of contents or syllabus to get a Big Picture of the
course.

(2) Figure out where you are in the Big Picture. The syllabus tells you what pages to read for a
class. Sometimes the syllabus also tells you what topics you will be learning about.
Sometimes the syllabus does not tell you the topics, so you need to check the table of contents
in the casebook.

(3) Pre-read. Read about that topic in a hornbook, a Nutshell, or some other good supplemental
source. Don’t worry about trying to understand every detail for that topic. Instead, try to find
the main ideas: the elements and the major rules that govern that topic. This will give you the
Big Picture of that topic.

(4) Look for the law in the case. As you read a case for the first (or second) time, skim past the
facts until you find the law. Look at each paragraph in the case and ask yourself whether it
contains law. Cases usually (though not always) start with facts about the case – what
happened before and, sometimes, after the lawsuit began. After the court finishes with the

27
facts, it usually turns to law.

Sometimes the court will specifically tell you when it begins discussing law. It may use cue
words like “The law is clear that,” “Numerous authorities hold,” “It is elementary that,” etc.

Sometimes that court will not use specific cue words to tell you that it is now giving law. It
may help to remember that the law is not about anybody in particular, unlike the facts.
Instead, the law applies to everyone: it is likely to be a general statement. So to find the law,
you will need to look for statements that are not about the particular people in this case (“One
who,” “Any person who,” etc.)

The times when it is hardest to spot the law will be when the court has blended the law and the
reasoning together. Then you may have to figure out what the elements or rules are from what
the court reasons about the specific people in that case.

(5) Break the law down into component ideas. Sometimes a case contains only one piece of
law – for example, one rule; sometimes, a case contains lots of law. As you read a case (or a
statute or court rule or Restatement or any other source of law that you come across), you must
find the elements of the law. Break the law down as much as possible into a bullet-point list of
ideas, so that you can see the individual pieces. Try to pull every scrap of law out of the
source that you’re reading, and organize it.

To help you do this, ask yourself basic questions about the law and how the pieces of the law
fit together. Asking questions focuses your reading. Here are some important questions:

How many different ideas are contained in this explanation of the law?

How do these ideas fit together? (For example, do they form a list of elements? Is one
idea a definition of another part? Is there an exception?)

What does each idea mean?

Can you translate each idea into your own words?


Can you find a definition?

What would be an example of each idea? What would be a non-example of each idea?

Example 1: Here is an example from Criminal Law, Hicks v. Sheriff, Clark County. In Hicks, the
court alternated between law and reasoning. The following excerpt contains only the statements
of law.

28
Only after the corpus delicti has been proved by lawful evidence may confessions and admissions be
considered in establishing probable cause to show that the accused was the criminal agency causing the
death. *** “It is not requisite, however, that the crime charged be conclusively established by evidence
independent of the confession or admission. It is sufficient if there be other competent evidence tending
to establish the fact of the commission of the crime.”***

At the very least there must be established, independent of any confession or admission by the
accused, the fact of death and that it resulted from the criminal agency of another and not from natural
causes, accident or suicide.

How many separate ideas appear in this statement of law?

• Only after the corpus delicti has been proved by lawful evidence may confessions and
admissions be considered in establishing probable cause to show that the accused was the
criminal agency causing the death.

• It is not requisite, however, that the crime charged be conclusively established by evidence
independent of the confession or admission.

• It is sufficient if there be other competent evidence tending to establish the fact of the
commission of the crime.

• At the very least there must be established,

o independent of any confession or admission by the accused,

o the fact of death

o and that it resulted from the criminal agency of another and not from natural causes, accident
or suicide.

(6) Paraphrase. Putting the law into your own words, instead of the stuffy language the court
uses, ensures that you understand it. Do not paraphrase the “terms of art” – that is, the legal
terms that the court uses. After all, the terms of art are part of the legal language that you are
learning to speak. Instead, find definitions (rules) that explain those terms of art.

29
• The prosecutor must show the corpus delicti first, before confessions and admissions can be
used to show probable cause that the defendant was the criminal agency that caused the death.

• The prosecutor doesn’t have to show, before the confession or admission can be used, that the
crime is conclusively proved.

• Instead, it is enough if there is some other competent evidence tending to show that the crime
was committed.

• The prosecutor must prove

o Without using any confession or admission by the defendant

o That someone has died

o And that the death was caused by the criminal agency of another and not by natural causes,
accident, or suicide.

(7) Organize the law. Sometimes the court has repeated certain ideas. If you organize the
information, you can cut down on repetition. Sometimes the court has stated an element in
one place and has stated the definition (rule) in another place.

Put the definition with the element and write the definition under the word that’s being
defined. If you see a subrule or exception, write it under the rule that it goes with. And add
headings so that you can see what each part of the law is for.

Here are some questions you can ask yourself.

How do all the pieces fit together? Does the case give a list of elements and rules? Does
it give a rule and an exception? Does it give a variety of rules to be used in different
situations? And so on.

Note that, in the information above, the court has repeated several ideas: the prosecution has
to prove corpus delicti before using confessions or admissions; the court has set out rules on
how to prove corpus delicti; and corpus delicti requires proof of (1) death and (2) criminal
agency. Those are the main ideas, so that gives you three main categories. Then figure out
which ideas go in each category.

30
Why show corpus delicti?
If the state cannot show corpus delicti, defendant’s confessions and admissions cannot be used against
defendant.

How do you prove corpus delicti?


Use any admissible evidence other than a confession or admission by defendant.
The crime doesn’t have to be conclusively proved for corpus delicti. There just has to be some
evidence of the corpus delicti.

What are the elements of corpus delicti for murder?


Someone has died and
The death was caused by the criminal agency of another
(Rule for criminal agency) death was not by natural causes, accident, or suicide.

(8) Find the reasoning and match the parts of the reasoning with the parts of the law.
Usually, the reasoning is after the statement of the law. Sometimes, the court alternates
between law and reasoning (as in the Hicks case). For each reasoning statement, ask yourself
what element or rule the court is talking about. Often, the court will explain a lot of law, but
the court’s reasoning will concentrate on only one or two pieces of that law. Write the
court’s reasoning next to the piece of law that it fits. Ask yourself:

What was the court’s reasoning?

What elements or rules was the court concentrating on?

Here is some of the reasoning from the Hicks case.

Except in the affidavit of King, we find no testimony or other evidence about the cause of death
of Christiernsson. All that we find relating to his death is testimony that his body was found on
December 6, 1967, in the desert; that it was identified by a military service identification tag and a
thumb print, and that the body was partially clothed. There is absolutely no evidence before either
the justice’s court or the district court that a criminal agency of the appellant or anyone else was
responsible for the alleged victim’s death.

What element or rule is the court talking about? Look for words that give clues. The court
repeats words or phrases from the law or repeats ideas from the law, to help you to identify what the
court is reasoning about.

Except in the affidavit of King, we find no testimony or other evidence about the cause of death
of Christiernsson. All that we find relating to his death is testimony that his body was found on
December 6, 1967, in the desert; that it was identified by a military service identification tag and a
thumb print, and that the body was partially clothed. There is absolutely no evidence before either
the justice’s court or the district court that a criminal agency of the appellant or anyone else was
responsible for the alleged victim’s death.

31
What element is the court talking about? Criminal agency. Criminal agency has to do with the
cause of death – whether it was caused by another person, or by accident, suicide, or natural causes. So
you can store this piece of reasoning with the piece of law that it matches.

Reasoning Law

Why show corpus delicti?


If the state cannot show corpus delicti, defendant’s
confessions and admissions cannot be used against
defendant.

How do you prove corpus delicti?


Use any admissible evidence other than a confession
or admission by defendant.
The crime doesn’t have to be conclusively proved for
corpus delicti. There just has to be some evidence of
the corpus delicti.

What are the elements of corpus delicti for


murder?
Someone has died and

The death was caused by the criminal agency of


another.

Except in the affidavit of King, we find no testimony (Rule for criminal agency) death was not by
or other evidence about the cause of death of natural causes, accident, or suicide.
Christiernsson. All that we find relating to his death is
testimony that his body was found on December 6,
1967, in the desert; that it was identified by a military
service identification tag and a thumb print, and that
the body was partially clothed. There is absolutely no
evidence before either the justice’s court or the district
court that the criminal agency of the appellant or any-
one else was responsible for the alleged victim’s
death.

Here is another part of the court’s reasoning.

The affidavit of the appellant’s fellow prisoner to the effect that the appellant admitted to him that he had murdered
the victim, does not supply the proof necessary to show that death was caused by criminal means.

Where is the court now?

The affidavit of the appellant’s fellow prisoner to the effect that the appellant admitted to him that he had
murdered the victim, does not supply the proof necessary to show that death was caused by criminal means.

32
Where in the law does the court discuss whether an admission is enough proof? In the part that
explains how you prove the corpus delicti.

Reasoning Law

Why show corpus delicti?


If the state cannot show corpus delicti, defendant’s
confessions and admissions can’t be used against
defendant.

The affidavit of the appellant’s fellow prisoner to How do you prove corpus delicti?
the effect that the appellant admitted to him that he Use any admissible evidence other than a
had murdered the victim, does not supply the confession or admission by defendant.
proof necessary to show that death was caused by
criminal means.
The crime doesn’t have to be conclusively proved
for corpus delicti. There just has to be some
evidence of the corpus delicti.

What are some elements of corpus delicti for


murder?
Someone has died and
The death was caused by the criminal agency
of another.
Except in the affidavit of King, we find no
testimony or other evidence about the cause of (Rule for criminal agency) death was not
death of Christiernsson. All that we find relating by natural causes, accident, or suicide.
to his death is testimony that his body was found
on December 6, 1967, in the desert; that it was
identified by a military service identification tag
and a thumb print, and that the body was partially
clothed. There is absolutely no evidence before
either the justice’s court or the district court that a
criminal agency of the appellant or anyone else
was responsible for the alleged victim’s death.

(9) Summarize the reasoning. After all, you don’t have time to write out all those details. Nor
would you want to.

33
Law
Reasoning
Why show corpus delicti?
If the state cannot show corpus delicti, defendant’s
confessions and admissions can’t be used against
defendant.

How do you prove corpus delicti?


Affidavit of D’s cellmate said D had admitted the
Use and admissible evidence other than a confession
murder, but court can’t use the admission to prove
or admission by defendant.
the death was caused by criminal means.
The crime doesn’t have to be conclusively proved
for corpus delicti. There just has to be some
evidence of the corpus delicti.

What are the elements of corpus delicti for


murder?
Someone has died and
The death was caused by the criminal agency of
No evidence other than the affidavit that D caused
another
the death: body was found in the desert; ID’d by
(Rule for criminal agency) death was not by
dogtag and print; body partly clothed; but nothing
natural causes, accident, or suicide.
to show the cause of death.

(10) Read the facts. Now go back and carefully read the facts of the case. Keep the elements
and rules in mind as you read. When you see a fact, ask yourself whether it “fits” with a
piece of law. Some facts will fit; other will not. The facts that fit are legally relevant; facts
that do not fit law are legally irrelevant. And remember that, sometimes, the court only tells
part of the story in the “facts” part of the case and tells the rest of the story in the reasoning.
That is what the Hicks case does; it gives a few facts in the beginning of the case, and it adds
more facts during the reasoning.

Here are the questions you could ask yourself:

What are the basic facts?

Which facts “fit” each piece of law?

Which facts do not “fit” any piece of law? (In other words, which ones are not legally
relevant?)

34
Hicks v Sheriff, Clark County, Nevada Supreme Court, 1970

Reasoning Law Facts


Why show corpus delicti?
If the state cannot show corpus delicti,
defendant’s confessions and admissions can’t be
used against defendant.

Affidavit of D’s cellmate said D had admitted How do you prove corpus delicti? D’s cellmate wrote in an affidavit that D had
the murder, but the court can’t use the admission Use any admissible evidence other than a admitted beating victim to death and then
to prove the death was caused by criminal confession or admission by defendant. stabbing the victim.
means.
The crime doesn’t have to be conclusively
proved for corpus delicti. There just has to be
some evidence of the corpus delicti.

What are the elements of corpus delicti for


murder?

(1) Someone has died

35
(2) The death was caused by the criminal agency D and victim had been seen together before
No evidence other than the affidavit that D of another. victim’s disappearance. D was driving victim’s
caused the death: body was found in the desert; car when he was arrested.
ID’d by dog tag and print; body partly clothed; (Rule for criminal agency) death was not by
but nothing to show the cause of death. If D had natural causes, accident or suicide.
beaten and stabbed the victim, then victim’s
body would have bruises, wounds, fractures or
something. And evidence connecting D to victim
is not enough for corpus delicti without evidence
that the death was not by natural causes

At this point, you have produced a T-chart. T-charting is a technique for taking notes over the material that you read. It is a system that helps you to
break the law down into small parts, understand the parts, see how the parts fit together, and sort out what kinds of facts relate to each part. Because the
T-chart is a picture of what your brain should be doing when it reads the law, using a T-chart is a way to make the thinking process visible. And once
you’re conscious of what your mind should be doing, the T-chart makes your mind work more efficiently.
(11) Figure out the issue. Once you have sorted out the law, reasoning, and facts, the issue
should stand out. In Hicks, the court spent all its time discussing one major idea: whether
there was any evidence to show criminal agency other than D’s confession to his cellmate.
So that would be the issue: whether the corpus delicti can be proved when there is a dead
body and D has admitted the murder, but when there is no evidence on how the victim died.

Notice the structure of the issue. The general formula for writing an issue is:

Whether [element/rule is broken/met] where [brief statement of core facts]

(12) Figure out the other parts of the brief. There are other aspects of the case that you should
try to understand and that your professor may want to ask you about: what the procedure
was, what each side would argue in that case, what the holding was, etc. Find answers to
those questions and write them down. If you are using the T-chart, put them wherever you
have room on the page.

36
Example 2: Garratt v Dailey

Reasoning Law Facts

This 5-year-old can be liable for intentional torts Can minors be held liable for intentional 5-year-old
just as grownups can, if the elements are torts?
fulfilled. In many jurisdictions, yes.

Elements of battery

Intent – two ways to prove

(1) Intent (purpose) to cause the contact or P had argued that the kid moved the chair so that
she would fall. But trial judge held that he didn’t
plan to make her fall.

If the kid knew that the woman moved too (2) Knowledge that the contact is substantially On appeal, P argued that the trial judge erred by
slowly to react when he moved the chair and that certain to occur. (Mere negligence or even not considering the second way to prove intent.

37
it was substantially certain that she would fall recklessness isn’t enough) D had moved a lawn chair that an elderly,
when he moved the chair, he would be liable. arthritic woman had been sitting in; she didn’t
notice, and she started to sit down in it.

Holding: Trial judge erred in not considering the


second way to prove intent. Remanded for
findings on that. On remand, trial judge held that
the kid had known and held for P.

Contact with another She fell to the ground

Harmful or offensive Broke her hip


In this example, notice that the law has been broken down vertically. There are two areas of law
discussed; (1) what the rule is when the defendant is a child and (2) the elements of battery and the rules
that explain the intent element. The facts are arranged to fit next to the piece of law that those facts fit:
“5-year-old kid” goes with the rules for minors because those facts show that he was minor. “Moved a
lawn chair that an elderly, arthritic woman had been sitting in; she didn’t notice, and she started to sit
down in it” goes next to “knowledge that the contact is substantially certain to occur” because those are
the facts that the child knew, and they should have told him that she was likely to end up falling. There
is reasoning next to both of these rules because those were the rules that the court was discussing.

Notice also that you can tell what the issue was in the case by looking at the chart. The court
spent no time discussing whether contact occurred or whether it was harmful. Instead, it spent its time
on two rules only. So those rules were the two issues. (1) Can a small child be liable for an intentional
tort? (2) Can a person have the intent required for battery when the person didn’t actually act with the
purpose of causing the contact? So these notes will show you what this case adds to what you’ve
already learned, and they will help you to see why this case was included in your assignment.

Using the T-chart as a Substitute for Traditional Briefs

Traditional briefs have certain advantages: briefs make you practice writing complete sentences
to explain legal topics, which is valuable; they also are easy to recite from because you can just read
them out loud. However, they have some drawbacks.

1. The law is buried in the brief; when it’s time to write your outline, you have to dig the law back out.

2. The brief is a block of text; therefore, you can’t add anything to the middle of it.

3. It can be difficult to see the relationship between different ideas in the brief.

4. Briefs take a long time to write.

You can use the T-chart as an effective substitute for traditional briefs. Not only will the T-chart
solve some of the problems inherent in traditional briefing, but it will also (1) help you to organize the
material in your mind so that you can put it into your long-term memory; (2) help you to spot the main
point of the case; (3) create an outline of the material that will help you follow along in class; and (4)
pre-organize the material, which will make it much easier to create an outline. Here’s one way to do it.

Write out your T-chart. State the issue at the top of the chart. Put any important procedure into
the “facts” column. Include the parties’ arguments. In short, find a place for the parts that you would
put into a traditional brief.

38
Issues: Can a small child beheld liable for an intentional tort? Can a person have the intent required for battery when the person
didn’t actually intend to cause the contact?
Reasoning Law Facts

This 5-year-old can be liable for Can minors be held liable for 5-year-old kid
intentional torts just a grownups can, if intentional torts?
the elements are fulfilled. In many jurisdictions, yes. Procedure: P had argued that the kid
moved the chair so that she would fall.
But trial judge held that he didn’t intend
Elements of battery to make her fall.

Intent – 2 ways to prove


Intent (purpose) to cause the contact or On appeal P argued that trial judge erred
by not considering the second way to
prove intent.
If the kid knew that the woman moved Knowledge that the contact is
too slowly to react when he moved the substantially certain to occur The kid had moved a lawn chair that P, an

39
chair and that it was substantially certain (Mere negligence or even elderly, arthritic woman had been sitting
that she would fall when he moved the recklessness isn’t enough) in; she didn’t notice and started to sit
chair, he would be liable. Holding: Trial down in it.
judge erred in not considering the second
way to prove intent. Remanded for
findings on that. On remand, trial judge
held that the kid had known and held for
P. Cause contact with another She fell to the ground

Harmful or offensive Broke her hip

Notice that, in the chart, there is no discussion next to the elements of “contact” or “harmful or offensive.” Instead, the discussion
clusters next to two points: (1) whether a child can be liable for an intentional tort and (2) intent. This shows you, therefore, that
the case was meant to teach you those two points. Knowing this makes it easier to state the issues in the case (see the issues at the
top of the chart).
Using the T-chart to recite

When you are called on to recite, your professor may require you to state all of the facts of the
case, and not just the legally relevant ones. The T-chart usually only contains the legally relevant facts
because it requires you to sort out which facts go with each piece of law. Facts that do not fit a piece of
law are not included. Furthermore, the facts in the T-chart are often not listed chronologically because
they are sorted to the piece of law they fit. If your professor wants you to recite all the facts
chronologically, you can try one of two techniques:

First, you could highlight the facts in your casebook and recite the facts by looking at the
casebook. The advantage of doing it this way is that you don’t have to write the facts out, which
saves time.

Or, second, you can write out a paragraph of chronological facts on a separate sheet of paper or
in a blank corner of the T-chart.

Using the T-chart in Class

To use the T-chart as an effective tool in class, you could set up a notebook this way: Put the T-
chart on the two pages of your notebook. Draw two columns on the left-hand page: one for the court’s
reasoning and one for your notes on what is said about the case in class. Draw two columns on the
right-hand page: one for the law and one for the facts. Use different-colored inks for what you write
before class and what you write in class. That way, you can tell the difference between (1) what you
picked up on when you read the case and (2) what your professor had to say about the case.

(Left side of notebook) (Right side of notebook)

Class Notes Reasoning Law Facts

Think of the notes you take in class as a confirmation of what you learned as you briefed your
cases. The professor wants to make sure that you understand the law and how the law is applied to
facts; therefore, if the class discussion confirms that the notes you took are accurate, then you know that
you’re on the right track.

The notes in your T-chart and in the “Reasoning” column will also form an excellent outline to
use in class. Professors tend to be very interested in the details of the law (in the “Law” column), in the
facts (in the “Facts” column), and in the court’s reasoning (in the “Reasoning” column). If you have a
professor who jumps from topic to topic, the T-chart makes it easier to see what the professor is
discussing now. For example, if the professor begins discussing the element of “contact,” you can write
your notes across from the word “contact” in the T-chart (or you can put your notes right in the T-chart
itself). If the professor gives you a hypothetical, you can write it into the “facts” column next to the
element that the hypothetical relates to. If the professor explains something about the reasoning, you

40
can write your notes next to that part of the reasoning on your chart. In short, you’ll be more able to
follow where the professor is going.

Summary of How to Read Cases


The first step to effective, efficient reading of cases is to understand what your brain must do.
The second step is to practice. The third step is to check your progress.

What your brain must do:

Reading cases effectively is about selecting and sorting information.

To select information, you must think about the important categories of information as you read:
law, reasoning, facts, issues, procedure, etc.

To sort information, you must break all the law into the separate ideas, figure out the
relationships among the pieces, and then match reasoning and facts with the pieces of law.

Practice:

You don’t have much choice about whether to practice because your professor assigns plenty of
cases to practice on!

Check your progress:

The easiest way to check your progress is to listen in class. Does your professor bring out
important things that you missed? If so, go back to the case and see why you missed those
things. If your brief contains all important points, then pat yourself on the back!

41
42
Reading and Analyzing Notecases, Problems, Etc. in the Casebook

Some casebooks have notecases or questions or both after the principal cases. The Torts book is one
example. Some casebooks have problems before or after the principal case. The Contracts and Property
books are examples.

Notecases: Notecases usually serve one of three purposes. They


1. explain or clarify points from the principal case;
2. supplement the law given in the principal case (in other words, notecases may
teach you law that the principal case did not mention); or
3. set out additional fact patterns for you to analyze.

First ask yourself which purpose the notecase serves.

If the notecase explains or clarifies points from the principal case, then
1. ask yourself which part of the law from the principal case the notecase is
addressing (the point of law is usually already in your brief);
2. ask yourself exactly how the notecase is clarifying the law;
3. write notes on your brief. On the part of the brief where that law appears,
write in the explanation or clarification that you got from the notecase.

If the notecase supplements the law given in the principal case, then
1. ask yourself how the new piece of law fits. Does the notecase give you an
element that the court had not mentioned in the principal case? Then write
that element into your brief. Does the notecase give a definition for an
element (the rule)? Then write that rule next to the element in your brief.
Does the notecase give you information about a different area of law? Then
take notes about that law and, when you are writing your Big Picture outline,
ask where that law should go in the outline.

If the notecase sets out a fact pattern for you to analyze, then
1. use the law you have learned to analyze the facts given and to reach a
conclusion; and
2. think about which piece of the law those facts are designed to make you
particularly think about. Usually, in the short fact patterns in notecases, some
of the parts of the law are obviously met and other parts are more problematic.
Then the purpose of that notecase is to make you think particularly about the
piece of law that is the real problem. This will help you on exams because, if
an exam question contains facts similar to the facts in the notecase, then you
will have already analyzed something like it. This will make it easier for you
to quickly and accurately do the analysis on the exam question because you
will have analyzed a problem like that before.

Problems and questions: Problems and questions in the casebook have one major purpose:
to make you think about and understand the law in that part of the book and to see how that
law would apply. If the casebook contains a problem or question, then
1. think about what law you have learned;
2. use that law to answer the question; and
3. explain to yourself exactly why you reached that answer.

43
After Class

Organize, Clarify, and Add to Your Notes

After class, you should take a break – obviously. But as soon as possible after class, go back
over your notes. Recopy illegible parts, rephrase points that are unclear, add details that you
remember but didn’t have time to write down, insert punctuation, number sub-points and so
on.

Add headings to your notes so that you can see instantly what topic was being discussed. If
the professor discussed a point but you weren’t sure what topic the professor was discussing,
see whether you can figure it out as you’re going through your notes. Once you do figure it
out, write a heading next to that part of your notes, stating what the topic was.

If you have questions about part of the material, write them next to the notes on that material.
Remember to ask the professor, the TA, or your study partners about those questions.

Translating – How to Find the Words

Paraphrasing (translating the court’s language into your own words) is a difficult skill. Yet it
is an important skill – one that you will use when briefing or T-charting, when reciting in
class, when writing a memorandum in Research & Writing, and when writing an exam. Here
are some suggestions that may help.

If you’re having trouble translating the court’s language into your own words, try
looking up terms in a legal dictionary or a thesaurus. If you find a synonym or a
phrase that will accurately explain the term’s meaning, then you can attain a deeper
understanding of the term. Finding an antonym for the term will also help.
Understanding what something is not is as important as understanding what
something is.

Here’s a simple, non-legal example. Say the word you need to understand is “large.”
Other words that mean “large” include “big,” “huge,” “immense,” and “enormous.”
Antonyms include “small,” “tiny,” and “little.”

Antonyms – large is not Synonyms – large is

Less tiny, little (in between, you might big, immense More

small put words like “medium”) huge, enormous

Here’s a legal example. One element of false imprisonment is “restraint.” Your first
idea of “restraint” might begin with the notion that “restraint” means being held in a jail cell.
However, your reading will further refine your understanding: “restraint” means being
confined in any kind of area. Finding synonyms will help you to “scale” the idea of restraint
– that is, to decide exactly how far the concept of “restraint” will extend.

44
Antonyms – restraint is not Synonyms – restraint is

Less freed (in between, you might held, contained More

loose put words like “hindered”) confined

45
TABLE OF CONTENTS

Acknowledgments xxvii
Introduction to the Study of the Law of Contracts xxix

CHAPTER

1
INTENT TO CONTRACT:
OFFER AND ACCEPTANCE

I. Introduction: The Principle of Mutual Assent 1


Problem 1 1
Lucy v. Zehmer 1
Note 8
Problem 2 8
Problem 3 8
Note on Social Contracts 9
Problem 4 10
Note on the Concept of Misunderstanding 10
II. The Offer 10
A, Preliminary Negotiations 10
Leeds v. First Allied Connecticut Corp. 10
B. Statement of Opinion or Intention 20
Cirafici v. Goffen 20
Problem 5 24
Problem 6 24
ix

46
Table of Contents

C. Solicitations 24
Problem 7 24
Lefkowitz v. Greater Minneapolis Surplus Store, Inc. 25
Note on Deceptive Practices 27
Problem 8 28
Problem 9 28
D. Written Contract to Follow 29
Connecticut Laboratories v. Scott Paper Co. 29
Notes and Questions 33
Problem 10 33
III. Acceptance 34
A. Effect of Acceptance 34
Problem 11 34
ProCD, Inc. v. Zeidenberg 35
B. Manifesting Assent 42
Restatement (Second) of Contracts §50 42
Beard Implement Co. v. Krusa 42
Notes and Questions 50
Fujimoto v. Rio Grande Pickle Co. 50
Problem 12 54
C. Silence as acceptance 55
Problem 13 55
Day v. Catan 55
Note and Questions 57
Problem 14 58
Problem 15 59
D. Knowledge of the Offer 59
Problem 16 59
E. Motive 60
Problem 17 60
F. Mode of Acceptance 61
Davis v. Jacoby 61
Questions 68
Problem 18 69
Problem 19 69
IV. Termination of the Power of Acceptance 70
Restatement (Second) of Contracts §36 70
A. Revocation by Offeror 70
Problem 20 70
Dickinson v. Dodd 71
Questions 74
Petterson v. Pattberg 75
Questions 79
Problem 21 80

47
Table of Contents

Marchiondo v. Scheck 82
Note on Revocation and Option Contracts 85
Problem 22 85
Problem 23 86
Problem 24 86
B. Lapse of Time 87
Restatement (Second) of Contracts §41 87
Loring v. City of Boston 87
Question 91
Problem 25 91
Problem 26 91
Phillips v. Moor 92
Note on the Risk of Loss 94
C. Termination by Death or Incapacity of the Offeror
or Offeree 95
Swift & Co. v. Smigel 95
D. Termination by Rejection 100
Problem 27 100
Restatement (Second) of Contracts §38 100
E. The “Mail Box” rule 101
Morrison v. Thoelke 101
Problem 28 108
Note on Option Contracts 108
Note on Offers in International Sales 109
F. Termination by Counteroffer and the “Battle of
the Forms” 109
Livingstone v. Evans 109
Problem 29 111
C. Itoh & Co. (America) Inc. v. Jordan
International Co. 112
Notes and Questions 119
Problem 30 122
Gardner Zemke Co. v. Dunham Bush, Inc. 123
Problem 31 132
Problem 32 132
Note on the Battle of the Forms in Other
Areas 133
Problem 33 133
V. Indefiniteness 133
Corbin on Contracts §29 134
Walker v. Keith 134
Notes and Questions 142
Rego v. Decker 142
Question 146

48
Table of Contents

Problem 34 146
Problem 35 146

CHAPTER

2
CONSIDERATION

I. The Basic Concept 149


A. Definition 149
Adam Smith, The Wealth of Nations 149
Problem 36 150
Corbin on Contracts §110 150
Restatement (Second) of Contracts §71 150
Hamer v. Sidway 151
Questions 154
Problem 37 155
B. Sufficiency 155
Problem 38 155
C. Adequacy of Consideration 156
Batsakis v. Demotsis 156
Questions 159
Schnell v. Nell 159
Question 161
Problem 39 161
Restatement (Second) of Contracts §87 162
II. Forbearance as Consideration 163
Fiege v. Boehm 163
Restatement (Second) of Contracts §74 168
Problem 40 168
Problem 41 169
Problem 42 169
III. The Illusory Promise 170
Wood v. Lucy, Lady Duff-Gordon 170
Note 171
Sylvan Crest Sand & Gravel Co. v. United States 172
Question 175
Problem 43 176
McMichael v. Price 176
Problem 44 180
Corbin on Contracts §156 181

49
CHAPTER

1
INTENT TO CONTRACT:
OFFER AND ACCEPTANCE

I. INTRODUCTION: THE PRINCIPLE


OF MUTUAL ASSENT

At the very heart of contract law is the determination of the parties’ intent to
contract. A showing of mutual assent is necessary for an enforce-able contract. In
determining whether there is mutual assent, courts typically ask whether there has been
an offer to contract and an acceptance of that offer. If the offeror has clearly manifested a
willingness to enter into a contract in such a that the other party, the offeree, knows that
assent is all that is necessary to cement the deal, and the offeree accepts, the requisite
mutual assent exists.

Problem 1

Should a court find there is the requisite mutual assent if the response to the following
statements is “I accept”?
“I’m considering selling my car to you for $1,200.”
“I will sell you my car for $1,200.”
“Would it be a good deal if I sold you my car for $1,200?”
“You wouldn’t consider paying $1,200 for my car, would you?”

LUCY v. ZEHMER
Supreme Court of Virginia. 1954
196 Va. 493, 84 S.E.2d 516.

BUCHANAN, Justice.
This suit was instituted by W.O. Lucy and J.C. Lucy, complainants, 1

50
2 Chapter 1. Intent To Contract: Offer and Acceptance

against A.H. Zehmer and Ida S. Zehmer, his wife, defendants, to have specific
performance of a contract by which it was alleged the Zehmers had sold to W.O. Lucy a
tract of land owned by A.H. Zehmer in Dinwiddie county containing 471.6 acres, more or
less, known as the Ferguson farm, for $50,000. J.C. Lucy, the other complainant, is a
brother of W.O. Lucy, to whom W.O. Lucy transferred a half interest in his alleged
purchase.
The instrument sought to be enforced was written by A.H. Zehmer on December
20, 1952, in these words: “We hereby agree to sell to W.O. Lucy the Ferguson Farm
complete for $50,000.00, title satisfactory to buyer," and signed by the defendants, A.H.
Zehmer and Ida S. Zehmer.
The answer of A.H. Zehmer admitted that at the time mentioned W.O. Lucy offered
him $50,000 cash for the farm, but that he, Zehmer, considered that the offer was made in
jest; that so thinking, and both he and Lucy having had several drinks, he wrote out “the
memorandum” quoted above and induced his wife to sign it; that he did not deliver the
memorandum to Lucy, but that Lucy picked it up, read it, put it in his pocket, attempted
to offer Zehmer $5 to bind the bargain, which Zehmer refused to accept, and realizing for
the first time that Lucy was serious, Zehmer assured him that he had no intention of
selling the farm and that the whole matter was a joke. Lucy left the premises insisting that
he had purchased the farm.
Depositions were taken and the decree appealed from was entered holding that the
complainants had failed to establish their right to specific performance, and dismissing
their bill. The assignment of error is to this action of the court.
W. O. Lucy, a lumberman and farmer, thus testified in substance: He had known
Zehmer for fifteen or twenty years and had been familiar with the Ferguson farm for ten
years. Seven or eight years ago he had offered Zehmer $20,000 for the farm which
Zehmer had accepted, but the agreement was verbal and Zehmer backed out. On the night
of December 20, 1952, around eight o'clock, he took an employee to McKenney, where
Zehmer lived and operated a restaurant, filling station and motor court. While there he
decided to see Zehmer and again try to buy the Ferguson farm. He entered the restaurant
and talked to Mrs. Zehmer until Zehmer came in. He asked Zehmer if he had sold the
Ferguson farm. Zehmer replied that he had not. Lucy said, "I bet you wouldn't take
$50,000.00 for that place." Zehmer replied, “Yes, I would too; you wouldn't give fifty.”
Lucy said he would and told Zehmer to write up an agreement to that effect. Zehmer took
a restaurant check and wrote on the back of it, “I do hereby agree to sell to W. O. Lucy
the Ferguson Farm for $50,000 complete.” Lucy told him he had better change it to "We"
because Mrs. Zehmer would have to sign it too. Zehmer then tore up what he had written,
wrote the agreement quoted above and asked Mrs. Zehmer, who was at the other end of
the counter ten or twelve feet away, to sign it. Mrs. Zehmer said she would for $50,000
and signed it. Zehmer brought it back and gave it to Lucy, who

51
I. Introduction: The Principle of Mutual Assent 3

offered him $5 which Zehmer refused, saying, "You don't need to give me any money,
you got the agreement there signed by both of us."
The discussion leading to the signing of the agreement, said Lucy, lasted thirty or
forty minutes, during which Zehmer seemed to doubt that Lucy could raise $50,000.
Lucy suggested the provision for having the title examined and Zehmer made the
suggestion that he would sell it “complete, everything there," and stated that all he had
on the farm was three heifers.
Lucy took a partly filled bottle of whiskey into the restaurant with him for the
purpose of giving Zehmer a drink if he wanted it. Zehmer did, and he and Lucy had one
or two drinks together. Lucy said that while he felt the drinks he took he was not
intoxicated, and from the way Zehmer handled the transaction he did not think he was
either.
December 20 was on Saturday. Next day Lucy telephoned to J.C. Lucy and
arranged with the latter to take a half interest in the purchase and pay half of the
consideration. On Monday he engaged an attorney to examine the title. The attorney
reported favorably on December 31 and on January 2 Lucy wrote Zehmer stating that the
title was satisfactory, that he was ready to pay the purchase price in cash and asking when
Zehmer would be ready to close the deal. Zehmer replied by letter, mailed on January 13,
asserting that he had never agreed or intended to sell.
Mr. and Mrs. Zehmer were called by the complainants as adverse witnesses.
Zehmer testified in substance as follows:
He bought this farm more than ten years ago for $11,000. He had had twenty-five
offers, more or less, to buy it, including several from Lucy, who had never offered any
specific sum of money. He had given them all the same answer, that he was not interested
in selling it. On this Saturday night before Christmas it looked like everybody and his
brother came by there to have a drink. He took a good many drinks during the afternoon
and had a pint of his own. When he entered the restaurant around eight-thirty Lucy was
there and he could see that he was "pretty high." He said to Lucy, “Boy, you got some
good liquor, drinking, ain't you?” Lucy then offered him a drink. “I was already high as a
Georgia pine, and didn't have any more better sense than to pour another great big slug
out and gulp it down, and he took one too.”
After they had talked a while Lucy asked whether he still had the Ferguson farm.
He replied that he had not sold it and Lucy said, “I bet you wouldn't take $50,000.00 for
it.” Zehmer asked him if he would give $50,000 and Lucy said yes. Zehmer replied, “You
haven't got $50,000.00 in cash.” Lucy said he did and Zehmer replied that he did not
believe it. They argued “pro and con for a long time,” mainly about “whether he had
$50,000 in cash that he could put up right then and buy that farm.”
Finally, said Zehmer, Lucy told him if he didn't believe he had $50,000, “you sign
that piece of paper here and say you will take $50,000.00 for the farm.” He, Zehmer,
“just grabbed the back off of a guest check there” and

52
4 Chapter 1. Intent to Contract: Offer and Acceptance

wrote on the back of it. At that point in his testimony Zehmer asked to see what he had
written to "see if I recognize my own handwriting." He examined the paper and
exclaimed, “Great balls of fire, I got 'Firgerson' for Ferguson. I have got satisfactory
spelled wrong. I don't recognize that writing if I would see it, wouldn't know it was
mine."
After Zehmer had, as he described it, “scribbled this thing off,” Lucy said, “Get
your wife to sign it.” Zehmer walked over to where she was and she at first refused to
sign but did so after he told her that he “was just needling him [Lucy], and didn't mean a
thing in the world, that I was not selling the farm.” Zehmer then “took it back over there
… and I was still looking at the dern thing. I had the drink right there by my hand, and I
reached over to get a drink, and he said, ‘Let me see it.’ He reached and picked it up, and
when I looked back again he had it in his pocket and he dropped a five dollar bill over
there, and he said, ‘Here is five dollars payment on it.’ … I said, 'Hell no, that is beer and
liquor talking. I am not going to sell you the farm. I have told you that too many times
before.’ ”
Mrs. Zehmer testified that when Lucy came into the restaurant he looked as if he
had had a drink. When Zehmer came in he took a drink out of a bottle that Lucy handed
him. She went back to help the waitress who was getting things ready for next day. Lucy
and Zehmer were talking but she did not pay too much attention to what they were
saying. She heard Lucy ask Zehmer if he had sold the Ferguson farm, and Zehmer replied
that he had not and did not want to sell it. Lucy said, “I bet you wouldn't take $50,000
cash for that farm,” and Zehmer replied, “You haven't got $50,000 cash.” Lucy said, “I
can get it.” Zehmer said he might form a company and get it, “but you haven't got
$50,000.00 cash to pay me to-night.” Lucy asked him if he would put it in writing that he
would sell him this farm. Zehmer then wrote on the back of a pad, “I agree to sell the
Ferguson Place to W. O. Lucy for $50,000.00 cash.” Lucy said, “All right, get your wife
to sign it.” Zehmer came back to where she was standing and said, “You want to put your
name to this?” She said “No,” but he said in an undertone, “It is nothing but a joke,” and
she signed it.
She said that only one paper was written and it said: “I hereby agree to sell,” but the
“I” had been changed to “We”. However, she said she read what she signed and was then
asked, “When you read ‘We hereby agree to sell to W. O. Lucy,’ what did you interpret
that to mean, that particular phrase?” She said she thought that was a cash sale that night;
but she also said that when she read that part about “title satisfactory to buyer” she
understood that if the title was good Lucy would pay $50,000 but if the title was bad he
would have a right to reject it, and that that was her understanding at the time she signed
her name.
On examination by her own counsel she said that her husband laid this piece of
paper down after it was signed; that Lucy said to let him see it, took it, folded it and put it
in his wallet, then said to Zehmer, “Let me give you $5.00,” but Zehmer said, “No, this
is liquor talking. I don't want

53
I. Introduction: The Principle of Mutual Assent 5

to sell the farm, I have told you that I want my son to have it. This is all a joke.” Lucy
then said at least twice, “Zehmer, you have sold your farm,” wheeled around and started
for the door. He paused at the door and said, “I will bring you $50,000.00 tomorrow. . . .
No, tomorrow is Sunday. I will bring it to you Monday.” She said you could tell
definitely that he was drinking and she said to her husband, “You should have taken him
home,” but he said, ‘Well, I am just about as bad off as he is.”
The waitress referred to by Mrs. Zehmer testified that when Lucy first came in “he
was mouthy.” When Zehmer came in they were laughing and joking and she thought they
took a drink or two. She was sweeping and cleaning up for next day. She said she heard
Lucy tell Zehmer, “I will give you so much for the farm,” and Zehmer said, “You haven't
got that much.” Lucy answered, “Oh, yes, I will give you that much.” Then “they jotted
down something on paper … and Mr. Lucy reached over and took it, said let me see it.”
He looked at it, put it in his pocket and in about a minute he left. She was asked whether
she saw Lucy offer Zehmer any money and replied, “He had five dollars laying up there,
they didn't take it.” She said Zehmer told Lucy he didn't want his money “because he
didn't have enough money to pay for his property, and wasn't going to sell his farm.”
Both of them appeared to be drinking right much, she said.
She repeated on cross-examination that she was busy and paying no attention to
what was going on. She was some distance away and did not see either of them sign the
paper. She was asked whether she saw Zehmer put the agreement down on the table in
front of Lucy, and her answer was this: "Time he got through writing whatever it was on
the paper, Mr. Lucy reached over and said, 'Let's see it.' He took it and put it in his
pocket," before showing it to Mrs. [*500] Zehmer. Her version was that Lucy kept raising
his offer until it got to $50,000.
The defendants insist that the evidence was ample to support their contention that
the writing sought to be enforced was prepared as a bluff or dare to force Lucy to admit
that he did not have $50,000; that the whole matter was a joke; that the writing was not
delivered to Lucy and no binding contract was ever made between the parties.
It is an unusual, if not bizarre, defense. When made to the writing admittedly
prepared by one of the defendants and signed by both, clear evidence is required to
sustain it.
In his testimony Zehmer claimed that he “was high as a Georgia pine,” and that the
transaction “was just a bunch of two doggoned drunks bluffing to see who could talk the
biggest and say the most.” That claim is inconsistent with his attempt to testify in great
detail as to what was said and what was done. It is contradicted by other evidence as to
the condition of both parties, and rendered of no weight by the testimony of his wife that
when Lucy left the restaurant she suggested that Zehmer drive him home. The record is
convincing that Zehmer was not intoxicated to the extent of being unable to comprehend
the nature and consequences of the instrument he

54
6 Chapter 1. Intent to Contract: Offer and Acceptance

executed, and hence that instrument is not to be invalidated on that ground. C.J.S.
Contracts, §, 133, b., p. 483; Taliaferro v. Emery, 124 Va. 674, 98 S.E. 627. It was in
fact conceded by defendants' counsel in oral argument that under the evidence Zehmer
was not too drunk to make a valid contract. . . .
The appearance of the contract, the fact that it was under discussion for forty
minutes or more before it was signed; Lucy's objection to the first draft because it was
written in the singular, and he wanted Mrs. Zehmer to sign it also; the rewriting to meet
that objection and the signing by Mrs. Zehmer; the discussion of what was to be included
in the sale, the provision for the examination of the title, the completeness of the
instrument that was executed, the taking possession of it by Lucy with no request or
suggestion by either of the defendants that he give it back, are facts which furnish
persuasive evidence that the execution of the contract was a serious business transaction
rather than a casual jesting matter as defendants now contend.
On Sunday, the day after the instrument was signed on Saturday night, there was a
social gathering in a home in the town of McKenney at which there were general
comments that the sale had been made. Mrs. Zehmer testified that on that occasion as she
passed by a group of people, including Lucy, who were talking about the transaction,
$50,000 was mentioned, whereupon she stepped up and said, “well, with the high-price
whiskey you were drinking last night you should have paid more. That was cheap.” Lucy
testified that at the time Zehmer told him that he did not want to “stick” him or hold him
to the agreement because he, Lucy, was too tight and didn’t know what he was doing, to
which Lucy replied that he was not too tight; that he had been stuck before and was going
through with it. Zehmer’s version was that he said to Lucy: “I am not trying to claim it
wasn’t a deal on account of the fact the price was too low. If I had wanted to sell
$50,000.00 would be a good price, in fact I think you would get stuck at $50,000.00.” A
disinterested witness testified that what Zehmer said to Lucy was that “he was going to
let him up off the deal, because he thought he was too tight, didn’t know what he was
doing, Lucy said something to the effect that ‘I have been stuck before and I will go
through with it.’ ”
If it be assumed, contrary to what we think the evidence shows, that Zehmer was
jesting about selling his farm to Lucy and that the transaction was intended by him to be a
joke, nevertheless the evidence shows that Lucy did not so understand it but considered it
to be a serious business transaction and the contract to be binding on the Zehmers as well
as on himself. The very next day he arranged with his brother to put up half the money
and take a half interest in the land. The day after that he employed an attorney to examine
the title. The next night, Tuesday, he was back at Zehmer's place and there Zehmer told
him for the first time, Lucy said, that he wasn't going to sell and he told Zehmer "You
know you sold that place fair and square." After receiving the report from his attorney
that the title was good he wrote to Zehmer that he was ready to close the deal.

55
I. Introduction: The Principle of Mutual Assent 7

Not only did Lucy actually believe, but the evidence shows he was warranted in
believing, that the contract represented a serious business transaction and a good faith
sale and purchase of the farm.
In the field of contracts, as generally elsewhere, “We must look to the outward
expression of a person as manifesting his intention rather than to his secret and
unexpressed intention. ‘The law imputes to a person an intention corresponding to the
reasonable meaning of his words and acts.’ ” First Nat. Exchange Bank of Roanoke v.
Roanoke Oil Co., 169 Va. 99, 114, 192 S.E. 764, 770.
At no time prior to the execution of the contract had Zehmer indicated to Lucy by
word or act that he was not in earnest about selling the farm. They had argued about it
and discussed its terms, as Zehmer admitted, for a long time. Lucy testified that if there
was any jesting it was about paying $50,000 that night. The contract and the evidence
show that he was not expected to pay the money that night. Zehmer said that after the
writing was signed he laid it down on the counter in front of Lucy. Lucy said Zehmer
handed it to him. In any event there had been what appeared to be a good faith offer and a
good faith acceptance, followed by the execution and apparent delivery of a written
contract. Both said that Lucy put the writing in his pocket and then offered Zehmer $5 to
seal the bargain. Not until then, even under the defendants' evidence, was anything said
or done to indicate that the matter was a joke. Both of the Zehmers testified that when
Zehmer asked his wife to sign he whispered that it was a joke so Lucy wouldn't hear and
that it was not intended that he should hear.
The mental assent of the parties is not requisite for the formation of a contract. If the
words or other acts of one of the parties have but one reasonable meaning, his
undisclosed intention is immaterial except when an unreasonable meaning which he
attaches to his manifestations is known to the other party. Restatement of the Law of
Contracts, Vol. I, § 71, p.74.

… The law, therefore, judges of an agreement between two persons exclusively from those
expressions of their intentions which are communicated between them. . . . Clark on Contracts,
4th ed., §3, p.4.

An agreement or mutual assent is of course essential to a valid contract but the law
imputes to a person an intention corresponding to the reasonable meaning of his words
and acts. If his words and acts, judged by a reasonable standard, manifest an intention to
agree, it is immaterial what may be the real but unexpressed state of his mind. C.J.S.
Contracts, §32, p. 361; 12 Am.Jur., Contracts, §19, p. 515.
So a person cannot set up that he was merely jesting when his conduct and words
would warrant a reasonable person in believing that he intended a real agreement. 17
C.J.S., Contracts, §47, p. 390; Clark on Contracts, 4th ed., §27, at p.54.
Whether the writing signed by the defendants and now sought to be

56
8 Chapter 1. Intent To Contract: Offer and Acceptance

enforced by the complainant was the result of a serious offer by Lucy and a serious
acceptance by the defendants, or was a serious offer by Lucy and an acceptance in secret
jest by the defendants, in either event it constituted a binding contract of sale between the
parties. . . .
The complainants are entitled to have specific performance of the contract sued on.
The decree appealed from is therefore reversed and the cause is remanded for the entry of
a proper decree requiring the defendants to perform the contract in accordance with the
prayer of the bill.
Reversed and remanded.

NOTE

The court mentioned the concession at oral argument that Zehmer was not too drunk
to contract. As we shall see in Chapter 6, sometimes the impaired mental state of one of
the parties prevents that person from having the legal capacity to enter into a contract.

Problem 2

John Falstaff bought a new car for $15,000, but the first day he drove it to work it
broke down on the highway and stranded him. John finally managed to get to his
neighborhood bar where he loudly trumpeted his disgust with the car. When the busboy,
Francis Feeble, who was known by all to have a very low I.Q., said, “I love your new
car,” John raised his stein of beer and toasted Francis, replying, “It’s yours for $1,500 –
just go get it.” Francis said, “Thanks, Mr. Falstaff, I’ll do it now and pay you this
evening.” Everyone in the tavern laughed as Francis rushed off to rescue the car.
Later that day, after Francis had had the car towed to his home, Francis showed up at
the bar where John was still consoling himself and put $1,500 on the counter in front of
John. When John asked, “What’s this?” Francis explained that it was the payment for the
car. John refused to take the money, saying that of course he had not been serious earlier
in the day. Is Francis entitled to the car in your opinion?

Problem 3

U. N. Owen was a fanatical bridge player. Once a year he invited a number of people
to his remote home on Indian Island for a weekend of nonstop bridge. He sent the
invitations two months in advance and followed them with phone calls to make sure
everybody was really going to

57
I Introduction: The Principle of Mutual Assent 9

come. One of the guests, a retired jurist named Justice Wargrave, assured Mr. Owen that
he would make the journey but backed out at the last minute and went to a pinochle
tournament instead. When guests arrived at the island, Owen was dismayed to find that
he did not have enough people for the complicated bridge tournament he had planned, so
he cancelled the entire weekend and sent everyone home. He then sent Wargrave a bill
for $2,750, the expenses he had undergone in setting up the weekend. Must Justice
Wargrave pay?

NOTE ON SOCIAL CONTRACTS

The existence of a familial relationship may affect the ability to create an


enforceable contract. For example, courts have been traditionally reluctant to become
involved in disputes within a harmonious, unbroken marital relationship. A husband and
wife are not precluded from enforcing such agreements as they make in a commercial
transaction, but contracts regulating matters essential to the marital relationship are
viewed with great suspicion. See McDowell, Contracts in the Family, 45 B.U. L. Rev. 43
(1965). For example, a spouse employed in a business must be paid wages, See, e.g.,
Peterson v. Massey, 155 Neb. 829, 53 N.W.2d 912 (1952). Similarly, a spouse can bring
an action to recover on a loan made to the other spouse for an independent business
enterprise. See, e.g., Kraemer v. Kraemer, 76 Nev. 265, 352 P.2d 253 (1960). On the
other hand, courts will not usually concern themselves with enforcing domestic
agreements controlling relations within the marriage. Typically, for example, agreements
for bearing children are unenforceable. Similarly, courts generally will not involve
themselves with living arrangements within the home, such as privileges to be afforded
children from prior marriages. As a practical matter, courts are not likely to be confronted
with these kinds of disputes in an ongoing marriage since breaches serious enough to
result in litigation also terminate the marriage. See Weitzman, Legal Regulation of
Marriage: Tradition and Change, 62 Cal. L. Rev. 1169, 1270 (1974).
The law does recognize some spousal agreements that affect the relationship:
agreements that anticipate the eventual dissolution of the marital relationship, whether by
death or divorce, and that seek to control the disposition of the parties’ respective
interests upon death or dissolution. The courts limit their enforcement to matters of
dissolution, custody of children and apportionment of property. See Loeb, Prenuptial and
Postnuptial Agreements, 54 Wis. B. Bull. 3 (1981).
Recently some courts have become involved in disputes arising from agreements
between unmarried cohabitants. The enforceability of an alleged contract in this context
is discussed in Chapter 6.

58
6. TRESPASS TO LAND
Dougherty v. Stepp
Supreme Court of North Carolina, 1835
18 N.C. 371

This was an action of trespass quare clausum fregit, tried at Buncombe on


the last Circuit, before his Honor Judge Martin. The only proof introduced by the
plaintiff to establish an act of trespass, was, that the defendant had entered on the
unenclosed land of the plaintiff, with a surveyor and chain carriers, and actually
surveyed a part of it, claiming it as his own, but without marking trees or cutting
bushes. This, his Honor held not to be a trespass, and the jury under his
instructions, found a verdict for the defendant, and the plaintiff appealed. * * *

RUFFIN, CHIEF JUSTICE. In the opinion of the Court, there is error in the
instructions given to the jury. The amount of damages may depend on the acts
done on the land, and the extent of injury to it therefrom. But it is an elementary
principle, that every unauthorized, and therefore unlawful entry, into the close of
another, is a trespass. From every such entry against the will of the possessor, the
law infers some damage; if nothing more, the treading down the grass or herbage,
or as here, the shrubbery.

Judgment reversed, and new trial ordered.

NOTES AND QUESTIONS


1. We are here concerned only with intentional trespass to land. There may be negligent
trespass, but it is governed by the ordinary rules applicable to negligence actions. One of
these is that when the entry upon the land is merely negligent, proof of some actual
damage is essential to the cause of action. Thus, the word trespass may be used to
describe the kind of interest that defendant has invaded. Traditionally, the interest has
been described as the right to exclusive possession of land. Restatement (Second) of
Torts § 165.

2. Is the trespass intentional when the defendant enters the land in the honest and
reasonable belief that it is his own? See Glade v. Dietert, 156 Tex. 382, 387, 295 S.W.2d
642, 645 (1956) (dictum). Cf. Ranson v Kitner, page 23 and Serota v. M. & M. Utilities,
Inc., 55 Misc.2d 286, 285 N.Y.S.2d 121 (1967), page 24, note 2.

3. In addition to a cause of action for trespass, the plaintiff might have a cause of action
for ejectment. Ejectment would lie if the defendant was in possession of the land, rather
than just having been on it temporarily. A judgment in her favor would entitle plaintiff to
the sheriff’s help in removing the defendant from her land. If the plaintiff is in
possession of the land, but the defendant is claiming to own it, a quiet title action would
lie.

59
4. In a trespass action, plaintiff will be awarded nominal damages if there are no actual
(compensatory) damages. Why nominal damages when the trespass does no harm? Why
bother? The explanation sometimes given is that the action of trespass is frequently
either a suit intended to try title or is directed at the vindication of the legal right, without
which the defendant’s conduct, if repeated, might ripen into a prescriptive right. Hence,
there is no room for the application of the maxim, de minimus non curat lex. See 1 T.
Street, Foundations of Legal Liability, 25 (1906). On the other hand, if the law declared
that there was no cause of action for invasions that caused only nominal damages, it
would not be possible to acquire a “prescriptive” right. At common law, one reason
underlying the rule was to keep the peace. Is the rule justified today? What about
punitive damages? Should they be available if only nominal damages were awarded?
See Jacques v. Steenberg Homes, Inc., 209 Wisc.2d 605, 563 N.W.2d 154 (1997)
(upholding punitive damages award even though no compensatory damages were proved
where defendant had bulldozed a path in the snow across plaintiff’s field to deliver a
mobile home despite plaintiff’s repeated refusals to permit the defendant to cross his land
to make the delivery).

5. When a trespassory invasion is found, the fact that defendant’s conduct was socially
useful or even beneficial to plaintiff does not affect liability. See Harmony Ditch Co. v.
Sweeney, 31 Wyo. 1, 222 P. 577 (1924); Longenecker v. Zimmerman, 175 Kan. 719, 267
P.2d 543 (1954).

6. Of course, if the plaintiff does suffer compensatory damages, those too are
recoverable in the trespass action. What about emotional distress damages? Defendant
landlord, seeking to persuade tenants to vacate a townhouse, entered the premises without
their consent, changed the locks on the garage, blocked the driveway, and repeatedly
threatened and harassed the tenants. Johnson v. Marcel, 251 Va. 58, 465 S.E.2d 815
(1996) (where alleged trespass is deliberate and accompanied by aggravating
circumstances, emotional distress damages may be recovered even in the absence of
physical injury).

60
Sample Brief: Dougherty v. Stepp

Dougherty v. Stepp, NC Supreme Court, 1835

Facts: D apparently thought that the land was his, so he went onto P’s land with a
surveyor, and they surveyed the land. However, they didn’t do any damage because they
didn’t mark any trees or cut any bushes.

Procedure: Trial judge directed a verdict for D, and P appeals.

Issue: Whether a plaintiff must show actual injury to the land to win a case for trespass?

P might argue: Going onto my land without my permission violates my property rights,
so D should have to pay something for that.

D might argue: No harm, no foul. If I didn’t hurt your land, why should you be able to
sue?

Law: Elements of trespass:


without permission (unlawful)
entry
onto land of another
actual damage isn’t required
Reasoning: (Apparently, the trial judge thought that if there wasn’t any damage, there
couldn’t be trespass.) The law infers damage, even if we have to pretend that the
“damage” was walking on the grass. So if D does do actual damage, that just increases
the amount of money that P gets.

Holding: The trial judge was wrong. Reversed and remanded for a new trial.

61
Sample Brief: Dougherty v Stepp, NC Supreme Court, 1835
Issue: Whether a plaintiff must show an actual injury to the land to win a case for trespass.

Reasoning Law Facts

(These four elements are clearly met.) Elements of trespass:


• without permission (unlawful) No facts to show P gave permission

• entry D went onto

• onto land of another Land belonging to P

• intent (we got this from notecase) the


only intent required is the intent to go
onto the land – it doesn’t matter whether

62
you know it’s someone else’s land

(Apparently, the trial judge thought actual harm isn’t required to win the they surveyed the land. However, they
that if there wasn’t any damage, case didn’t do any damage because that
there couldn’t be any trespass.) The didn’t mark any trees or cut any bushes.
law infers damage, even if we have
to pretend that the “damage” was
walking on the grass. So even if D P’s argument: Going on to my land
does do actual damage, that just without my permission violates my
increases the amount of money that P property rights, so D should have to pay
gets. something for that.
Procedure: Trial judge directed a verdict D’s argument: No harm, no foul. If I
for D, and P appeals. didn’t hurt your land, why should you be
able to sue?
Sample In-Class Essay
Trespass to Land
“The Hills Have Eyes”

Leah and Jim own 240 acres in New Hampshire. They live in Maryland, but they visit

their New Hampshire property as often as they can. They plan to build a house there eventually.

Shortly after they bought the land, they posted “No Trespassing” signs around the

boundaries. However, because the piece of land is so big, sometimes the signs are rather far

apart. There are no fences around the land.

Maria was walking through the beautiful New Hampshire mountains. She crossed a little

brook and walked into a clearing. There she saw Leah and Jim having a picnic. Maria had

walked right through the boundary without seeing any “No Trespassing” signs.

Leah and Jim want to sue Maria for trespass to land. Can they win? Discuss and decide.

63
Sample Answer to In-Class Essay
“The Hills Have Eyes”
The issue is whether Leah (L) and Jim (J) can win a trespass to land case against Maria
(M).

The first element is intent. Intent means either entering the land on purpose or knowing
with substantial certainty that one will enter the boundaries of the land. It doesn’t matter
whether one knows that one is going onto the land of another. Here, M was walking through the
mountains and crossed L and J’s unfenced boundary without seeing any “No Trespassing” signs.
Because there was no fence, M wouldn’t know that she was crossing a boundary from one parcel
to another, and because she saw no signs, she wouldn’t know that she was entering land that she
was forbidden to enter. However, it won’t make any difference that M didn’t know that she was
doing wrong: she was purposefully propelling herself onto L and J’s land by walking there. She
wasn’t carried there against her will. So the intent element is met.

The next element is unauthorized (no permission) or unlawful. Here, L and J had posted
their land with “No Trespassing” signs. The whole point of putting up those signs was to tell
strangers like M to stay out because they didn’t have permission to enter the land. So this
element is met.

The next element is entry. M walked right through the boundary of L and J’s land, which
broke the close. So this element is met.

The next element is “onto land of another.” M walked onto part of L and J’s 240 acres of
land, so M was actually on their land. That is always sufficient for entry, so this element is met.

Finally, to win the case, it isn’t necessary for L and J to show that M actually harmed the
land. Instead, the entry itself is the injury. So even though M was just walking and hurt nothing,
L and J can still win. However, they will get only nominal damages because there is no injury
for which to allow them compensatory damages.

Because all the elements are met and M has no counter-arguments, L and J will win.

64
Sample Template Answer to In-Class Essay
“The Hills Have Eyes”

Students: Note that the text in this template is exactly the same as the text in the
paragraph-by-paragraph answer for this essay question. The point is that the template
helps you to organize the pieces of the answer. After you practice this pattern enough, you
will have a template in your head that will help you organize your answer. It will become
automatic.

Issue: The issue is whether Leah (L) and Jim (J) can win a trespass to land case against Maria
(M).

E – element: The first element is intent.

R – rule: Intent means either entering the land on purpose or knowing with substantial certainty
that one will enter the boundaries of the land. It doesn’t matter whether one knows that one is
going onto the land of another.

F – facts: Here, M was walking through the mountains and crossed L and J’s unfenced
boundary without seeing any “No Trespassing” signs.

R – reasoning: Because there was no fence, M wouldn’t know that she was crossing a boundary
from one parcel to another, and because she saw no signs, she wouldn’t know that she was
entering land that she was forbidden to enter. However, it won’t make any difference that M
didn’t know that she was doing wrong: she was purposefully propelling herself onto L and J’s
land by walking there. She wasn’t carried there against her will.

C – conclusion: So the intent element is met.

E – element: The next element is unauthorized (no permission) or unlawful.

R – rule: (When we did this essay in class, we didn’t have a rule for this yet, other than to say
that unauthorized means “no permission.”)

F – facts: Here, L and J had posted their land with “No Trespassing” signs.

R – reasoning: The whole point of putting up those signs was to tell strangers like M to stay out
because they didn’t have permission to enter the land.

C – conclusion: So this element is met.

E – element: The next element is entry.

R – rule: (When we did this essay in class, we didn’t have a rule for this yet.)

65
M walked right through the boundary of L and J’s land,

R – reasoning: which broke the close.

C – conclusion: So this element is met.

E – element: The next element is “onto land of another.”

R – rule: (When we did this essay in class, we didn’t have a rule for this yet.)

F – facts: M walked onto part of L and J’s 240 acres of land,

R – reasoning: so M was actually on their land. That is always sufficient for entry,

C – conclusion: so this element is met.

E – element: (At this point, we’re out of elements – that is, there are no more pieces that do have
to be proved. The part about not having to prove damages is a rule that further defines trespass.)

R – rule: Finally, to win the case, it isn’t necessary for L and J to show that M actually harmed
the land. Instead, the entry itself is the injury.

F – facts: So even though M was just walking

R – reasoning: and hurt nothing, L and J can still win. However, they will get only nominal
damages because there is no injury for which to allow them compensatory damages.

C – conclusion: (Note that two conclusions were slipped into the reasoning, above. One
conclusion was that L and J can still win; the other was that there was no injury to give them
compensatory damages.)

C – final conclusion: Because all the elements are met and M has no counter-arguments, L and
J will win.

66
Week Three:

Multiple Choice

67
68
Week 3 Pre-Reading Questions
(1) What are “divergent” and “convergent” thinking? Which is appropriate for multiple-
choice questions? Why?

(2) What are the steps for solving multiple-choice questions?

(3) How are these steps similar to, or different from, what you have done before when
solving multiple-choice questions?

(4) What are “transition words”? How many types are there? What are they?

(5) Why should you pay attention to and distinguish between the different types?

(6) What are the steps for analyzing each type?

(7) In what important ways are the memory demands in law school different than in other
educational venues?

(8) What is the “Memory Chain”? How does it work?

(9) How do you create retrievable long-term memory? What steps can you use?

(10) What is “chunking” information? How and why would you do it?

69
70
Divergent Versus Convergent Thinking:
Making the Transition from Essay to Multiple-
Choice Questions

“Divergent” thinking means opening up your mind to think of all possible pieces of law that
might be reflected in the essay facts and the call of the question. Essay questions require you to
think divergently. That is, you read the facts and ask, “What law is represented by this fact?”
Your goal is to think of all the legal concepts that pertain to those facts. When writing your
answer, you discuss as many legal concepts as those facts (and the call of the question) allow.

Divergent thinking is the opposite of what you must do on multiple-choice questions. Instead,
you must think convergently, which means limiting your mind to specific pieces of law.
Multiple-choice questions require you to figure out exactly which pieces of law are being tested.
The only pieces of law worth thinking about are the pieces that relate to what the question is
asking you. So your goal is to narrow your thinking. You do this by using an effective reading
strategy.

The strategy includes not reading the facts first. Reading the facts first would encourage you to
think about all the legal concepts that might have something to do with the fact pattern. That is
exactly what you do not want to do. Instead, first find out what law the question pertains to.
That will make it easier to figure out which facts are important to the test writer.

Reading the Multiple-Choice Question

• Multiple-choice questions are meant to test concepts. Multiple-choice questions require


analytical reasoning skills, just as essays do.

• Your job is to figure out what you are being asked.

71
1. Read the call of the question.
• Look for hints about what law is being tested.
• Look for direction words.
• Look for the stars (who or what you are to focus on in the facts).
• Underline the direction words, the stars, and the law.

2. Read the options.


• Look for the law.
• Underline the law.

3. Form a new question.


• The basic question is, “Now that I know what law is being tested, which
facts am I looking for in the fact pattern that will help me analyze the
question?”

4. Using your question, read the fact pattern.


• Ask yourself, “What law or attribute of the law is this fact pattern
describing?”
• Ask yourself, “How are these facts formatted?”

5. Go back to the options (also called “picks”).


• Eliminate the wrong options.
• Choose the option that answers your question.

6. Make a reasoning statement to check your answer.


• If you can explain to yourself why you are picking one option and
negating the others, then you are no longer answering things
impressionistically, but putting affirmative reasons on your decision-
making.

72
Example:

Chuck had obtained a permit from the national forest service to cut some
firewood in the national forest, and he had driven his pickup to the designated
area. He had begun to cut down a marked tree with his axe when Chloe, a
member of the “Save the Vegetation” movement approached him and began
berating him for cutting the tree. (“Save the Vegetation” members believed that
plants were a higher form of life than animals and advocated a total ban on the
killing of plants.) Chuck told her that he had a permit to cut and to leave him
alone. Chloe persisted, however, shouting that Chuck was a “moronic, murdering
plant killer!” Intending to frighten Chloe away, Chuck said, “Get away or I’ll
start cutting on people!” He swung his axe as if to strike her. The manufacturer
of the axe had neglected to insert a metal pin that secured the axe handle to the
blade, and Chuck’s previous chopping had so loosened the head that it flew off
the axe handle and struck Chloe in the upper chest, crushing her clavicle.

If Chloe brings an action for battery against Chuck, will she recover?

(A) No, because a reasonable person would have been angered by Chloe’s remarks.
(B) No, because the manufacturer of the defective axe should be liable since the
defective axe was the cause in fact of Chloe’s injuries.
(C) Yes, because Chuck intended to frighten her.
(D) Yes, because she intended to provoke Chuck.

1. Read the call of the question.


• The law being tested is battery. Underline “battery,” and mentally run
through what you know about battery.
• The direction word is whether Chloe can “recover,” so underline that. To
“recover,” Chloe must prove all the elements, so the question is asking
you to decide whether the facts prove those elements.

73
• The stars are Chloe (plaintiff) and Chuck (defendant). Focus on them and
their respective roles.

2. Read the options.


• The law in (A) is “reasonable person.” Underline that. Is that related to
battery? Possibly, because the contact must be offensive to a reasonable
person. But that is not how “reasonable person” is being used here
because, here, “reasonable person” is referring to a defense that can be
used to justify battering someone because someone is “reasonably” being
angered. Cross it out!
• The law in (B) is “axe is cause in fact of injuries.” Underline that. Is that
related to battery? Possibly, because if the only cause of the injuries is a
defective axe, then Chuck may lack intent to cause the contact. Leave this
as an option.
• The law in (C) is “intended to frighten.” Underline that. Is that related to
battery? Yes, because although it is not the correct intent for battery, it
could be the correct intent for an assault claim (so long as the “intent to
frighten” is to frighten someone into thinking they are going to be
contacted). Battery and assault are two of the intentional torts that allow
tort-to-tort transferred intent. Leave this as an option.
• The law in (D) is “Chloe intended to provoke.” Underline that. Is that
related to Chuck’s alleged battery on Chloe? Not likely. Usually, we only
care about the defendant’s intent, not about the plaintiff’s intent. There is
a very small possibility that it could give Chuck a defense, but it’s unlikely
that this is correct. Don’t choose (D) unless the facts give you specifics
for Chuck’s defense.

3. Form a new question.


• You only have two options left, (B) and (C). (B) will negate the intent
element if it is correct. (C) will use Chuck’s intent to assault Chloe to
transfer into a battery claim. Because the only two good options both deal
74
with intent, the only facts that are going to be relevant to the question are
those that explain what Chuck knew about and what Chuck meant to do.
So the new question might be something like this: “What did Chuck mean
to do to Chloe, if anything?”

4. Using your question, read the fact pattern.


• What are the most important facts in the fact pattern, given the question
you’ve focused on? Chuck intended to frighten Chloe. He swung the axe
at her as if to strike her. The axe head flew off the handle, struck Chloe in
the upper chest, and crushed her clavicle.
• So Chuck did have the intent to make Chloe think she was going to be hit
by the axe. And when he swung at her to make her think that, it ended up
causing a harmful contact.
• We also have a clear contact and clear harm.

5. Go back to the options.


• Eliminate the bad options.
• (B) is bad because Chuck did have the intent for assault. And the question
isn’t asking us about whether there may be a products liability claim. The
manufacturer of the axe may be liable also, but this question focuses only
on Chuck’s alleged battery.
• (D) is definitely out now because her nasty words would not give Chuck a
right to retaliate with harmful contact. There is no defense here.
6. Make a reasoning statement to check your answer.
• Because Chuck had the requisite intent for assault and ended up
committing a battery through his actions, the intent for the assault can
transfer over to the battery claim. And the other elements of a battery are
present to round out the battery claim. The answer is (C).

75
76
Multiple-Choice Questions
Understanding Transition Words

What are Transition Words?


“Transition words” are logical connectors. Not all multiple-choice questions contain
transition words. However, if a question does contain them, it is important to understand what
the transition words mean. Otherwise, you may not understand what the question is asking.
Using them is a part of the test-writer’s strategy; therefore, understanding them must be part of
your, the test-taker’s, strategy.

Where are the Transition Words?


Transition words are usually found in one of four options or “picks” at the end of the
question. Occasionally, they will be found in the call of the question or even the fact pattern.

The four most common categories of transition words are:

1. Because (which can also be phrased as “since,” “for the reason that,” “as,” or
some other word or phrase that means “because”)
2. If
3. Only if
4. Unless

Because each of these means something different and creates a different relationship between the
answer and the reasoning for the answer, each must be analyzed differently.

Example:
Here is an example of a multiple-choice question with transition words in each pick.
There is also a transition word (“if”) in the call of the question. For each of the four transition
words, you should learn the pattern of the pick and the steps for analyzing it.

Dante just received a letter from Thomas M. Cooley Law School informing him that he
had been accepted to begin law school in the Michaelmas term. He immediately informed
his friends and scheduled a big party. During the party, Dante decided to “get rowdy” and
began to shoot his Smith and Wesson .357 Magnum into the air. Down the block, Penny
was lounging by her swimming pool and heard the gunshots. As she headed into the house
to use the telephone to call the police, she noticed that her dog was lying on the ground
with blood all around. Penny was very upset because her prize poodle had been struck by
the bullet shot by Dante.

If Penny sues Dante, will she win?


A. No, because shooting the gun in the air was not extreme and outrageous.
B. Yes, if the bullet hit the dog in Penny’s yard.
C. Yes, but only if the dog died as a result of the gunshot.
D. No, unless Dante meant to cause Penny apprehension about leaving the dog
alone by the poolside.
77
The question is, assuming that Penny sues Dante, will Penny win on some tort? Now look at
each pick.

**********
A. No, because shooting the gun in the air was not extreme and outrageous.

Pattern:
answer because reason

Here, the answer is that Penny will not win.


The reason is that “shooting the gun in the air was not extreme and outrageous.”

Steps for analyzing a “because” pick:

1. Look at the reason and ask whether it is true. Sometimes the reason contains a
rule: then you ask whether the statement of the rule is accurate. Here, the reason
contains law: “extreme and outrageous,” which is an element of IIED. Sometimes (as
here) the reason contains facts: then ask whether the facts in the reason are true or
whether they contradict what the fact pattern says; you can also evaluate whether the
application of the law to those facts is reasonable. If the reason is not true, then the
entire pick is false, so it is not the right answer.

Here the reason sounds true: shooting a gun in the air might not be extreme and
outrageous, and we’ll have to read the facts before we know whether it contradicts the
fact pattern. So you need to go to step 2.

2. Ask whether the reason justifies the answer. Would doing something that is not
extreme or outrageous make the plaintiff lose? If the tort is IIED, it would. So this
pick is not obviously erroneous. Go on to the next pick.

**********

B. Yes, if the bullet hit the dog in Penny’s yard.

Pattern:
answer if reason

Here, the answer is that Penny will win.


The reason is that “the bullet hit the dog in Penny’s yard.”

Steps for analyzing an “if” pick:

1. Look at the reason and assume it is true. Do not question it, and do not waste time
asking whether Penny can prove that it is true. Whatever is in the reason after an “if”
78
is true for this pick only. This is a way for the professor to add some new facts to the
fact pattern and to see whether you know whether the new facts are, or are not, legally
significant. Here, the pick talks about the bullet’s entering Penny’s yard. What tort
does that make you think of? Trespass to land.

2. Ask whether the reason leads to or justifies the answer. Does the fact that a bullet
entered Penny’s yard and struck her dog justify her winning a trespass-to-land claim?
Well, it doesn’t hurt. It makes it possible for Penny to win by proving an entry into
the close that was unauthorized and caused harm.

**********

C. Yes, but only if the dog died as a result of the gunshot.

Pattern:

answer only if reason

Here, the answer is that Penny will win.


The reason is that “the dog died as a result of the gunshot.”

Steps for analyzing an “only if” pick:

1. Look at the reason and assume that it is true. Just as with the “if” pick, this reason
is true for this pick only. So you add the new facts in the reason to the fact pattern and
ask whether those new facts are legally significant. Here, once we know that the dog
died, trespass to chattels or conversion comes to mind.

2. Ask whether the reason is the ONLY thing that will lead to or justify the answer.
In other words, is the reason necessary for the answer to occur? Here, the question
will be whether it is necessary for the dog to die for Penny to win either a trespass-to-
chattels or conversion claim. Can Penny win only if the dog died? No. Neither
trespass to chattels nor conversion requires that the dog die. Lesser harm is sufficient.
Penny can win if the dog is maimed, but lives. So this pick is erroneous and cannot be
the right answer.

**********

D. No, unless Dante meant to cause Penny apprehension about leaving the dog alone by
the poolside.

Pattern:
answer unless reason

79
Here, the answer is that Penny will not win.
The reason is that “Dante meant to cause Penny apprehension about leaving the dog alone by the
poolside.”

Steps for analyzing an “unless” pick:


“Unless” is the most complicated of the four common categories of transition words.

1. Look at the reason and assume it is true. Just as with “if” and “only if,” the reason
after an “unless” is true for this pick only. So you add the new facts in the reason to
the fact pattern, and ask whether those new facts are legally significant.
“Apprehension” should make you think of assault.

2. Ask whether the reason is the ONLY thing that will change the answer. “Unless”
means “exception.” The ways an exception works is that the answer is true unless the
exception applies, in which case the answer is not true. So this pick asks whether,
without the facts in the reason, Penny will lose, and whether adding the facts in the
reason is the only way Penny can win. In other words, this pick asks whether, just
given the facts in the fact pattern, Penny won’t win, and the only way for her to win is
that Dante wanted to cause her apprehension about leaving the dog alone by the
poolside.

Is that the only way for Penny to win an assault claim?

Of course not. First, “apprehension” refers to the plaintiff’s seeing someone making a
threatening gesture toward her. It has nothing to do with being nervous about leaving
a dog alone. Furthermore, there are other picks referring to other torts that Penny
could win on, so this can’t be the only way that Penny can win.

In the alternative, you can turn this pick into an “only if” format and follow the “only if”
analysis steps. This is much easier for most students to analyze.

To do this, you must make the following changes to the option:

1. Change the answer that is to the left of the transition word to its opposite. (Examples:
“lose” becomes “win,” “plaintiff” becomes “defendant,” “admissible” becomes
“inadmissible,” “constitutional” becomes “unconstitutional,” “yes” becomes “no,” etc.)

2. Change the transition word from “unless” to “only if.”

answer (win) only if

Now you can use the “only if” analysis steps to analyze the option, and it will come out the
same as if you used the word “unless” and followed the steps to analyze “unless.”

80
FINAL OUTCOME:

The correct answer here is B. When Dante shot the bullet up into the air, he knew with
substantial certainty that it would have to land in someone’s yard. And the bullet did enter
Penny’s yard without permission.

A is wrong because Penny can win on an IIED claim because shooting a gun in the air in the
middle of a party with a lot of people around in a residential neighborhood for no reason besides
to get rowdy could be considered extreme and outrageous.

C is wrong: we already negated C because it is clearly erroneous to say that the only way Penny
can win on trespass to chattels or conversion is if the dog dies.

D is wrong: we again already negated D because it is clearly erroneous to say that the only way
Penny can win on assault is if Dante meant to make her nervous about leaving the dog alone.

81
This is a chart that summarizes the analysis for each of the four categories:

Category Step 1 Step 2

Because Ask whether the reason is true. Ask whether the reason leads
to or justifies the answer.

If yes, go on to step 2 If yes, the option might be the


If no, stop here because the right answer.
option is wrong. If no, the option is wrong.
If Assume that the reason is true. Ask whether the reason leads
to or justifies the answer.

If yes, the option might be the


right answer.
If no, the option is wrong.
Only if Assume that the reason is true. Ask whether the reason is the
only way or is necessary for
the answer to be true.

If yes, the option might be the


right answer.
If no, the option is wrong.
Unless Assume that the reason is true. Ask whether the reason is the
only way or is necessary for
the answer to change to the
opposite.

If yes, the option might be the


right answer.
If no, the option is wrong.
Shortcut for “Unless” Turn the option into an only Ask whether the reason is the
if. only way or is necessary for
the answer to be true.
(1) Change the answer to its
opposite. If yes, the option might be the
right answer.
(2) Add only if. If no, the option is wrong.

(3) Add the reason.

Then assume that the reason is


true.

82
Multiple Choice: Focused Practice
Step 2 in reading a multiple-choice question is to find the law in each pick. Step 3 is to form a new
question. This assignment is intended to give you focused practice on those two steps.

First, find some multiple-choice questions. You can take them from any source. Second, write what
law is suggested by each pick. Third, looking at the call of the question and the law in the picks, write
out the new question – that is, what are you looking for in the facts?

Question number:

A.___________________________
B.___________________________
C.___________________________
D.___________________________

New question:________________________________________________________________

Question number:
A.___________________________
B.___________________________
C.___________________________
D.___________________________

New question:________________________________________________________________

Question number:
A.___________________________
B.___________________________
C.___________________________
D.___________________________

New question:________________________________________________________________

83
Question number:
A.___________________________
B.___________________________
C.___________________________
D.___________________________

New question:________________________________________________________________

Question number:
A.___________________________
B.___________________________
C.___________________________
D.___________________________

New question:________________________________________________________________

Question number:
A.___________________________
B.___________________________
C.___________________________
D.___________________________

New question:________________________________________________________________

84
Introduction to Law
Multiple-Choice Questions

1. Pete and Dan had a longstanding feud. During an argument, Pete called Dan several
very offensive names. Afterward, Pete got into his car and drove away. Dan,
becoming angrier by the minute, got into his own car and followed Pete. Dan caught
up with Pete and started to pass Pete’s car. As he did, he intentionally swerved at
Pete’s car to frighten Pete. Pete didn’t realize that Dan was the one swerving at him,
but he thought that he was about to be forced off the road. As he veered away from
Dan’s car, Pete’s car went into a ditch and Pete was injured.

If Pete sues Dan, Pete should win on the following cause(s) of action:

A. Battery only.
B. Assault only.
C. Both battery and assault.
D. Neither battery nor assault.

2. Pat and Dave were arguing. Dave slapped Pat in the face. Angry, Pat pointed an
unloaded pistol at Dave. Dave immediately drew a knife and stabbed Pat with it,
injuring Pat severely. Pat sued Dave for battery. The only defense that Dave raised
was self-defense.

Pat’s best argument to show that Dave is liable would be:

A. Dave’s use of the knife constituted deadly force.


B. Dave knew or should have known that he could safely and easily retreat
without sustaining harm.
C. Dave was the initial aggressor.
D. Pat knew that the pistol was unloaded.

85
3. Joan had a fifteen-year-old daughter, Betty. Betty went on a camping trip to
Michigan’s Upper Peninsula. Galen knew that Betty was going camping, so he
decided to make some money for himself. After Betty left, Galen called Joan and,
while disguising his voice, told her that he had kidnapped Betty. He also said that he
had already cut Betty’s left ear off and that he would kill Betty if Joan didn’t deliver a
cash ransom to a particular place the next day. Joan didn’t know exactly where
Betty’s campsite was, and Betty had forgotten to take her cellular phone with her, so
Joan had no way to find out whether Galen was telling the truth. So Joan left the
money in the specified location. Joan was frantic for the next three days, until Betty
came home from the camping trip. When Joan realized the whole thing had been a
hoax, she suffered a minor stroke.

If Joan sues Galen for assault, the court should find for:

A. Joan, because Galen was aware that his conduct would frighten Joan.
B. Joan, because the court will transfer Galen’s intent.
C. Galen, because Joan did not see Galen injure Betty.
D. Galen, because Joan had no reason to expect to be touched by Galen.

4. Darcy, who was armed with a pistol, was hunting on his own land when he saw Pat
carrying a shotgun and trying to enter Darcy’s land by climbing over a fence. Darcy
waited until Pat got over the fence and onto Darcy’s land. Then he told Pat that he
was trespassing. Darcy ordered Pat to leave. Pat refused. Darcy placed his hand on
Pat’s chest and gently pushed Pat backward, again ordering Pat to leave. Pat shoved
Darcy away from him and pointed his shotgun at Darcy, still refusing to leave. Darcy
then pulled the pistol from his belt and shot Pat in the arm, which caused Pat to drop
the shotgun.

If Pat sues Darcy for battery, the court should find for:

A. Darcy, if he fired at Pat to defend his land against a trespass.

86
B. Darcy, if he fired at Pat to defend himself against Pat’s threat with the
shotgun.
C. Pat, because Darcy struck the first blow.
D. Pat, because Darcy did not use force against him until his entry onto the land
was complete.

Use the following facts for questions 5-7

Missile Company (MC) was building a space shuttle for the government. MC had built a
huge, solid-fuel rocket engine for its prototype shuttle. MC tested the engine in a remote area of
the desert by mounting it on a concrete test stand, firing it up, and directing the blast from the
engine downward into the ground. During the test, huge clouds of flame and smoke filled the air,
and particles of debris thrown from the rocket fell on an adjoining farm owned by Homesteader.
The engine blast also caused heavy vibrations in the ground beneath the test site, which caused
subsurface earth structures surrounding the site to collapse, which in turn caused Homesteader’s
water well to collapse.

5. If Homesteader sues MC for trespass, which of the following facts, if proved


would be most helpful to MC in avoiding liability?

A. When Homesteader bought the farm, he knew that MC used the adjoining
property for testing rocket engines.
B. No employee of MC had ever set foot on Homesteader’s land.
C. MC had no reason to anticipate that the tests would cause any of the results
that occurred.
D. The rocket-testing program is essential to national security; therefore, MC’s
conduct was completely privileged as a public necessity.

6. If Homesteader sues MC on the theory that MC had engaged in an ultra-


hazardous activity, which of the following arguments, if proved, would be the
least helpful to MC in avoiding liability?

87
A. Homesteader’s farm is so far from the test site that a reasonable person would
not have foreseen the risk of harm that occurred.
B. MC had exercised due care and caution in constructing the test site and in
conducting the tests.
C. The tests conducted by MC did not pose a serious risk of harm to either
Homesteader or his property.
D. Other companies frequently carried out similar rocket engine tests in the same
locale.

7. If Homesteader maintains his action against MC on the theory that MC was


negligent, which of the following would be most helpful to MC in avoiding
liability?

A. The subsurface earth structures that collapsed as a result of the tests were
unstable before the tests took place.
B. Homesteader’s farm is so far from the test site that no risk to Homesteader
was foreseeable.
C. MC exercised due care in selecting the personnel who chose the test site and
conducted the tests.
D. MC built its test site and conducted the tests in conformity with safety
procedures and standards used by all other companies engaged in similar tests.

8. Paula loaned her car to her girlfriend Daphne specifically so that Daphne could pick
up a pizza that Paula had ordered for her and Daphne’s dinner. Daphne drove to the
pizzeria and parked the car. But instead of going straight in, Daphne went to a
bookstore next door. In the bookstore, Daphne browsed for a while and then bought a
book. She then went to the pizzeria to pick up the pizza, which had been ready for
fifteen minutes. Just as Daphne was walking out of the pizzeria with the pizza,
another car, driven by Tammy, smashed into Paula’s parked car. Fixing the damage to

88
Paula’s car will cost $2,000, and Paula didn’t have insurance. If Paula sues Daphne
for negligence, who will win?

A. Paula, because Daphne exceeded her authority when she went into the
bookstore.
B. Paula, because but for Daphne’s delay in getting the pizza, Paula’s car would
not have been damaged.
C. Daphne, because she did not create a foreseeable risk of damage to Paula’s
car.
D. Daphne, because the family car doctrine imputes any of Daphne’s negligence
to Paula.

Use the following facts for questions 9-10.


Bill was walking along an unpaved road on his way to work. The road passed through a
residential neighborhood with fine homes and lovely flower gardens. Suddenly, a pickup truck
coming in the opposite direction began to veer toward Bill; the truck’s driver had momentarily
lost control of the truck while attempting to light a cigarette.
Just as the truck was about to hit him, Bill jumped off the road into Carol’s yard, hurling
himself into a flowerbed containing Carol’s prize-winning zinnias. As he picked himself up from
the now-destroyed flowerbed, his wallet fell out of his pocket, although Bill didn’t realize it at
the time. Later, when he couldn’t find his wallet, he realized that it must be in Carol’s yard. He
called Carol, and she told him to come on over to look for the wallet. However, while looking for
the wallet in the flowerbed, Bill brushed up against an exposed electric wire that was partly
hidden by a bush. Bill was severely burned.

9. If Bill sues Carol for his injuries,

A. Carol is liable, because Bill entered with her permission.


B. Carol is liable, because she failed to repair a dangerous condition on her
property.

89
C. Carol is liable if she failed to reasonably inspect the property and, as a result,
was unaware of the dangerous condition of the wire.
D. Carol is not liable unless she knew of the condition of the wire and failed to
warn Bill of the danger.

10. If Carol sues Bill for damage to her zinnias,

A. Bill is liable for any damage because the first time he went onto Carol’s land,
he had no privilege to do so.
B. Whether Bill is liable depends on whether he exercised due care.
C. Bill may be held liable for damage to the zinnias.
D. Bill is not liable for any damage to the zinnias because the first time he went
onto Carol’s land, his entry was privileged.

11. Gloria was shopping in a grocery store. A banana peel was in the middle of the aisle
in the produce department, but Gloria didn’t see it. She slipped on it, fell, and was
injured. If Gloria sues the store, will she win?

A. Yes, if the banana peel had been on the floor for an hour.
B. Yes, because the banana peel was on the floor.
C. Yes, only if a store employee knocked the banana peel on the floor.
D. No, unless a store employee knew that the banana peel was on the floor.

12. Jesse was a member of an extremely right-wing paramilitary organization. While out
drinking with some friends, Jesse got into a scuffle with a soldier from the nearby
Army base. The soldier easily bested Jesse. Feeling humiliated, Jesse and his friends
went to a different bar and drank a considerable amount of liquor. Once they were
totally boozed up, Jesse and his friends drove out to the Army base to get revenge on
the soldier who had beaten Jesse. They surreptitiously approached what they believed
to be the barracks where the soldier slept. Jesse opened a window; however, as he
was climbing through, a military police officer (MP) happened by and challenged
90
him. Jesse and the MP began to fight, and Jesse struck the MP with the MP’s own
baton. The MP died immediately.

The jurisdiction’s statutes define murder as “the premeditated and intentional killing
of another or the killing of another in the commission of a robbery, rape, burglary, or
arson.” The statutory definition of burglary is identical to the common law except that
the prohibited conduct need not occur in the nighttime. The jurisdiction’s statutes also
provide that intoxication is not a defense to a crime unless it negates an element of the
offense.

Jesse is charged with murdering the MP. At trial, the court should instruct the
jury on the defense of intoxication that:

A. Voluntary intoxication is a defense to the crime of murder if Jesse would not


have killed the MP but for the intoxication.
B. Jesse is guilty of murder despite his intoxication only if the prosecution proves
by clear and convincing evidence that Jesse acted intentionally and with
premeditation.
C. Voluntary intoxication is no defense to the crime of murder.
D. Intoxication is a defense to the crime of burglary if it prevented Jesse from
forming the intent to commit a crime inside the barracks, in which case he
could only be convicted of murder if the prosecutor can show beyond a
reasonable doubt that Jesse acted intentionally and with premeditation.

13. Under which of the following circumstances would the named defendant be least
likely to be convicted of the charged offense?

A. Walter, a chemical engineer, painstakingly constructs an explosive device


from readily available materials and hides it beneath the house where his ex-
wife and her boyfriend are living. He waits until he is sure that both have had
time to get home from their jobs, and then he detonates the device by remote
91
control, totally demolishing the house. Unknown to Walter, his ex-wife and
her boyfriend had impulsively decided to spend the evening in a motel
downtown. Walter is charged with attempted murder.
B. Yvette’s boyfriend had stolen a woman’s purse. Yvette takes one of the
woman’s credit cards to a store and tries to use the card to buy some
expensive items. However, the store’s credit reporting system indicates that
the credit card was stolen, and the store clerk refuses to complete the
transaction. Yvette is charged with attempting to obtain property by false
pretenses.
C. Hazel, seeing a well-heeled couple cross the park, decides to rob them and use
the money for groceries. Hazel approaches the couple, pulls out a gun, and
tells them to hand over all of their money and valuables. However, the couple
was having an unusually bad day; they had just been mugged five minutes
before and were on their way to notify the police when Hazel approached
them. Consequently, they had no money or valuables. Hazel is charged with
attempted robbery.
D. Farley, a married man who believes adultery to be a felony, begins a torrid
affair with Edna, his best friend’s wife. Unknown to either of them, the
jurisdiction they reside in has a statute that expressly states that any sexual act
between consenting adults in private is not a crime. Farley is charged with
attempted adultery.

14. Global Studios was filming part of a movie at a large residential apartment building
with the permission of the building’s owner. To avoid interference by curious
onlookers, Global’s security set up a command post in the lobby of the building. No
persons were allowed to enter the building without identifying themselves and
explaining their reasons for being there. Rezzie, who lived in an apartment in the
building, was returning from a fishing trip late one night. Unaware of Global’s
activities, he was stopped by Global employees as he tried to enter. Because he was
not carrying identification, Rezzie was unable to establish his identity. For this
reason, the employees refused to allow him to enter. After trying unsuccessfully to
92
convince them that he lived there, Rezzie stayed with his sister, who lived a block
away. The following morning, he contacted the building owner, who spoke with
Global officials and arranged to have them allow Rezzie to enter.

If Rezzie asserts a claim against Global for false imprisonment, which of the
following would be Global’s most effective argument in defense?

A. Global employees did not know that Rezzie was entitled to enter the building.
B. The conduct of Global employees was not unreasonable.
C. Rezzie was not imprisoned.
D. Rezzie sustained no damage as a result of the conduct of Global employees.

Questions 15 and 16 are based on the following fact situation.

Poe ordered some merchandise from Store. When the merchandise was delivered, Poe
decided that it was not what he had ordered, and he returned it for credit. Store refused to credit
Poe’s account, continued to bill him, and after 90 days, turned the account over to Kane, a bill
collector, for collection.
Kane went to Poe’s house at 9 p.m. on a summer evening while many of Poe’s neighbors
were seated on their porches. When Poe opened the door, Kane, who was standing just outside
the door, raised an electronically amplified bullhorn to his mouth. In a voice that could be heard
two blocks away, Kane called Poe a “deadbeat” and asked when he intended to pay his bill to
Store.
Poe, greatly angered, slammed the door shut. The door struck the bullhorn and jammed it
forcibly against Kane’s face. As a consequence, Kane lost some front teeth.

15. If Poe asserts a claim against Kane based on intentional infliction of emotional
distress, will Poe prevail?

A. Yes, if Kane’s conduct was extreme and outrageous.


B. Yes, because Kane was intruding on Poe’s property.
93
C. No, unless Poe suffered physical harm.
D. No, if Poe still owed Store for the merchandise.

16. If Kane asserts a claim for battery against Poe, will Kane prevail?

A. Yes, because Poe had not first asked Kane to leave his property.
B. Yes, if Poe knew that the door was certain to strike the bullhorn.
C. No, if Kane’s conduct was extreme and outrageous.
D. No, because Poe doesn’t owe the store any money.

17. Tabak, Gruner, and Kahan, all minors, stood on the edge of a dock, watching boats
enter and leave the bay. They decided it would be fun to throw rocks at passing
vessels. To show off his superior rock-throwing skills, Gruner picked up a large rock
and threw it. It landed next to Skipper, who was sleeping on the deck of his yacht.
Gruner picked up another rock, corrected his aim, and threw it. The second rock
landed even closer to Skipper. Gruner picked up a third rock and hurled it directly at
Skipper. That rock hit Skipper right in the face. If Skipper sues Gruner, Skipper
will

A. Win, because Gruner committed a battery.


B. Win, because Gruner committed an assault.
C Lose, because Gruner was a minor.
D. Not win, unless Skipper also sues Gruner’s parents.

Questions 18 and 19 are based on the following fact situation.

Customer went into Store at approximately 6:45 p.m. to look at some suits that were on
sale. The clerks were busy and did not notice that he came in. Customer selected three suits from
a rack and went into a dressing room to try them on.
Signs posted on the walls of the Store stated that closing time was 9:00 p.m.; however,
because of a special awards banquet for employees, Store was going to close at 7:00 p.m. on this
94
day. The employees, in a hurry to get to the banquet, did not check the dressing rooms or turn off
the lights before leaving. When Customer emerged from the dressing room a few minutes after
7:00, he was alone and locked in. Customer tried the front door, but it was secured on the outside
by a bar and a padlock, so he went to the rear door. Customer grabbed the doorknob and
vigorously shook the door. It did not open, but the shaking set off a mechanism that had been
installed because of several recent thefts committed by people who had hidden in the store until
after closing time. The mechanism sprayed a chemical mist in Customer’s face, causing him to
become temporarily blind. The mechanism also activated an alarm. A night watchman was just
coming to work and heard the alarm. The watchman unlocked the front door, ran into the store,
and grabbed customer. The watchman then identified himself, and Customer did the same. After
assuring himself that Customer was telling the truth, the watchman allowed him to leave.

18. If Customer is to prevail on a claim against Store based on battery from the use
of the chemical spray, Customer must establish that

A. He suffered severe bodily harm.


B. The spray mist was an offensive or harmful contact.
C. He suffered severe emotional distress.
D. His conduct did not cause the chemical to spray him.

19. If Customer sues Store for false imprisonment, Customer will

A. Win, because he could not get out.


B. Win, because he had no reasonable avenue of escape.
C. Lose, because he was not injured.
D. Lose, because Store’s employees did not know that he was in the dressing
room.

20. Professor Merrill, in a lecture hall in her psychology course at a private university,
described an experiment in which a group of college students rushed out and washed
cars stopped at traffic lights during the rush hour. She described how people reacted
95
differently – with shock, fear, joy, or surprise. At the conclusion of her lecture, she
said, “You understand, of course, that you are not to undertake this or any other
experiment unless you first clear it with me.” Four of Merrill’s students decided to try
the same experiment but did not clear it with Merrill.

One subject of their experiment, Carr, later said, “I was shocked. There were two
people on each side of my car. At first, I was afraid. I thought they were going to
attack me, and I even thought of driving away. Then I quieted down and decided that
there were too many dirty cars in the city anyway.”

If Carr sues the students who washed his car, his best theory is

A. Assault
B. Battery
C. Conversion
D. False Imprisonment

96
Questions 21 through 23 are based on the following fact situation.

Husband and Wife, walking on a country road, climbed over a fence to get onto the
adjacent property, owned by Grower. Husband and Wife did not know whose land it was. After
climbing over the fence, Husband and Wife damaged some of Grower’s plants, which were near
the fence. Grower saw Husband and Wife and came toward them with his large watchdog on a
long leash, shouting that he was going to sic his dog on them if they didn’t leave. The dog rushed
at Wife. Grower had intended only to frighten Husband and Wife, but the leash broke, and before
Grower could restrain the dog, the dog bit Wife.

21. If Grower sues Husband and Wife for trespass to land, he will

A. Lose, because Husband and Wife did not do enough damage.


B. Win, if Husband and Wife were there without Grower’s permission.
C. Lose, because there were no “No Trespassing” signs on the fence.
D. Lose, because Husband and Wife did not know the land was Grower’s.

22. If Wife sues Grower for battery, will Wife prevail?

A. Yes, because Grower wanted the dog to frighten Wife.


B. Yes, because the bite caused severe injury.
C. Yes, because Wife saw the dog coming at her.
D. Yes, because Grower meant to confine Wife inside the fence.

23. If Husband sues Grower for assault, will husband prevail?

A. No, because Grower’s dog did not bite Husband.


B. Yes, but only if Husband was afraid for his life.
C. Yes, if Husband saw the dog coming at him.
D. No, because Grower did not mean for the dog to touch Husband.
97
24. Which of the following is a battery?

A. Dan steps into a crowded elevator, and his briefcase touches Paula’s leg. Paula
is a stranger.
B. Dan taps Pam, a stranger, on the shoulder and asks whether she can tell him
the time.
C. Dan pats Pauline, a stranger, on the bottom.
D. Dan pats his wife Peggy on the bottom.

25. Which of the following is an assault?

A. A points a gun at B while B is looking at a classic car that is driving by.


B. A raises his fist and starts walking toward B, who is looking at A from four
feet away.
C. A tells B, “If you do that again, I’ll punch you in the nose.”
D. A yells at B, “I hate you!”

26. In which of the following will the plaintiff not win?

A. Defendant built his summer cottage near plaintiff’s property line. Defendant
did not know it, but one corner of his porch extended a few inches onto
plaintiff’s property. Plaintiff sued for trespass to land.
B. Defendant loaned money to plaintiff, but plaintiff didn’t pay it back on time.
Defendant met plaintiff on the sidewalk, where plaintiff was walking his toy
poodle on a leash. Defendant scolded plaintiff, which upset the poodle. The
poodle growled and barked. Defendant struck the poodle with his umbrella,
which caused a gash on the poodle’s face. Plaintiff sued for trespass to
chattels.
C. Plaintiff’s employee took a stereo system from plaintiff’s warehouse without
paying for it. The employee used it for a week, then sold it to defendant.
98
Defendant was unaware that the employee did not rightfully own the stereo.
Defendant later sold the stereo to another person. Plaintiff sued defendant for
conversion.
D. Defendant assaulted Robby in plaintiff’s full sight. Defendant thought that
Robby was a mere passerby, but plaintiff was, in fact, Robby’s brother.
Plaintiff sued defendant for IIED.

27. Joe went to a store and started asking questions in a loud and abrasive manner.
Salesman said to the Store Owner, “This guy has got to be drunk. What a jerk.” Joe
heard what Salesman said and was highly insulted. He said to Salesman, “If I weren’t
in such a good mood, I would flatten your face.” Joe then prodded Salesman’s
shoulder with his index finger.

If Salesman sues Joe, his best cause of action will be

A. Assault
B. Battery
C. Trespass to land
D. IIED

28. Which of the following is true?

A. Defendant was up to bat in a softball game. Plaintiff, the pitcher, struck him
out. As defendant walked back to the dugout, he shouted, “I’m gonna kill that
pitcher!” If plaintiff sues for assault, plaintiff will win.
B. Defendant was watching his girlfriend, who was across the street, and held up
his hand to wave at her. Plaintiff, who was just in front of defendant, thought
that defendant was about to hit plaintiff. Plaintiff was terrified. If plaintiff sues
for assault, plaintiff will win.
C. Defendant was a convicted murderer. He was in a holding cell in the
courthouse. Plaintiff, the judge’s secretary, walked by. The defendant shouted,
99
“I’m gonna get you, baby!” He stretched his hand out between the bars to grab
plaintiff, but he was at least six feet too far away. If plaintiff sues for assault,
plaintiff will win.
D. Defendant walked into a bar. He saw plaintiff at the other end of the room.
Defendant ran toward plaintiff with his fist raised, but tripped over a chair and
fell down when he was halfway to the plaintiff. If Plaintiff sues for assault,
plaintiff will win.

29. Flyer was a helicopter pilot employed by a radio station as a traffic reporter. One day,
while flying in his helicopter, he hovered over the home of Pauline. Using powerful
binoculars, he looked into her window to watch her while she was exercising in the
nude. If Pauline sues him for trespass to land, which of the following facts or
inferences must she prove to win the case?

A. The altitude at which Flyer hovered overhead.


B. Damage to her land resulting from Flyer’s conduct.
C. That she had a reasonable expectation of privacy while exercising nude in her
own home.
D. That Flyer knew with substantial certainty that the land was Pauline’s.

30. Dan owned a meat-packing plant. One day, he summoned Peggy, a long-time
employee, to his office. Dan accused Peggy of having SARS and demanded that
Peggy resign from her job. Peggy did not, in fact, have the disease, nor did Dan have
any reason to believe that she did. To recover for intentional infliction of emotional
distress, Peggy can win if she can prove that

A. She was extremely sensitive to allegations that she carried diseases.


B. She was extremely sensitive to allegations that she carried diseases, and Dan
was careless in not realizing this.
C. Accusing somebody of having SARS can cause that person to suffer
emotional distress.
100
D. Dan knew that accusing her of having SARS would cause severe emotional
distress to an average person.

The following facts apply to questions 31 and 32.

Donald placed an ad in the newspaper, stating that he was looking for a waitress between
sixteen and seventeen years old. Pat, sixteen years old, answered the ad, but she was hesitant
when she heard that the job involved making deliveries to the top floors of many construction
sites in the downtown area. Pat met Donald at the foot of a forty-story office building under
construction. “I’m scared to death, but I need the money badly to help feed my little brothers,”
she said. Donald ordered Pat to walk along a steel beam, four hundred feet above the street, and
hand a can of soda pop to one of the workers. Pat was afraid for her life, but she made it. On the
way down in the elevator, Pat asked, “Did I get the job?” Donald laughed, answering, “There is
no job. I was only kidding. I’m a construction worker. I do this all the time for kicks.” Pat had a
nervous breakdown that required a six-month hospital stay.

31. If Pat sues Donald for assault, which of the following will be the best evidence of
his intent?

A. Donald knew that the walk was dangerous.


B. Donald knew that Pat thought she might fall.
C. Donald wanted to fool Pat.
D. Donald had intended for Pat to feel severe emotional distress.

32. Which of the following is incorrect?

A. Donald lacked any intent to inflict emotional distress.


B. It is possible for Pat to recover for severe emotional distress even though she
suffered no physical injuries.
C. Donald’s actions might be “extreme and outrageous conduct.”
D. An action for IIED can arise from an act that was supposed to be a joke.
101
33. Patsy was divorced from her husband Joe, and she had custody of their four-year-old
son. Patsy often let the son spend weekends with Joe at the home of Joe’s father,
Dick. But one weekend, when the son was staying with Joe at Dick’s house, Patsy’s
friend called up and said that she had heard that Joe was going to remove the son
from the state permanently.

Panicked, Patsy ran to Dick’s house and pounded on the door. When Dick came to
the door, Patsy demanded that Dick tell her where Joe and the son were. Dick knew
that Joe had taken the son to the movies and would be back soon, but Patsy’s manner
scared him, so Dick said that he had no idea where they were or when they were
coming back. After Patsy left, she became highly upset. She went to the doctor, who
gave her a strong tranquilizer. However, she remained upset until Joe brought the son
back that evening.

If Patsy sues for false imprisonment, who should win?

A. Dick, because Patsy did not suffer any physical injury.


B. Dick, because he did not prevent Patsy from leaving his home.
C. Patsy, because she was entitled to legal custody of her son.
D. Patsy, because Dick had kept her from seeing or communicating with her son.

34. Penelope owned a beautiful tract of land in the Upper Peninsula of Michigan. The
property was purchased by Penelope and used as a family get-away. About 200 yards
of Penelope’s property was beachfront along Lake Superior. Donald lived along a
stream that separated his land from Penelope’s. The stream flowed into Lake
Superior. When Penelope bought the land, she had a channel dredged across her land
from the stream to the lake at a point some distance away from the mouth of the
stream where it emptied into Lake Superior. Donald, unaware that the channel was
not a public waterway, frequently used it as a shortcut to the lake where he enjoyed

102
the fishing. Donald’s use of the channel in no way caused any damage to Penelope’s
channel or to her land.

After Penelope learned about Donald’s use of the channel, she asked Donald not to
use the channel any more. Surprised at learning the channel belonged to Penelope,
Donald agreed not to use it any more.

If Penelope sues Donald for trespass to land, who will win?

A. Donald, because when he used the channel, he believed it was a public


waterway.
B. Donald, if he discontinued using the channel after learning of Penelope’s
ownership claim.
C. Penelope, but recovery is limited to nominal damages for Donald’s intentional
use of the channel.
D. Penelope, unless Donald had no other navigable access to the lake.

35. When the traveling circus came to town, an elephant escaped its restraints and began
wandering through the town. Bill and Ted were on their patio drinking a few pina
coladas when the elephant passed through the back yard of their apartment building.
Even though they believed they were hallucinating, Bill told Ted to call the police.
Due to their quick action, the police were able to capture the elephant, but not before
the elephant trampled through the landlord’s garden, destroying the very expensive
landscaping.

If the landlord sues the circus owners to recover damages to the landscaping
caused by the elephant, who is likely to prevail?

A. Landlord, because the circus is strictly liable for the elephant’s trespass.
B. Landlord, but the circus is subject to liability for trespass only.
C. Landlord, but the circus is subject to liability for nuisance only.
103
D. Landlord, only if the circus failed to exercise the utmost care to confine the
elephant or otherwise prevent it from escaping.

The following facts apply to questions 36 and 37.

Phil Powerhouse was the star weightlifter for the Cooley Heavyweights. After coming in
last place at an important meet, Coach Dan dropped him from the squad. Powerhouse met with
Coach Dan and asked to be let back on the team. Coach Dan told Powerhouse that he would
never let him back on the team because everybody hated him. As Powerhouse was sadly walking
out of the weight room, Coach Dan said to him, “You know Powerhouse, I hope you transfer to
DCL because they deserve a loser like you.”

Later that night, Powerhouse wrote a suicide note in which he stated, “Coach Dan was
responsible for my despondency. If I can’t be a weight lifter for Cooley, I don’t want to live.”
After swallowing a quart of tequila with a dozen sleeping pills, Powerhouse passed out and fell
to the floor in his dorm room. A few minutes later, Powerhouse’s roommate Pete Parker came in
and saw Powerhouse on the floor. Pete read the suicide note and attempted to administer aid.
Having no luck, Pete picked up Powerhouse and carried him to the first aid center. Powerhouse
recovered from his overdose, but Pete suffered several compressed vertebrae in his back by
carrying Powerhouse’s 260-pound body.

36. If Powerhouse asserts a claim against Coach Dan for his injuries, who will win?

A. Powerhouse, if Coach Dan intended to cause him to suffer emotional distress.


B. Powerhouse, because Coach Dan’s remark did in fact cause Powerhouse to
suffer emotional distress.
C. Coach Dan, because Powerhouse’s overdose resulted from his own voluntary
act.
D. Coach Dan, unless he knew that Powerhouse was an extremely sensitive
person.

104
37. If Pete Parker asserts a claim against Powerhouse to recover damages for his
injuries, who will win?

A. Powerhouse, because he did not intend to harm anyone other than himself.
B. Pete, unless attempted suicide was a criminal offense in the jurisdiction.
C. Pete, because Powerhouse intentionally put himself in a position of peril.
D. Pete, because he succeeded in saving Powerhouse’s life.

38. Sandstone had a distinctive accent that Dave, also known to his friends as Weasel,
could recognize anywhere as being from the south side of Cooleyville. Unfortunately
for Sandstone, he was a really bad bookie who never seemed to have enough money
to pay his gambling debts. Dave won a $1,000 bet on a football game from
Sandstone, which made Sandstone angry. That night, while drinking in a dimly lit
bar, Dave heard a voice: he knew it was Sandstone’s because of the unique south-side
Cooleyville accent. Suddenly, the voice grew loud and sounded angry. Dave heard
the voice say in a drunken tone, “I’m going to pound that little Weasel to a pulp!” A
figure suddenly loomed out of the next booth. Fearing that he would be attacked,
Dave leaped from his chair at the figure and punched him in the face, hearing his nose
break. The bar manager heard the commotion and turned up the lights. Dave learned
to his horror that he had actually knocked out Billy Joe Crooner, a world-famous
country western singer who was unwinding after a local concert in town.

Crooner sued Dave for battery, and Dave claimed self-defense. Who should win
this case?

A. Dave, if he honestly believed his action was necessary to prevent an attack on


his person.
B. Dave, if he reasonably believed such action in self-defense was necessary.
C. Crooner, because an aggressor may never rely on self-defense.
D. Crooner, because Dave attacked him without provocation.
105
39. Tony Drugdealer learned that his next-door neighbor Peter Pious was selling his
house to a fellow church member. Drugdealer went to Pious’s house and told Pious
that Pious, his wife, and children would all meet with “unfortunate accidents” if he
sold to another church-going family. Pious then called his fellow church-goer and
told him that the house was no longer available because he, Pious, had decided to take
the house off the market.

If Pious sues Drugdealer for assault, will he win?

A. Yes, if Drugdealer intended to place Pious in fear of physical harm.


B. Yes, because Drugdealer’s conduct was extreme and outrageous.
C. No, if Drugdealer took no action that threatened immediate physical harm to
Pious.
D. No, because Pious’s actions removed any threat of harm.

40. Dante just received a letter from Thomas M. Cooley Law School informing him that
he had been accepted to begin law school in the Michaelmas term. He immediately
informed his friends and scheduled a big party. During the party, Dante decided to
“get rowdy” and began to shoot his Smith and Wesson .357 Magnum into the air.
Down the block, Penny was lounging by her swimming pool and heard the gunshots.
As she headed into the house to use the telephone to call the police, she noticed that
her dog was lying on the ground with blood all around. Penny was very upset because
her prize poodle had been struck by the bullet shot by Dante.

If Penny sues Dante, will she win?

A. No, unless Dante meant to cause Penny apprehension about leaving her dog
alone by the poolside.
B. Yes, but only if the dog died as a result of the gunshot.
C. No, because shooting the gun in the air was not extreme and outrageous.
106
D. Yes, if the bullet hit the dog in Penny’s yard.

41. On January 1st, Shaftum and Howe Realty Corp. mailed a written offer to
Martin Enterprises for the sale of a large tract of land. The offer included the
following terms:

This offer expires on February 1st, if the offeree has not caused an acceptance
to be received by the offeror on or before that date.

The first thing in the morning, on February 1st, Martin sent a telegram of acceptance,
but the telegraph company negligently withheld delivery to Shaftum until February
2nd. On February 4th, Shaftum entered into a contract for sale of the tract to another
buyer but did not inform Martin of the transaction. As a result, Martin contacted
Shaftum by phone on February 10th and was told that no contract between Shaftum
and Martin existed. Which of the following is the most correct statement?

A. No contract between Shaftum and Martin arose on February 2nd.


B. A contract would have arisen if a letter of acceptance had been mailed on
February 1st.
C. Shaftum’s silence constituted an acceptance of Martin’s telegram on February
2nd.
D. A voidable contract arose on February 1st.

Questions 42 and 43 are based on the following fact situation.

Fox mailed the following offer to Sack, a designated offeree: “I hereby offer to sell my property
consisting of a house and lot at 337 Green Street for $100,000. Terms are $30,000 cash, the
balance secured by a first mortgage. Advise immediately if you accept.” This offer was mailed
on February 3rd, and it reached Sack on February 5th. On February 8th, Sack replied: “Your offer
received and under advisement. I would much prefer a straight cash deal. Would you consider
107
an immediate purchase for $90,000 cash?” On February 10th, Fox received this reply, and
immediately responded with a one-word telegram: “No.” Receiving this telegram on February
11th, Sack wired back: “Telegram received. I accept your offer of February 3rd. Tender the deed
at closing.”

42. If Fox now refuses to sell and Sack sues, the court would probably hold that

A. A valid contract exists.


B. No contract exists because Sack’s response of February 8th operated to
terminate Fox’s offer.
C. No contract exists because Sack’s communications to Fox both contained
alterations of the terms of the offer.
D. No contract exists because the offer relates to real estate, and the
communications failed to establish the terms of the proposed agreement with
sufficient definiteness.

43. Assume that the day after Fox mailed his offer to Sack, Fox mailed a revocation,
which arrived one day after the offer. As the mail carrier handed the letter of
revocation to Sack, Sack simultaneously handed his unqualified acceptance to the mail
carrier. What result?

A. The revocation was effective upon mailing, and the acceptance would be
treated as a counteroffer.
B. The acceptance was effective, as long as Sack had no knowledge of the
contents of Fox’s letter when he handed his letter to the mail carrier.
C. The outcome would depend on the court’s determination of whether Fox’s
letter had been received by Sack before he entrusted the letter of acceptance to
the mail carrier.
D. Handing a letter to a mail carrier is not a proper posting of the acceptance;
hence, Sack’s purported acceptance was not timely.

108
Questions 44-46 are based on the following fact situation.

At the start of the school year, September 1st, Professor announced, in class, that he
would buy a bar review course for the student who received the highest grade in Professor’s
Constitutional Law class. The next day, Professor put this offer in writing and posted it on the
student bulletin board. The notice stated the same essential terms with the added statement,
“Maximum award, $1,000.”

Stillwell, a student in Professor’s class, told Professor before class began on September
8th, “I’m really going to give my best effort to get the highest grade in your class and win that
prize. I started studying three days ago, and yesterday I purchased every constitutional law
outline the bookstore had, just so that I can win that prize.”

Shortly afterward, the Dean of the law school told Professor that she had been receiving
complaints from many of the other law professors: students were no longer studying for their
other classes because they were concentrating all their efforts on Constitutional Law. The Dean
also stated that the Professor’s offer could be interpreted as an improper school policy. Professor
stated that, although he still wished to perform on his agreement, he would post a notice
announcing the withdrawal of his offer. Professor posted the withdrawal notice on October 1st
and told his class why he was forced to withdraw the offer. He apologized, but said that in the
interest of the school, he had no choice.

Later that day, Stillwell told Professor that the offer could not be withdrawn after he,
Stillwell, had started to perform. Professor again stated that he had no choice. Stillwell received
the highest grade in Professor’s class and now requests performance by Professor.

44. Professor’s offer to his class on September 1st and the notice posted on the
bulletin board the next day would be interpreted as:

A. Preliminary negotiations.
B. A promise to make a conditional gift.
109
C. A contractual offer, creating a power of acceptance.
D. A mere statement designed to induce the students to study harder.

45. If the Professor’s announcement to his class were held to be an offer, it would be
an offer for

A. A bilateral contract.
B. A unilateral contract.
C. A bilateral contract or a unilateral contract, according to the offeree’s
intentions.
D. A unilateral contract that became a bilateral contract when Stillwell began to
perform.

46. Professor’s announcement to his class of the withdrawal of the offer and the
posting of the withdrawal notice on the bulletin board would most likely be
interpreted as

A. Having no legal effect, because no offer had been made.


B. An effective revocation of the offer.
C. An ineffective revocation as to any student to failed to hear Professor’s
announcement or read the notice on the bulletin board.
D. An ineffective revocation as to Stillwell because Stillwell relied on the
promise.

110
Practicing Multiple Choice

Time Finished: ____________ Name:______________________________

Time Began: ______________

Total Time: _______________ % Correct:___________________________

Mis- Misread
Question Your Correct marked transition Misread Didn’t know the law or misapplied the law
Number Answer Answer answer word Facts (write the correct law here)

111
4

10

My analysis of the practice


Practicing Multiple Choice

Time Finished: ____________ Name:______________________________

Time Began: ______________

Total Time: _______________ % Correct:___________________________

Mis- Misread
Question Your Correct marked transition Misread Didn’t know the law or misapplied the law
Number Answer Answer answer word Facts (write the correct law here)

112
4

10

My analysis of the practice


Practicing Multiple Choice

Time Finished: ____________ Name:______________________________

Time Began: ______________

Total Time: _______________ % Correct:___________________________

Mis- Misread
Question Your Correct marked transition Misread Didn’t know the law or misapplied the law
Number Answer Answer answer word Facts (write the correct law here)

113
4

10

My analysis of the practice


Practicing Multiple Choice

Time Finished: ____________ Name:______________________________

Time Began: ______________

Total Time: _______________ % Correct:___________________________

Mis- Misread
Question Your Correct marked transition Misread Didn’t know the law or misapplied the law
Number Answer Answer answer word Facts (write the correct law here)

114
4

10

My analysis of the practice


Practicing Multiple Choice

Time Finished: ____________ Name:______________________________

Time Began: ______________

Total Time: _______________ % Correct:___________________________

Mis- Misread
Question Your Correct marked transition Misread Didn’t know the law or misapplied the law
Number Answer Answer answer word Facts (write the correct law here)

115
4

10

My analysis of the practice


Practicing Multiple Choice

Time Finished: ____________ Name:______________________________

Time Began: ______________

Total Time: _______________ % Correct:___________________________

Mis- Misread
Question Your Correct marked transition Misread Didn’t know the law or misapplied the law
Number Answer Answer answer word Facts (write the correct law here)

116
4

10

My analysis of the practice


Practicing Multiple Choice

Time Finished: ____________ Name:______________________________

Time Began: ______________

Total Time: _______________ % Correct:___________________________

Mis- Misread
Question Your Correct marked transition Misread Didn’t know the law or misapplied the law
Number Answer Answer answer word Facts (write the correct law here)

117
4

10

My analysis of the practice


Practicing Multiple Choice

Time Finished: ____________ Name:______________________________

Time Began: ______________

Total Time: _______________ % Correct:___________________________

Mis- Misread
Question Your Correct marked transition Misread Didn’t know the law or misapplied the law
Number Answer Answer answer word Facts (write the correct law here)

118
4

10

My analysis of the practice


Maximizing Memory
(or “How Am I Going To Remember All this Stuff?”)
Overview: In college, most students were able to succeed by “cramming” just before
exams. They memorized enough information to get through each exam. After the exam was
over, much of the information disappeared from memory. But that was usually okay because
many college courses of study do not require students to build on information from previous
semesters.

The memory demands in law school are different in at least four important ways.

(1) Depth. Rote memorization was sufficient for most high school and college
courses. However, law school requires you to use what you learn to analyze new
situations. Thus, the depth of your understanding has to be much greater that ever
before. On the other hand, you will have to memorize a great deal of information.
Simply understanding the concepts will not be enough if you have not memorized
the details that flesh out each concept.

(2) Quantity. The amount of information for which you will be held responsible
in each course is at least triple what you had to know in a college course.

(3) Continuity. Some law school classes build on classes that you have already
taken. For these courses, you will have to know old concepts to learn new
concepts. Some courses do not build on others, but they do build on information
that you learn throughout the course itself. If you do not learn the information as
you go, you cannot understand how the parts relate to each other.

(4) The Bar Exam. Most law school graduates take at least one bar exam, and
that bar exam tests most of the law school curriculum. Taking the bar exam is
bad enough: having to learn three years’ worth of material right before the bar
exam is horrifying.

The purpose of this lesson is to summarize memory techniques that you can use (and that you
may already be using) to maximize your memory. You will notice that some parts of this section
repeat ideas already covered in this course. That is because much of this course is built on these
proven memory techniques.

How Memory is Built:

The following chart (“The Memory Chain”) demonstrates the stages of human memory.

119
The Memory Chain

Sensory Storage
Record and Fixate
7-second storage

Short-Term Memory
20-minute duration
Must organize, recite, and
review to move into long-term
memory

Long-Term Memory
Store and file
What you use remains in active
memory storage

Active Storage Inactive Memory


Find and Retrieve Can’t Retrieve

120
(1) Sensory Storage (also called immediate memory). Every moment, your mind is bombarded
with thousands of stimuli from both inside and outside your body. The light in the room, the air
temperature, sounds, thoughts, the rumbling of your stomach, the soreness of a muscle –
thousands of tiny stimuli are competing for your brain’s attention. Most of these stimuli pass by
without your ever noticing them consciously. However, if you focus on a stimulus, you put it
into your sensory memory.

Example of sensory memory: Calling for pizza delivery

Have you ever looked up the number for pizza delivery and then been distracted by something
before you could dial? Did you forget the number and have to look it up again?

When you focused on the number, the number went into sensory memory. Sensory memory will
retain the number just long enough to dial it – approximately seven seconds. But if you get
distracted before you can dial, the number evaporates from your memory. The only way to keep
the number beyond about 7 seconds is to do something with it – write it down, keep repeating it,
think about relationships among the numbers, or do something else to keep the number in your
memory. This will move the number into short-term memory.

Remember, sensory memory is only the first step in the memory chain. If information does not
first move into sensory memory, it can never get into any subsequent stage of memory. And
information only moves into your sensory memory if you focus on it.

The problem of focus in law school: Imagine that you are in one of your
law school classes. The professor is lecturing, and you are taking notes.
Your immediate problem is one of focus. If you are daydreaming, you
will miss important information. But even if you are not daydreaming,
focus is still a problem because you have to learn what to focus on.

In college, this was not a problem because you knew from experience what parts of the
information were important. It’s also likely that your professors used familiar visual and
verbal cues to tell you what was important. They wrote on the board, they gave you handouts,
and they said things like, “This is important!”

In law school, the parts are different. The patterns are different. The questions you must
answer are different. You need to focus on elements, rules, specific facts, specific pieces of
reasoning, the process of legal reasoning, social policy, etc. – and at first these are likely to be
unfamiliar. Much of the difficulty in law school centers on your having to learn to focus on
different information than ever before. If your attention is focused on the wrong things, you
cannot learn the right things: your brain can only record and analyze what you have focused
on.

121
Calling for pizza delivery – revisited.

In the pizza delivery example given above, pretend the telephone number is 456-5000. How
could you move this telephone number into your short-term memory? Organize the information,
recite it, or both.
1. Write it down or keep repeating it. (Recite.)
2. Look at the parts and associate them with something. If it’s a local number, the “456”
part might be easy to remember. “55” is the year the Brooklyn Dodgers finally won the
World Series (if you don’t like baseball, think of some other reason that “55” might be
significant). And “00” is easy to remember because many business numbers end with
“00.” Then repeat the parts to yourself. (Organize and recite.)
3. Think about the relationships among numbers. “456” is easy because those three numbers
are all in a row. “55” is easy because it just repeats the middle number from “456.” If you
add 5 and 5, you get 10, which ends in 0. That gives you “00.” Then repeat these parts to
yourself. (Organize and recite.)
4. If you are musically inclined, put the numbers on a musical staff and think about the tune
they will create (as in Close Encounters of the Third Kind) If you are mathematically
inclined, you can put the numbers on a graph and think about the pattern they create.
(Organize and recite.)

______________________________________________
______________________________________________
6
______________________________________________
5 5 5
______________________________________________
4
______________________________________________
0 0

9
8
7
6 Ξ
5 Ξ Ξ Ξ
4 Ξ
3
2
1
0 Ξ Ξ

122
(2) Short-term memory only lasts for approximately twenty minutes. The “shortness” of short-
term memory explains why good note-taking skills are so important. You may completely
understand everything that is going on in class, but you will lose most of the information from
your short-term memory. Taking good notes will allow you to further manipulate the information
so that you can put it into long-term memory.

(3) Long-term memory. The final stage is long-term memory. This is memory that can
potentially last a lifetime. However, not all long-term memory is useful because not all long-term
memories are retrievable. How do you create retrievable long-term memory? Two things are
necessary: the first is having the right attitude; the second is using the right process.

The Right Attitude: Planning to Remember. Planning to remember means that you
actually intend to remember information as you encounter it. Many law students decide
not to remember materials that they study until they start to make outlines, and some
students don’t start outlines until week 12. That’s far too late. It accounts for the sinking,
stressful, overwhelmed feeling that many students report.

The Right Process: SOS. Three steps are necessary to build long-term memory. Not
coincidentally, you use most of these steps to move information into your sensory and
short-term memory. The four steps are:

1. Select the information to remember. (Focus!)


2. Organize the information to see the Big Picture, find the parts, and see how the
parts fit.
3. Space the learning over time by reciting the information, out loud or in writing,
frequently.

The following outline describes these steps and how each step fits with law school.

SOS

I. Select the important information. You won’t be able to memorize every single detail
of every single case. So you have to use judgment to decide what is really important,
what is somewhat important, and what is not important at all.

A. Reading assignments
1. Preview each case by using these questions: What is the law? What
are the rules? What facts fit the law? How did the court reason?
What was the outcome of the case?

2. Then read, underline, and take notes.

3. For that day’s assignment, think about (a) what pieces of law each
case, statute, Restatement, etc., contained and (b) the relationship
123
between the law and facts for each case. Write this information down
in an outline.

4. For each class, memorize this outline and the relationship between
the law and facts. Then you will be ready to focus on the important
information in each class.

B. Lecture notes
1. As the professor lectures, continually ask yourself what topic the
professor is discussing.

2. Take notes right into your case brief. Include questions the professor
asked and the professor’s hypotheticals. Do not assume that the
professor will write all the important information on the board.

3. After the lecture, go through and edit your notes. Find the main
points and sub-points and highlight them. Then summarize them in
the margin with questions, cue words, and acronyms.

II. Organize. This is the step that you were unlikely to have done in college because the
textbook, the teacher, or both, probably organized the information for you.

A. Get the Big Picture first. Then find the “parts” that make up the topics you are
studying.

B. Sort out which parts go with each topic, and then organize the parts for each
topic.

C. Think about what each concept means: what are the critical attributes of each
concept; what are examples of each concept; what would not be an example of
each concept; how is the concept different from other, related concepts; and
how are the concepts related? (See the section on “Concept Development
and Data-Organization Devices.”)

D. Put what you are studying in a form that shows the material’s organization
and how the parts relate to each other. This may be a numbered list, a chart, an
outline, or a graphic organizer such as a T-chart. (See the section on “Concept
Development and Data-Organization Devices.”)

E. The number of parts in each section, subsection, or sub-subsection should not


exceed 7, plus or minus 2. (See material on “Chunking Information.”)

F. Use mnemonics sparingly. Mnemonics are groups of letters used to prompt


memory. (Example: HOMES – mnemonic for the names of the Great Lakes.)
Because they do not usually show how ideas are related, mnemonics are

124
useful only when it is impossible to further chunk the information into
categories.

III. Space the learning over time. To do this, recite the information so that you can learn
the details. Recitation is oral (or written) review that is done from memory. It is not
just reading your notes. Reading your notes over and over, without more, probably
means that you are testing whether you understand your notes. This will not help on
an exam because the exam requires you to output the information. Recitation enables
you to practice outputting the information.

A. Recite (talk out loud or write out) from memory your notes the same day that
you take them. Recite this new material every day. Once you have it down
pat, recite it once a week to keep it fresh.

B. Turn the headings in your outline into questions. Answer those questions out
loud without looking at your outline. Then check to see whether you recited
accurately. If not, try again.

C. Recreate the Big Picture in your memory. Whether you use a picture or a
traditional outline, visualize all the big concepts and how they fit together;
then visualize the parts that make up each concept; then visualize the details
that go with each part.

D. Make the organization of your notes and outline visible by using highlighting,
underlining, stars, stickers, numbered lists, charts, etc. Make the organization
of the ideas clear.

Space the learning: People remember best when recitation


and review are spaced over time. Therefore, it is best to recite
and review daily over new material and weekly over older
material.

125
“Chunking” Information

Explanation:
Miller (1954) noted that the immediate memory of an adult appears to be fixed at approximately
seven (plus or minus two) separate pieces of information. So adults tend to remember between
five and nine separate pieces of information from a category. Whether the pieces or categories of
information are big or small doesn’t matter; only the number of pieces of information matters.
This immediate memory capacity has been referred to as the “magical number seven.” Baddeley,
who reviewed the literature on immediate memory in 1994, reiterated Miller’s results. The
“magical number seven” still holds true.

Application:
Chunking information into categories helps you to remember the information. Outlining is
chunking information because it requires you to think about the relationships among ideas.
Thinking about relationships among ideas facilitates “deep processing” of the ideas.

Steps:

(1) Organize information you must learn into seven, plus or minus two, categories (chunks).

Example: There are seven intentional torts. So that could be one category. Or you could divide it
into two categories: (1) intentional torts against people and (2) intentional torts against property.

(Cont’d on next page)

Baddeley, A. (1994). The “magical number seven”: Still magical after all these years?
Psychological Review, 101, 353-356.
Miller, G. (1954). The Magical number seven, plus or minus two: Some limits on our
capacity for processing information. Psychological Review, 63, 81-97.

126
“Chunking” Information (continued)

(2) Add bits of information that go with each category. By remembering the categories, you
will likely remember the bits of information within the categories.

Example: One intentional tort against people is battery. So the name of this category is “Battery.”
Battery has three parts: (1) intent, (2) contact, and (3) harmful or offensive. These are some of
the details that go with this category. Note that there are only three, which is well within the
“magical number seven.” The first sub-part of this category, intent, has three rules because it can
be proved in three ways: (1) purpose, (2) know with substantial certainty, or (3) transferred
intent. Again, there are only three sub-sub-parts, which is well within the “magical number
seven.”

(3) Create names or labels for categories and sub-categories. The names of the categories and
the links between categories should be in your own words so that the categories form a
continuous story in your mind.

Example: The first element of battery occurs in the defendant’s head. The other two describe
what the defendant’s body does. You could use this information to label the parts, to make the
categories more memorable to you.

Labels Elements Rules


D’s Head Intent (1) purpose to cause contact
(2) know with substantial
certainty contact will occur
(3) transferred intent
D’s Body Contact (1) body to body
(2) touching something
intimately connected
(3) set forces in motion to
cause contact at a distance
Harmful or Offensive (define)

How to find the categories: To help you form the categories or chunks, you can use the table of
contents and the syllabus from the course. Supplements also help. Establishing the chunks early
in the term will enable you to add the bits of information under each category as you study and
learn from class and outside sources. (That is why it helps to form the Big Picture as early as
possible in the term.)

Memorizing of the chunks: The items that you place first and last in each chunk are the easiest
to recall. Therefore, when you are trying to memorize a chunk, give the items in the middle of
the list some extra attention.

127
Summary

Of the three steps, the second – organization – is likely to be the most difficult. It is
unlikely that you had to do much of this kind of organization in college, so this process may be
new to you. Furthermore, many college students simply memorize without understanding what
they are memorizing. They don’t even exercise any critical thinking.

However, law school requires critical thinking. Its importance cannot be overstated. The
process of finding and sorting the parts is crucial to both memory and understanding. Knowing
what the elements are, knowing whether there are rules and where those rules fit, knowing what
kind of facts and reasoning go with each element – this kind of sorting into categories (chunking)
and understanding the relationships is required for success.

Finally, creating long-term memory is hard work for everyone. You must assess how
much work is necessary for you to build memory. Over-learning – continuing to review past the
point where you are sure that you know the material – is a good idea.

______________________________________

128
Week Four:

Organizing Information and Outlining

129
130
Week 4 Pre-Reading Questions
(1) What is “concept development”? What strategies would help you to develop concepts?

(2) What are data-organization devices? What can you use them for?

(3) What are some types of data-organization devices?

(4) How do you choose which data-organization devices to use? What type of information is
each data-organization device best suited for?

(5) What is an outline? Why should you write an outline? What is it for?

(6) What types of information should be included in your outline?

(7) What is the “Big Picture”? Where can you find it?

(8) How do you select and organize information in an outline?

(9) What is the difference between an attack outline and a course outline?

131
132
Concept Development and Data-Organization Devices
Overview: The most important skill in law school is the ability to organize large quantities of
detailed information. Organizing the information is necessary both so that you can understand it
and so that you can put it into long-term memory. This section is intended to answer two
questions: (1) how to understand individual concepts and figure out how those concepts relate to
each other and (2) what kinds of data-organization devices you can use to make the concepts and
relationships clear.

I. Concept Development. Some students come to law school with a naïve perception of
what they must do to learn the law. Usually, this is because they have memorized
instead of learned. Truly learning and understanding legal concepts requires
understanding the ideas behind the words. It also requires understanding relationships
among ideas. Creating this understanding within yourself requires a lot of effort –
these concepts do not teach themselves. But this effort will pay off. Once you have a
deep understanding of the concepts, you will be able to spot issues and analyze them.
Without this understanding, issue spotting and analysis are impossible.

How to develop a concept: Developing a deep understanding of a concept requires you to ask
yourself questions about the concept. Here is a list of important questions.

Question Explanation
(1) What is the name of the concept? Identify the term of art or legal name by
which the concept is known.
(2) What is the definition of the concept? Define the term. Definitions can be found in
the casebooks, in hornbooks, in Nutshells, or in
other supplemental sources; sometimes,
definitions are given in class.
(3) What are the critical attributes of the Think about the words in the definition and
concept? ask what ideas are described by what
definition. Translate those ideas into your
own words. The point is to ask what the
essential features of the concept are.
(4) What is the best example of the concept? Think about the cases and other examples you
have come across (in classroom hypos, for
example). Which example most clearly fits the
classification given in the definition?
(5) What are some other examples of the Ask what other examples also fit the
concept? classification given in the definition.
(6) What would be something that is not an Think about what examples you have seen that
example of the concept? do not fit the classifications given in the
definition.

133
When you encounter two or more concepts that are closely related, ask yourself these additional
questions.

Question Explanation
(7) What is the difference between these Think about or write down a list of the critical
concepts? attributes of each concept. Put the lists side by
side. Ask what is different in those lists.
(8) How are these concepts related? Ask what the concepts have in common, how
you know which one you’re dealing with, and
what a lawyer could use them for.

Example 1: Examine the contract concept of “offer.”

1. Name? Offer

2. Definition? One hornbook definition is “a manifestation of a


willingness to enter into a bargain so made as to justify
another person in understanding that assent to that bargain
is invited and will conclude it.”

3. Critical attributes? There are three major ideas. The first two are about the
offeror and the third is about the offeree.
(1) The offeror has to do or say something
(manifest) to show that he is ready to be in some kind of
deal with the offeree, and
(2) what the offeror did or said has to make it reasonable
for the offeree to think that the offeror wants him to agree
and, if he does, they will have a deal, and
(3) the offeree has to understand the offer.

4. Best example? “I offer to sell you my watch for $500.”

5. Other examples? “If you will sell me your car, I will pay you
$5,000.” “We will pay $100 to anyone who gets the flu
after using our new medicine.”

6. Non-examples? “I’m hoping to sell my car for $450.” (Statement of


intention.) “Will you sell me your mower for $100?”
(Inquiry.) “I couldn’t sell it for less than $200.” (Invitation
to make an offer.)

134
7. What is the difference between these concepts?

Offer Statement of Inquiry Invitation to make


intention an offer
The offeror says or What the “offeror” What the “offeror” What the “offeror”
does something to says does not give the says does not give the says does not give the
show he is willing to “offeree” reason to “offeree” reason to “offeree” reason to
make a deal right believe that the believe that the believe that the
now, and the offeree “offeror” wants to “offeror” wants to “offeror” wants to
has reason to believe make a deal now (no make a deal now (no make a deal now (no
that all he has to do is commitment). commitment). commitment).
agree and they will Instead the “offeror” Instead, the “offeror” Instead, the “offeror”
have a deal. is saying what he is asking for is asking the
wants to do in the information. “offeree” to suggest
future. terms.

8. How are these concepts related? At first glance, they might all look like offers. But
an offer can be accepted. The other three cannot.

Example 2: Examine the tort concept of “battery.”

1. Name? Battery

2. Definition? The Nutshell definition is “an intentional and


unpermitted harmful or offensive physical contact with the
plaintiff’s person by defendant or by an agency defendant
has set in motion.”

3. Critical attributes? There are four major ideas.

(1) intent to cause contact (“intentional”)


(2) P didn’t give permission (and nothing made it okay for
D to do it) (“unpermitted”). (Note: some sources seem to
treat “unpermitted” as an element of battery. Others may
not list it as an element; instead they seem to treat the
presence of permission as a defense. The difference is who
has the burden of proof. The outcome could be the same
either way because when there is permission, P cannot
win.)
(3) physically hurtful, or hostile or insulting (“harmful or
offensive”)
(4) D touched P directly or D made something touch P
(“physical contact with the plaintiff’s person by defendant
or by an agency a defendant has set in motion”).

4. Best example? On purpose, D punches P in the nose.


135
5. Other examples? D throws a stick and hits P; D grabs a plate out of
P’s hands; D pulls a chair out and P lands on the patio.

6. Non-examples? D accidentally brushes up against P (no intent); D


asks to kiss P, P says “okay,” and D kisses P (not
unpermitted); D swings at P and misses (no contact).

To make sure that you understand the concept of battery, you will want to contrast it with other,
similar concepts.

7. What is the difference between assault and battery? A hornbook definition of assault
in tort is “an act that is intended to and does place the plaintiff in apprehension of an
immediate unconsented-to touching that would amount to a battery.”

Battery Assault
Intent to put the other in apprehension of a
Intent to cause contact
contact.
Unpermitted Unpermitted
Apprehension of immediate contact occurs, but
Contact occurs no contact needs to actually occur: P just needs
to be aware that it’s coming.
Contact is harmful or offensive No contact is necessary
P doesn’t have to be aware of the contact at the
P has to be aware that a contact is imminent.
time.

Thus, several things are different: (1) though both require intent, the intent required is different;
(2) battery requires a touching, but assault does not; and (3) battery doesn’t require that P be
aware of what’s going on, but assault does.

8. How are these concepts related? Both are intentional torts against people, but
the actions required to prove the tort will look
different. Swinging a bat at someone’s head is an
assault (if the person knows what’s happening). But
as soon as the bat touches the head, it’s battery,
whether the person knew about it or not.

Summary of concept development: The key to understanding the concepts is to ask (and
answer) the right questions. The questions given above will help you understand what to call the
concept, what it means, what the characteristics are, what will and will not be examples of the
concept and how the concept is different from or similar to other, related concepts. A deep
understanding of the concepts will enable you to spot issues in essay questions, in multiple-
choice questions, and in the practice of law.

136
II. Data-Organization devices. Using data-organization devices helps people to
visualize abstract concepts. You can use data-organization devices to help you (1)
understand relationships among ideas and (2) record the ideas in a way that makes
those relationships visible. Thus, to use data-organization devices effectively, you
must solve two problems: (1) you must be able to figure out what those relationships
are and (2) you must understand how each data-organization device functions so that
you can choose the one that is most appropriate.

The section on concept development, above, is intended to help you solve the first
problem: what the relationships are. Furthermore, you will probably find that
choosing and using data-organizations devices clarifies those relationships. The
remainder of this section is intended to help you solve the second problem: deciding
which data-organization device is most appropriate.

Over the years, you may have become familiar with many devices for organizing
information (data). Each device is good for some things and not good for other things.
After you understand what relationship you want to illustrate among the ideas, choose
the device that “fits” what you are trying to do. Here is a summary of some devices
and how they are used.

(1) Numbered lists. Numbered lists are used to enumerate items that are of equal weight or have
the same logical relationship to each other.

Example: Elements of battery


1. Intent
2. Contact
3. Harmful or offensive

A numbered list works here because all three items are of equal weight and have some logical
relationship to the concept of battery: all three must be proved for defendant to be liable for
battery. All three are equally necessary.

A numbered list will not be effective when the items are not of equal weight or do not have the
same logical relationship.
Example: Elements of battery
1. Intent to cause contact
2. Purpose to cause contact
3. Know with substantial certainty that contact will occur
4. Transferred intent
5. Contact
6. Harmful or offensive

In this example, a list does not work. The list structure implies that all six items have the same
logical relationship to the concept of battery, but they do not. Intent, contact, and harmful or
offensive are all elements that must be proved for defendant to be liable for battery. The other
three ideas (purpose, know with substantial certainty, and transferred intent) are alternative ways
137
of proving one of the elements – intent. All three are not necessary; instead, only one of the three
has to be present to prove the element of intent. So using a list structure implies a relationship
between the items that is inaccurate and misleading.

(2) Outlines. An outline structure is used to show the relationship between broader concepts and
subdivisions of the broader concepts. Headings contain the major concepts; subheadings show
the parts of those major concepts. An outline is useful to show which ideas are subordinate to or
form the parts of the major concepts.

Example: A. Intentional Torts


1. Battery
a. Intent to cause contact
i. Purpose to cause contact
ii. Know w/substantial certainty that
contact will occur
iii. Transferred intent
b. Contact
c. Harmful or offensive
2. Assault
3. False Imprisonment
4. Intentional infliction of mental distress
5. Trespass to land
6. Trespass to chattels
7. Conversion
B. Negligence
1. Duty
2. Breach
3. Causation
4. Harm
C. Strict Liability

In this example, the outline structure shows three major concepts from the
Torts table of contents; it also shows the seven intentional torts that form
the parts of the major concept of “Intentional Torts.” The three elements
of battery are underneath the sub-concept of battery. And the three ways
of proving intent are underneath the sub-concept of intent. This outline
structure accurately portrays the logical relationships among the different
ideas.

138
(3) Tree diagrams. Tree diagrams work in much the same way as outlines, but they demonstrate
the relationship between major concepts and sub-concepts pictorially instead of through text. The
branching usually runs downward so that one encounters smaller and smaller subdivisions as one
moves down the page. (These trees grow upside down!)

Example: Torts

Intentional Torts Strict Liability

Battery Negligence
Intent
Contact

Harmful or
Offensive

This tree diagram portrays the same logical relationships between the concepts
and sub-concepts as the outline above.

(4) T-charts. T-charts are used to compare ideas. They are also useful for sorting information
sideways: that is, they show which ideas “fit” with other ideas. So the T-chart tells your mind to
compare the information in two directions: vertically and horizontally. T-charts can have as
many columns as you need.

Example: If you have to make an important decision, you may put the pros and
cons of the decision in a T-chart.

Pros (+) Cons (-)

Using the T-chart in this way may help you to compare the advantages of a certain decision with
the disadvantages.
Example: One excellent use of the T-chart is to see how the parts of a case
relate to each other.

Reasoning Law Facts

Reasoning that goes with


Element 1 Facts that go with element 1
element 1

139
Reasoning that goes with
Element 2 Facts that go with element 2
element 2

Reasoning that goes with


Element 3 Facts that go with element 3
element 3

Using the T-chart in this way may help you see which ideas cluster around each idea found in the
center column.

(5) Feature analysis charts. Feature analysis charts are very similar to T-charts. They are also
useful for comparing ideas to see in what ways they are similar and how they differ.

Example: In Criminal Law, you learn that there are many different
varieties of homicide. Some of the elements are similar, but others are different.
You could construct a feature analysis chart to see which elements are in all the
types, which are in some of the types, and which are in only one of the types.

Element 1 Element 2 Element . . . Last Element


“death has .... .... “during the
occurred” commission of
a felony”
Willful, deliberate, yes (fill these rows and columns in no
premeditated murder yourself)
Intent-to-do-great-bodily- yes no
harm murder
Depraved-heart murder yes no
Felony murder yes yes

Using a chart may help you to see exactly what elements these have in common
and how they differ. Notice that the feature analysis chart is perfect for step 7 of
concept development, where you asked, “What is the difference between these
concepts?”

140
Torts Against People Torts Against Property

Battery Assault False IIED Trespass to Trespass to Conversion


Imprisonmen Land Chattels
Intent: Purpose or know Purpose or know Purpose or know Purpose or know Purpose or know Purpose or know Purpose or know
with substantial with substantial with substantial with substantial with substantial with substantial with substantial
what’s in certainty certainty certainty certainty certainty certainty certainty
Ds head ------------ ---------- ----------- ---------- ----------- ---------- ----------
Result desired Result desired Result desired Result desired Result desired Result desired Result desired
or known: or known: or known: or known: or known: or known: or known:
To cause a harm- To cause the
ful or offensive other to
contact with the apprehend an
person imminent battery

Transferred intent

person to person
or tort to tort.

141
Result: • Bodily contact • Unlawful
(Direct or touch
what D’s something • Well-founded
body did intimately apprehension
connected with (If you don’t
the person) know then no
apprehension)
• Harmful or
offensive • Offer to touch
(Words alone are
not enough)

• Rude or angry

• Apparent ability
(don’t need
actual ability)

• Don’t have to
prove actual
harm

Note that the above chart is incomplete. You can fill in the rest yourself. The chart works well for other areas of the law where concepts are similar
in some ways and different in others. For example, in criminal law, it works for homicide, criminal sexual conduct, and theft crimes.
Common Law Modern Statutory Model Penal Code
Law
Murder (Put the different (Put the different (Put the different
types of common law types of modern types of Model Penal
murder, their elements statutory murder, their Code murder, their
and definitions, and elements and elements and
examples here) definitions, and definitions, and
examples here) examples here)
Manslaughter (Put the different (Put the different (Put the different
types of common law types of modern types of Model Penal
manslaughter, their statutory Code manslaughter,
elements and manslaughter, their their elements and
definitions, and elements and definitions, and
examples here) definitions, and examples here)
examples here)

Criminal Sexual CSC1 CSC3 CSC2 CSC4


Conduct
Sexual penetration (definition) Sexual contact (definition)
(Put the rest of (Put the rest of (Put the rest of (Put the rest of
the elements and the elements and the elements and the elements and
aggravating aggravating aggravating aggravating
factors here) factors here) factors here) factors here)

Theft Crimes False Pretenses Larceny Embezzlement (What other


theft crimes are
there? Add
columns for
them, too.)
Elements (Write the (Write the (Write the (Write the
elements, elements, elements, elements,
definitions, and definitions, and definitions, and definitions, and
examples here) examples here) examples here) examples here)

142
Example: In Property, you must learn estates in land and future interests.
A chart can be used to sort information “sideways” so that you can see the
characteristics of each estate in land. You can also use it to compare the
different estates to see the similarities and differences.

Name Words used Future If future Notes What can A How long
to create the interest interest given do with his will A’s
estate retained by O to 3rd party estate? estate last?
(buzzwords) (if O doesn’t grantee
give his entire
estate away)
Fee Simple “To A and his None (O has None (there is This is the Use it, abuse Forever (it
Absolute heirs.” “To all the no 3rd party largest estate it, have naturally
A,” “To A in property grantee) in land exclusive terminates
fee simple,” rights) possible. possession, when the
or “To A to take its fruits, owner dies
be used for and dispose without heirs)
church of it by deed
purposes.” or will
Fee Simple “To A while Possibility of None Only applies Same as fee Potentially
Determinable …,” “To A Reverter (not when O simple forever, but it
with during …,” a reversion, doesn’t absolute extinguishes
Possibility of “To A because specify that automatically
Reverter until…,” “To determinable the land will (naturally) the
A for as long estate is not go to moment the
as ….” legally less someone named event
than a fee other than O. occurs.
simple)
Fee Simple Same as None T has Only applies Same as fee Potentially
Determinable above: for Executory when O does simple forever, but if
Subject to example, “To Interest (not specify that absolute it
Executory A while the remainder the land will extinguishes
Interest land is used because go to automatically
as a church, remainder someone (naturally) the
and when it is can’t follow other than O. moment the
not, to A and fee simple) named event
her heirs.” occurs.

And so on.

(6) Maps. Of course, maps can be used to show where certain geographical features are situated
in relation to each other. However, they can also be used to show where certain events occurred
when there has been a sequence of events.

Example: A case or hypothetical might state the following: “O owned


Blackacre, a 40-acre parcel bound on the north by a state highway and on
the west by a narrow, unimproved road. The east and south abut a state
park, consisting of a large forest. Midway through Blackacre and
equidistant from the east-west borders is a fence running from north to
south. O has a house in the northwest corner.” Some people have no

143
problem visualizing the situation described; others find that creating a map
makes everything much clearer.

State highway

House

Fence

State Park
Narrow Road

Blackacre

Example: Maps can show a sequence of events. Imagine that a case tells you the following: “P
was walking north along Main Street. At the intersection of Main and Elm Streets, P stopped and
waited for the ‘walk’ sign to come on. As P waited, D was driving westbound on Elm Street. The
Smiths were proceeding southbound on Main. D failed to stop for the red light and struck the
Smiths’ car as it proceeded through the intersection. A piece of debris from the collision flew
through the air and struck P.”

Smiths

D Elm St.

P
Main
St.

144
(7) Flowcharts. Flowcharts are used to show the decision steps that you may have to go through
in an analysis. Flowcharts are very useful for showing analyses requiring multiple steps.

Example: In Contracts, a rejection usually terminates the power to accept;


however, there are exceptions.

(8) Time Lines. Time lines are used to show a sequence of events. They may or may not
include dates. Time lines are particularly useful in procedure classes like Civil Procedure (a
second-term class) to show when particular rules come into play.

Pre-lawsuit events During lawsuit Post-lawsuit events

Complaint Trial
filed

145
(9) Other. You may be familiar with “other” data-organization devices. These are less likely to
be useful in law school.

Venn Diagram

(10) Composite. It is possible to combine these different data-organization devices to create new
ones. If you need to organize data in a particular way, but none of these quite fit, you may have
to take elements from more than one and combine them.

(11) Create your own. Use your imagination. Ask yourself, “How do these ideas fit together?”
Then experiment. Try to think of how to record that information so that the relationship between
the ideas is apparent.

Example: Imagine that you are studying battery, and you are trying to
think of all the different ways that you’ve heard of that a contact can
occur. You could make up a picture to demonstrate that.

Force P off the road into a Throw a stick and hit P in the
ditch Pull a chair out so P falls eye

Contact

Punch P in the nose Grab a plate out of P’s hand

146
Summary

There are three basic steps to choosing the right data-organization device.

1. Choose the concepts you want to include.


2. Identify the relationships among the concepts.
3. Choose the data-organization device that reflects the correct relationship.

The process of organizing information helps you to understand it and remember it. Using
the data-organization devices effectively will make understanding and remembering ideas much
more efficient.

________________________________________

147
148
Creating an Outline

What is an outline? An outline is a tool that you create to organize information. When it is
finished, it is a document that contains (1) all the important rules, elements, policies, etc., that
you learned in the class and (2) examples of what will, or will not, fulfill the requirements of the
law. However, its main value is that all of that information has been organized so that you can
see exactly how it all fits together. This will enable you to remember all of the information, and
it will create a template for how to write an essay answer or how to think through a multiple-
choice question.

Why write an outline? To write a good outline, you must select the information that is
important enough to put into your long-term memory. Then you must organize that information
so that you can see the major concepts; what details make up the sub-concepts that are part of
those major concepts; and what details make up the sub-concepts. In performing these tasks, you
force yourself to deep-process the information. So when you create an outline, you do not
simply memorize or record words – you create deep, true knowledge and understanding about
the subjects you study in law school. In other words, it is not the product – the finished outline
itself – that is valuable so much as the process that you will go through to create the outline.

Procedure:
1. Create the Big Picture. Use the table of contents or course syllabus (or both) to get a
list of the major concepts and sub-concepts taught in that course. Create a Big Picture
that shows how the concepts and sub-concepts fit together. If you like pictures, you
might draw a tree diagram, like this:

Torts

Intentional Torts Strict Liability


Assault

False Imprisonment

T to Chattels
Battery

T to Land

Conversion
IIMD
7 intentional
9 privileges,
(write in the

wrote in the

Negligence
just as you
Privileges

torts)

If you don’t favor pictures, you may want to use a traditional outline format, like this:
149
I. Intentional torts
1. Battery
2. Assault
3. False Imprisonment
4. IIMD
5. Trespass to Land
6. Trespass to Chattels
7. Conversion

1. Privileges (Defenses to Intentional Torts)


1. Consent
2. Self-Defense
3. Defense of Others
4. Defense of Property
5. Recovery of Property
6. Necessity
7. Authority of Law
8. Discipline
9. Justification

II Negligence

………………..

III. Strict Liability

Et cetera

2. Select the important information. As you go through your case briefs, class notes, etc.,
decide what information to select. You will definitely want to remember all of the law,
so every time you find an element or rule, plan to put it into your outline. If your
professor discusses policy in class, you probably want to put that into your outline. And
so on.

The task here is to decide what is important enough that it needs to go into your long-
term memory. This is a judgment call, and everyone does it a little differently. Some

150
people tend to include a bit more detail and some include a bit less, which is fine. But be
careful to avoid extremes!

If your personality is . . .

Very detail-oriented type Very bottom-line, cut-to-the-chase,

The happy medium


just-give-me-the-answer type

Your outline may contain too much, be Your outline may omit too many
too long, and take too long to write details, resulting in only superficial
treatment of the material.

You should aim for the happy medium!

3. Sort the information according to topic. For every piece of information you select, ask
yourself what major topic or sub-topic that information goes with. Is it an element of
trespass to chattels? Then it goes under “Trespass to Chattels” in the Big Picture. Is it a
rule for how the “contact” element of battery can be proved? Then it goes under
“Battery” in the Big Picture. Is it the policy reason for why actual injury is usually not
required in a trespass-to-land-case? Then it goes under “Trespass to Land” in the Big
Picture.

4. Organize the information under each topic. Once you have gathered all the
information about a particular topic or sub-topic and you have put all that information
under that topic, organize it. Ask yourself how it all fits together. Here is an example.
Let’s say that you want to outline Battery. You have gone through the cases, the
notecases, and your class notes and have gathered the following pieces of information
that pertain to Battery.

No actual harm to the person is required.


Intent
Contact with the person of another
Purpose to cause contact
Battery is a dignitary tort, protecting against harm to a person’s dignity
Direct contact is sufficient
Know with substantial certainly that the contact will result
Harmful
Putting something in motion that causes contact
Battery is a people tort – can’t batter an animal
Indirect contact by touching something that is intimately connected to the person
Transferred intent applies to battery
Person can recover damages from mental suffering resulting from battery
Offensive to a reasonable person
Two types of transferred intent – (1) person to person and (2) tort to tort
151
As you read this list, you should see that certain ideas must be grouped together because
they pertain to the same idea. For example, “purpose to cause contact,” “know with substantial
certainty that contact will result,” and “transferred intent” all have to do with the same element
of battery – intent.

Before Organizing After Organizing

No actual harm to the person is required. Intent


Purpose to cause contact
Intent
Know with substantial certainty that
Contact with the person of another the contact will result

Purpose to cause contact Transferred intent

Battery is a dignitary tort, protecting against Two types of transferred intent


harm to a person’s dignity (1) Person to person
(2) Tort to tort
Direct contact is sufficient
Contact with the person of another
Know with substantial certainty that the (Battery is a people tort – can’t batter
contact will result an animal – contact must be with a person)

Harmful (1) Direct contact

Putting something in motion that causes (2) Indirect contact by touching


contact something that is intimately connected to the
person
Battery is a people tort – can’t batter an
animal (3) Putting something in motion that
causes contact
Indirect contact by touching something that is
intimately connected to the person Nature of contact

Transferred intent applies to battery Harmful

Person can recover damages for mental Offensive to a reasonable person


suffering resulting from battery
No actual harm to the person is required
Offensive to a reasonable person because battery is a dignitary tort, protecting
against harm to a person’s dignity. (That’s
Two types of transferred intent – (1) person to why a person can recover damages for mental
person and (2) tort to tort suffering resulting from battery.)

152
5. Add examples, explanations, questions, cues, and triggers. Now that the pieces are
sorted out, you can put finishing touches on. Add examples of the concepts from the
cases, the notecases, and your class notes. Add explanations of how the parts fit together.
Add questions and cues that will help you recite and review the material. Add the trigger
for that concept – a short description of what kind of situation you need to see in a fact
pattern before you need to discuss that legal topic.

Law on intentional torts Cases, Reasoning, and Examples


Battery (What is battery?) Trigger for battery: P has to be touched,
and there has to be some reason to
Battery is a people tort – can’t batter an believe that the touching could be on
animal purpose
3 elements
(1) Intent (how can intent be proved?)
This is Three ways to prove intent:
what must • Purpose to cause contact Pulling the chair out while lady sat down
be in D’s would show purpose
head • Know with substantial certainty that Moving the chair knowing she wouldn’t
contact will result notice in time would be substantial
certainty. Hypo: throwing softballs at
full speed into crowded classroom =
substantial certainty: you know you
won’t miss forever

• Transferred intent: 2 types


o Person to person (intending to Throwing stick at one kid and hitting
batter one person, but hit another another = person-to-person transferred
instead) intent
o Tort to tort (intending to commit Throwing a punch to scare someone, but
another tort [battery, false hitting the person = tort-to-tort
imprisonment, trespass to land, transferred intent (intent to assault
or trespass to chattels] and end transfers to battery)
up causing harmful or offensive
contact on P
This is (2) Contact with the person of another (what
is contact?)
what D’s Three ways to prove contact:
body must • Direct contact Hit nurse in head
do • Indirect contact by touching Grab plate from hand is sufficient
something that is intimately because plate is connected to hand
connected to the person
• Putting something in motion that Moving the chair and lady falls – put the
causes a contact force in motion
(3) Nature of contact (what kind of
contact?)
Two ways to prove:
• Harmful Broken hip is harmful
• Offensive to a reasonable person Grabbing plate and racial slur
Policy: No actual harm to the person is Cole v. Turner, the least touching is
153
required (why?) because battery is a sufficient
dignitary tort, protecting against harm to a
person’s dignity (that’s why a person can The essence of battery is personal
recover damages for mental suffering). indignity

154
If you prefer a traditional outline format instead of the chart form, go ahead. The Battery outline
might look something like this in the traditional outline format.

Battery (what is battery?)


Trigger for battery: P has to be touched, and there has to be some reason to believe that the
touching could be on purpose
3 elements (what are the elements?)
(1) Intent (how can intent be proved?)
3 ways to prove intent:
(a) purpose to cause contact
This is pulling the chair out from under lady to make her fall
would show purpose
what
(b) know with substantial certainty that contact will result
must be moving the chair knowing that the lady wouldn’t notice
in D’s in time would be substantial certainty
head Hypo: throwing softballs at full speed into crowded
classroom = substantial certainty: you know you won’t
miss forever
(c) transferred intent
Two types of transferred intent:
Person to person (intending to batter one person
but hit another instead)
Threw stick at one kid and hit another,
was sufficient for transferred intent
Tort to tort (intending to commit another tort
[battery, assault, false imprisonment, trespass to
land, or trespass to chattels] and end up causing
harmful or offensive contact on P
Can only transfer intent when both torts
are on this list
This is (2) Contact with the person of another (what is contact?)
what (Battery is a people tort – can’t batter an animal)
3 ways to prove contact:
D’s
(1) direct contact
body is hitting nurse in head
doing (2) indirect contact by touching something that is intimately
connected to the person
grabbing plate from hand is sufficient (plate is
connected to the person)
(3) putting something in motion that caused a contact
moving the chair and causing the lady to fall – put the
force in motion
(3) Nature of contact (what kind of contact?)
2 ways to prove:
(1) harmful

155
broken hip is harmful
(2) offensive to a reasonable person
grabbing plate and racial slur is rude and humiliating
No actual harm to the person is required (why?)
Cole v. Turner: the least touching is sufficient because battery is a dignitary
tort, protecting against harm to a person’s dignity
Fisher: essence of battery is personal indignity
(That’s why a person can recover damages for mental suffering
resulting from battery without actual physical injury

6. Use the outline to recite and review. Use the questions and cues to recite, out loud, the
information you have selected and organized. Review the parts and the Big Picture. Do
this frequently.

7. At the end of the term, create an attack outline. A week or two before exams, create a
very short outline for each class. This outline should usually be 10 pages or less and
contain the most crucial concepts from that class. Recite it over and over until you can
do it automatically, without thinking. See whether you can recall the details from your
big outline that go with each point in the attack outline.

(Note: The above Battery outline does not contain all the information
that it should. The notecases contain more information and examples
that do not appear in this outline, and you will receive even more
information and examples [hypotheticals] in class. This outline is to
give you the idea of how to create an outline. Its purpose is not to give
you a completed outline. )

156
Week Five:

Code

157
158
Week 5 Pre-Reading Questions
(1) What is code? What are examples of code?

(2) What are the steps to follow when breaking down code?

(3) Why break down code?

159
160
Reading Code

What is code? Code is pure law. Cases are not code: they may contain code, but they also
contain many other things, such as facts, procedure, issue statements, reasoning, and conclusions.
Code, on the other hand, is made up of sentences and paragraphs of law, exclusively.

Objective: to learn how to break down the law from statutes, Restatements, the Model Penal
Code, the UCC, the Constitution, etc., and to understand the parts and how the parts relate, so
that you can remember all the parts more easily and apply them effectively.

Steps: Ask yourself the following questions.

1. How many different ideas are contained in this explanation of the law?

2. How do these ideas fit together? Pay close attention to logical connectors like
“and,” “or,” “unless,” “but,” etc. “And” means that all the items in the list
must be proved. “Or” means that one item in the list must be proved, but it can be
proved in different ways. “Unless” signals an exception. And so on.

3. What does each idea mean?


(A) Can you translate each idea into your own words?
(B) Can you find a definition?

4. What would be an example of each idea? What would be a non-example of each


idea?

Look at the following Restatement section. Ask yourself whether it would be easy to
remember and apply it in its current form.

Example: Restatement section 69 – Acceptance by Silence or Exercise of Dominion

1) Where an offeree fails to reply to an offer, his silence and inaction operate as an acceptance
in the following cases only:

a) Where an offeree takes the benefit of offered services with reasonable opportunity to
reject them and reason to know that they were offered with the expectation of
compensation.

b) Where the offeror has stated or given the offeree reason to understand that assent may
be manifested by silence or inaction and the offeree in remaining silent and inactive
intends to accept the offer

c) Where because of previous dealings or otherwise, it is reasonable that the offeree


161
should notify the offeror if he does not intend to accept.

2) An offeree who does any act inconsistent with the offeror’s ownership of offered property is
bound in accordance with the offered terms unless they are manifestly unreasonable. . . .

Let’s work with the first two paragraphs of this Restatement.

1. How many different ideas are contained in this explanation of the law? Break it into
pieces. Make each idea a separate piece. The fastest, easiest way to do this is to put
slashes between the pieces.

1) Where an offeree fails to reply to an offer, / his silence / and inaction operate as an
acceptance in the following cases only:

a. Where an offeree takes the benefit of offered services / with reasonable


opportunity to reject them / and reason to know that they were offered with
the expectation of compensation.

2. How do these ideas fit together? Find the IFs, THENs, and UNLESSes.

Non-legal Example of IFs, THENs, and UNLESSes: Bedtime rules

IF your child is under 9 years old

THEN bedtime is 8:00

UNLESS it is a weekend or a holiday or a birthday

In the “IF” section, you find the elements: that is, what must be true for the rule to apply.

In the “THEN” section, you find the outcome: that is, what will result if the elements in the “IF”
section are true.

In the “UNLESS” section, you find a new set of elements. If these elements are true, then the
original outcome, the THEN, changes to some new outcome. In this case, the bedtime might
change from 8:00 to 10:00 or 11:00.

162
In our Restatement sections, ask which parts are IFs: that is, ask what must be true before the
rule can apply. Then ask which parts are THENs: that is, ask what the outcome will be if the rule
applies.

IF an offeree fails to reply to an offer (silence, inaction) and


IF the offeree takes the benefit of offered services and
IF the offeree has reasonable opportunity to reject them and
IF the offeree has reason to know that they were offered with the expectation of compensation

THEN his silence and inaction operate as an acceptance.

Let’s do it again, this time with the last three paragraphs of the Restatement.

a) Where the offeror has stated or given the offeree reason to understand that assent may
be manifested by silence or inaction / and the offeree in remaining silent and inactive
intends to accept the offer

b) Where because of previous dealings or otherwise, / it is reasonable that the offeree


should notify the offeror if he does not intend to accept.

2) An offeree who does any act inconsistent with the offeror’s ownership of offered
property / is bound in accordance with the offered terms / unless they are manifestly
unreasonable.

Now let’s look for the Ifs, THENs, and UNLESSes.

IF the offeror has communicated that assent may be manifested by silence or inaction and
IF the offeree in remaining silent and inactive intends to accept the offer

THEN his silence and inaction operate as an acceptance.

____________________________________

IF there have been previous dealings or otherwise and


IF the offeree does not intent to accept, it is reasonable that the offeree should notify the offeror

THEN his silence and inaction operate as an acceptance.

163
IF an offeree does any act inconsistent with the offeror’s ownership of offered property

THEN the offeree is bound in accordance with the offered terms

UNLESS they are manifestly unreasonable.

3. What does each idea mean? Paraphrase, or look for definitions if there are any. Law
is often written in stilted, pompous language. You can check your understanding by
putting the ideas into your own words. Also, if it looks as though there is more than one
idea on a line, split it into two or more. Let’s put all five paragraphs together.

IF the offeree says nothing and does nothing and


IF the offeror has offered something of value and
IF the offeree takes it and
IF the offeree was able to reject without much trouble or expense and
IF the offeree has reason to believe that the offeror expected to be paid

THEN the offeree has accepted the offer and must pay.

____________________________________

IF the offeror has communicated that the offeree can accept by saying nothing and doing nothing
and
IF the offeree says nothing and does nothing because he intends to accept the offer

THEN the offeree has accepted the offer and must pay.

____________________________________

IF the parties have dealt with each other before or


IF there is some other reason the offeree has reason to know that, if the offeree does not intend to
accept, it is reasonable that the offeree should notify the offeror and
IF the offeree does not notify the offeror that he does not want to accept

THEN the offeree has accepted the offer and must pay.

____________________________________

IF the offeror has offered property to the offeree and


IF the offeree treats the property as though it is his own

164
THEN the offeree has accepted the offeror’s terms

UNLESS the terms are completely unreasonable; THEN the offeree has not accepted those
terms.

Now you can create categories within the text by labeling the major parts. At this
point, it’s easy to see that these parts may cover different situations. Look at each one:
what purpose does each part serve? Label each part with a word or phrase that explains
its purpose.

“If I do this, I expect


you to pay.” IF the offeree says nothing and does nothing and
IF the offeror has offered something of value and
IF the offeree takes it and
IF the offeree was able to reject without much trouble or expense and
IF the offeree has reason to believe that the offeror expected to be paid

THEN the offeree has accepted the offer and must pay.

“If I don’t hear from


you, I’ll assume it’s IF the offeror has communicated that the offeree can accept by not
okay.” responding and
IF the offeree says nothing and does nothing because he intends to accept
the offer

THEN the offeree has accepted the offer and must pay.

“If you don’t want


to, let me know.” IF the parties have dealt with each other before or
IF there is some other reason the offeree has reason to know that, if the
offeree does not intend to accept, it is reasonable that the offeree should
notify the offeror and
IF the offeree does not notify the offeror that he does not want to accept

THEN the offeree has accepted the offer and must pay.

“You break it, you


buy it.” IF the offeror has offered property to the offeree and
IF the offeree treats the property as though it is his own

THEN the offeree has accepted the offeror’s terms

UNLESS the terms are completely unreasonable; THEN the offeree has
not accepted those terms.
165
4. What would be an example of each idea? What would be a non-example of each idea?
Let’s use the first section as an example.

Law Example Non-example


IF the offeree says nothing Offeree stands by and Offeree objects
and does nothing watches, saying nothing
IF the offeror has offered Offeror offers to paint Offeror dumps toxic waste in
something of value offeree’s house offeree’s pond
IF the offeree takes it Offeree’s house gets painted Offeror painted the wrong
house
IF the offeree was able to Offeree just had to say Offeree is in France, doesn’t
reject without much trouble or something speak French, and doesn’t
expense know the offeror’s number
IF the offeree has reason to Offeror tells offeree that he Offeror and offeree have never
believe that the offeror expects payment communicated
expected to be paid

Why go to all this trouble?


At this stage, we have accomplished two things. First, because we have organized the
information into checklists and developed the meaning of each part, the ideas inside the code are
much more memorable. Second, we have a template for answering an essay or multiple-choice
question.

Example: Short-answer essay question

Homeowner lives in Michigan, but will be vacationing in France for the next
three weeks. Painter sends Homeowner a letter that says that Homeowner’s
house could use a paint job. Painter offers to paint the house, but if
Homeowner doesn’t want it done, Homeowner should let Painter know
within the next week; otherwise, Painter will paint the house and charge
$1500. Homeowner takes the letter out of the mailbox, shoves it into his
pocket, and does not read it until the plane touches down in France.
Homeowner does not respond to the letter. Painter paints the house. How
can Homeowner argue that he has not accepted by silence?

Now we have a template for thinking through the question:

166
Elements Facts Element met?
IF the offeree says nothing and does Homeowner did not respond to the Yes
nothing letter

IF the offeror has offered something of A paint job is worth something Yes
value

IF the offeree takes it Homeowner’s house is now painted Yes

IF the offeree was able to reject Homeowner is vacationing in France No


without much trouble or expense

IF the offeree has reason to believe Painter said he would charge $1500 Yes
that the offeror expected to be paid

It is now easy to see that all of the elements are met except one: whether the offeree could reject
without much trouble or expense. So that is the element that Homeowner will argue.

Here is one possible answer to the question:

The issue is how Homeowner could argue that he has not accepted by silence.
One of the elements of silence by acceptance is whether the offeree would be
able to reject without much trouble or expense. Here, Homeowner, who lives
in Michigan, is vacationing in France for three weeks. Homeowner can
argue that he would definitely have to go out of his way. It would take both
time and expense to turn down this unsolicited offer. Homeowner would
have to figure out Painter’s phone number, figure out the French phone
system, spend money to get in touch with Painter, and take time away from
his vacation to do all of this. It would be a pain, especially if he doesn’t speak
French. So Homeowner has a tenable argument.

167
§ 2-205. Firm Offers
An offer by a merchant to buy or sell goods in a signed writing which by its terms give
assurance that it will be held open is not revocable, for lack of consideration, during the time
stated or if no time is stated for a reasonable time, but in no event may such period of
irrevocability exceed three months; but any such term of assurance on a form supplied by the
offeree must be separately signed by the offeror.

Definitional Cross References: “Signed”. Section 1-201


“Goods”. Section 2-105 “Writing”. Section 1-201
“Merchant”. Section 2-104

§ 2-105. Definitions: Transferability; “Goods”; “Future” Goods; “Lot”; “Commercial


Unit”
(1) “Goods” means all things (including specially manufactured goods) which are
movable at the time of identification to the contract for sale other than the money in which the
price is to be paid, investment securities (Article 8) and things in action. “Goods” also includes
the unborn young of animals and growing crops and other identified things attached to realty as
described in the section on goods to be severed from realty (Section 2-107).

§ 2-104. Definitions: “Merchant”; “Between Merchants”; “Financing Agency”

(1) “Merchant” means a person who deals in goods of the kind or otherwise by his
occupation holds himself out as having knowledge or skill peculiar to the practices or goods
involved in the transaction or to whom such knowledge or skill may be attributed by his
employment of an agent or broker or other intermediary who by his occupation holds himself out
as having such knowledge or skill.

§ 2-104. General Definitions [Selections]


(39) “Signed” includes any symbol executed or adopted by a party with present intention
to authenticate a writing.

(46) “Written” or “writing” includes printing, typewriting or any other intentional


reduction to tangible form.

168
Sample Essay

Skunk Oil Company is a company that removes or severs oil from its real estate in Texas.
Skunk Oil sells to a variety of companies. Joe Skunk is the president of Skunk Oil.

On June 1st, 2009, Joe received a letter from Bird Oil Company, which was interested in
buying some of Skunk’s oil. In the Texas region, Bird Oil is known for its television
commercials, which depict a big green mythical bird that flies around its oil stations. This same
bird is on its letterhead and its envelopes. The letter to Skunk Oil was printed on this same
letterhead.

The letter said, “Bird Oil hereby offers to buy 100 barrels of oil for $30 per barrel. The
total contract price is $3000. This offer is firm and will not expire until November 1, 2009.
Please ship on that date.”

Joe placed the order in his “Orders-To-Be-Filled” file. On the morning of November 1,
he called his foreman and told him to load 100 barrels of oil to be shipped to the Bird Oil
Company. As he got off the phone with his foreman, he received a phone call from Bird Oil,
telling him that Bird Oil had decided that it did not need the oil after all.

Do Skunk Oil and Bird Oil have a contract for the 100 barrels of oil?

169
§ 2-207. Additional Terms in Acceptance or Confirmation.

1. A definite and seasonable expression of acceptance or a written confirmation which is


sent within a reasonable time operates as an acceptance even though it states terms
additional to or different from those offered or agreed upon, unless acceptance is
expressly made conditional on assent to the additional or different terms.

2. The additional terms are to be construed as proposals for addition to the contract.
Between merchants such terms become part of the contract unless:

a. the offer expressly limits acceptance to the terms of the offer;


b. they materially alter it; or
c. notification of objection to them has already been given or is given within a
reasonable time after notice of them is received.

3. Conduct by both parties which recognizes the existence of a contract is sufficient to


establish a contract for sale although the writings of the parties do not otherwise establish
a contract. In such case the terms of the particular contract consist of those terms on
which the writings of the parties agree, together with any supplementary terms
incorporated under any other provisions of this Act.

170
Problem 31
The purchasing agent of the Galsworthy Oil Company sent off the usual company purchase order
to the Forsythe Shipbuilding firm for the purchase of a $100,000 tugboat. It contained a clause
stating “Buyer objects in advance to any changes Seller attempts to make to the terms of this
purchase order.” On receiving this, the sales agent of Forsythe Shipbuilding sent off the usual
company sales confirmation slip, which included the following clause: “Seller does not warrant
its goods in any way, and specifically disclaims any warranty of MERCHANTABILITY or of
fitness. This form is not an ‘acceptance’ unless Buyer expressly agrees to all changes proposed
by Seller.”

Prior to the delivery date and to the start of preparations by either party, the market changed so
that Galsworthy Oil wanted to get out of the deal. You are their lawyer. Is there a contract?
What are its terms? If the parties had performed without any further discussion of their
differences, would the deal include a warranty? See UCC 2-207(3) and 2-314.

171
§ 2-314. Implied Warranty: Merchantability; Usage of Trade.
1. Unless excluded or modified (Section 2-316), a warranty that the goods shall be
merchantable is implied in a contract for their sale if the seller is a merchant with respect
to goods of that kind. Under this section the serving for value of food or drink to be
consumed either on the premises or elsewhere is a sale.
2. Goods to be merchantable must be at least such as
a. pass without objection in the trade under the contract description; and
b. in the case of fungible goods, are of fair average quality within the description;
and
c. are fit for the ordinary purposes for which such goods are used; and
d. run, within the variations permitted by the agreement, of even kind, quality and
quantity within each unit and among all units involved; and
e. are adequately contained, packaged, and labeled as the agreement may require;
and
f. conform to the promise or affirmations of fact made on the container or label if
any.
3. Unless excluded or modified (Section 2-316), other implied warranties may arise from
course of dealing or usage of trade.

172
Week Seven:

Reasoning

173
174
Week 7 Pre-Reading Questions
(1) What are reasoning statements?

(2) Why do they matter?

(3) What are some specific techniques for creating good reasoning statements?

(4) How can you evaluate whether a reasoning statement is “good”?

175
176
Creating Good Reasoning Statements

Objective: to learn to recognize and write good reasoning statements

Why the skill is important: On a law school essay exam, most of the points are earned in
the “reasoning” portion of the exam. This reflects the reality of being a lawyer: one of a
lawyer’s main jobs is to educate and persuade judges and juries. This means that the lawyer
must clearly explain not only what the facts and law are, but must also explain exactly why the
facts fit the law (if that is what the client’s case requires) or why the facts do not fit the law (if
that is what the client’s case requires). The objective is to make the reasoning so compelling that
the judge or jury believes there is no choice but to find for the lawyer’s client. This is done
through strong reasoning statements.

What reasoning statements are: Reasoning statements explain the connection between
the facts and the law. They explain what the law and facts have in common. When doing a legal
analysis, you are expected to write down the relevant piece of law, choose the facts that “fit” that
law, and then create reasoning statements that explain why those facts do, or do not, contain the
same ideas as the law requires. These reasoning statements explain why you are reaching the
conclusion that you are reaching.

Law
Facts

First piece of law Facts that “fit” first piece of law

Conclusion for first piece of law

Second piece of law Facts that “fit” second piece of law

Conclusion for second piece of law


177
And so on.
Types of reasoning statements: There are at least four types of reasoning statements:

(1) A statement that incorporates synonyms for (that is, words or phrases that mean
the same thing as) the elements or rules and that describes the facts.

Example:
Law: Assault: P must apprehend an imminent touching, which means that
P must recognize that the touching will occur without delay unless it is
prevented. (Dobbs, THE LAW OF TORTS, p. 34 (West, 2000)).

Facts: During an argument between D and P, D swings his fist at P’s face;
P ducks, and D loses his balance and misses P. P does not fear being hit
by D.

Reasoning: P was aware (synonym) that D’s fist was moving toward
(synonym) P’s face, so P “apprehended” the imminent touching.

(2) A statement that incorporates antonyms for (that is, words or phrases that mean
the opposite of) the elements or rules and that describes the facts.

Example (using the same law and facts as above):


Reasoning: P could not be unaware (antonym) of the fist moving
toward his face because he ducked to get out of the way (cause and effect).

(3) A statement that explains cause and effect in the facts (that is, explaining how
the facts cause the effect required by the elements or rules). To form a reasoning
statement using cause and effect, ask yourself what effect those facts would have
in our culture or what your experience with life tells you about why those facts are
important.

Example (using the same law and facts as above):


Reasoning: “Ducking” is an intentional movement to dodge a threat
(synonym), so P must have known what was happening or P wouldn’t
have been aware that P had to duck (cause and effect).

(4) A statement that clarifies what the rule requires or does not require.

Example (using the same law and facts as above):


Reasoning: It doesn’t matter that P did not fear being hit because the law
requires only that P was aware of the threat, not that P was afraid
(clarifies what the law does not require).

Notice that, sometimes, it is possible to combine techniques. Number 3, above, contains both
synonyms and cause-and-effect statements.
178
Exercise: Identifying Good and Bad Reasoning Statements

Objective: To learn to spot the patterns that identify good or bad reasoning statements.

Why it’s important: Some students may think that they are writing good reasoning
statements, but in fact they may not be. To be able to evaluate whether you are writing good
reasoning statements, you need to be able to spot the patterns of what makes them good or bad.

Bad versus good reasoning statements:

Bad reasoning statements simply repeat words from the law or repeat words from the
facts.

Good reasoning statements incorporate one or more of the techniques listed above.
Creating good reasoning statements means finding other words to explain why the ideas
in the law are also present (or not present) in the facts.

Examples:

(Non-legal example) You are a teenager. The house rules require


you to be home by 10:00 on school nights. You arrive home at
11:00 on a school night. Your car tire had gone flat, the jack didn't
work, and your cell phone wouldn't function. Your father confronts
you and says, "Why are you late?" You say:

Bad reasoning statement: "Because I'm late." (You're likely to


get in BIG trouble for this answer: because you’ve simply repeated
what he said to you, you’ll sound like a smart aleck.)

Good reasoning statement: "The car got a flat tire, so I couldn't


drive home until I changed it. But the jack broke, so I couldn't get
the car up to loosen the lugnuts. A Good Samaritan finally stopped
by and lent me his jack. I would have called, but my cell phone
battery was dead, so it didn't work. I'm really sorry." (Assuming
that all of this is true, you'll probably be OK: these reasoning
statements use cause and effect.)

(Legal example) The intent required for trespass to land is either


purpose to go on or send something onto the land or knowing with
substantial certainty that something will end up on the land; the
law does not require that D know that the land is not his, nor does
the law require that D intend to do harm. D is driving during a
Michigan winter and hits a patch of ice. The car goes out of
control, rolls, and ends up in P's living room.

179
Bad reasoning statement: D is not liable because D did not have
purpose to go onto the land nor knew with substantial certainty that
he would end up on the land. (This simply repeats the words from
the law.)

Bad reasoning statement: D is not liable because D's car hit a


patch of ice, went out of control, rolled, and ended up on P's land.
(This simply repeats words from the facts.)

Good reasoning statement: D is not liable because D had no plan


(synonym) to end up on P's land: D's plan was to stay on the road
(antonym), but the ice made it so that he couldn't make the car go
where he wanted it to (cause and effect). Nor did D have reason to
know that he would end up hitting P's house because he didn't
know that the ice was there (cause and effect).

Instructions: The following chart contains examples of law, facts, and sample reasoning
statements. Evaluate the reasoning statements, decide whether they are good or bad, and circle
the appropriate answer.

Law Facts Reasoning Good ☺ or


Bad

False imprisonment P is on her first (1) P is confined because she is a


requires that P be date with D. P passenger in the car and D is driving at (1) ☺
confined, which means decides that D is a 55 miles per hour.
that P is not permitted to jerk. D is driving
go beyond boundaries the car and P is a (2) P is confined because there is no
set by D. There is no passenger, so P reasonably safe and appropriate means (2) ☺
confinement if there is a tells D to stop and of egress.
reasonably knowable let her out. D
and reasonably safe and continues driving (3) P is confined because she can't leave
appropriate means of at 55 miles per the car without being injured while it is (3) ☺
egress. hour. going so fast.

(4) P is confined because she'll get hurt


if she tries to get out at that speed, so D (4) ☺
is keeping her in the car.
One way to prove intent D fires a pistol in (5) The intent element of battery is met (5) ☺
for some intentional P's direction, because even though he only planned to
torts is through intending to make scare P and not shoot P, that intent to
transferred intent. P think that P is commit an assault is enough intent for
One type of transferred about to get shot. battery as well.
intent is when D However, D does
intended to commit one not actually intend (6) The intent element of battery is met
(6) ☺
type of intentional tort, for the bullet to hit because D fired a pistol in P's direction

180
Law Facts Reasoning Good ☺ or
Bad
but ends up committing P. The bullet intending to make P think that P was
another type. The intent accidentally about to get shot, but not intending for
to commit the first tort strikes P. the bullet to hit P, and the bullet
is sufficient intent to accidentally struck P.
prove the second tort.
(7) The intent element of battery is met
because P meant to commit an assault (7) ☺
by shooting toward P, wanting P to
believe that P would be struck, and the
law allows the intent to commit assault
to substitute for the intent to commit
battery.

(8) The intent element of battery is met


because D intended to commit one type (8) ☺
of intentional tort, but ended up
committing another type. The intent to
commit the assault is sufficient intent to
prove the battery.
Trespass to land is when D, angry with his (9) D had intent because he smacked the (9) ☺
one intentionally enters neighbor, sneaks neighbor's horse in the rump as hard as
or causes entry upon up behind the he could, the horse was obviously
land in possession of neighbor's excitable, and the horse was facing P's
another, unless the entry obviously land, 5 feet from the boundary.
is privileged or excitable horse
consented to. The intent and smacks the (10) D had intent because hitting the
(10) ☺
required is either horse in the rump horse was a volitional act that must have
purpose to enter or as hard as he can. been calculated to make the horse freak
cause entry, or knowing The horse is out. A freaked-out horse is going to
that entry is facing toward P's take off, and since it was right next to
substantially certain. land, about 5 feet the boundary, D must have known that
from the fence that the horse would end up on P's land.
marks the
boundary. The (11) D had intent because he knew that
horse rears and the horse was nervous (it was (11) ☺
bolts, jumps the "obviously excitable"), but he didn't just
fence, and tap it; instead, he created the maximum
tramples P's effect by whacking the horse as hard as
flowers. possible. He planned to make that horse
go onto P's land, or knew for sure that it
would.

(12) D had intent because he had (12) ☺


purpose to cause entry or knew that

181
Law Facts Reasoning Good ☺ or
Bad
entry was substantially certain.
Furthermore, the entry was on land in
possession of another, and the entry was
not privileged or consented to.
A statute states that any D buys beverages (13) D is guilty because he returned (13) ☺
person who returns or in glass bottles in glass bottles in a state other than the
tries to return glass or Michigan; he pays state where he bought the bottles.
plastic bottles or a ten-cent deposit
aluminum cans for a per bottle. He (14) D is guilty because he bought the
(14) ☺
refund in any state other then goes to New bottles in Michigan and returned them
than the state where the York, where the in New York.
person bought the bottle deposit is
bottles or cans is guilty only five cents. (15) Even though D lost money, the
of a misdemeanor. He returns the statute doesn't require that D do this for (15) ☺
bottles in New a profit. D bought bottles in one state
York and takes a and returned them in another, which is
five-cent loss on all that the statute requires.
each bottle.

The tort law in the preceding chart was taken from Dobbs, THE LAW OF TORTS (West, 2000).

182
Endnote Practice

Directions:
1. Read the endnote provided.
2. Determine what elements and rules are at issue and write them in the appropriate space.
3. Find the legally important facts and write them in the appropriate space.
4. Write a reasoning statement.

1. Defendant deliberately blew pipe smoke in plaintiff’s face, knowing she was allergic to it. Is
this a battery?

Law Fact(s) Reasoning Statement

2. Is there an assault if defendant threatens the plaintiff with an unloaded gun?

Law Fact(s) Reasoning Statement

183
3. Is there an assault if the defendant aimed a gun at the plaintiff, if the plaintiff did not
know the gun was aimed at him because his back was turned?

Law Fact(s) Reasoning Statement

4. An employee is suspected of stealing property from her employer and is told that a trip
to her home is necessary to recover the property. If the employee feels mentally compelled
for fear of losing her job to go in an automobile with her supervisor to her home, has she
been involuntarily confined?

Law Fact(s) Reasoning Statement

184
5. What if she agrees to go, but during the automobile trip, she asks to stop and is denied
permission? Has she been involuntarily confined?

Law Fact(s) Reasoning Statement

6. The Dean requires students who are readmitted to Cooley after being academically
dismissed to see their faculty advisor as a condition of their readmission. Are the students
who attend this appointment confined involuntarily?

Law Fact(s) Reasoning Statement

185
7. A invites B to come to her house for dinner and an evening of playing Scrabble.
Unknown to B, A calls B’s mom, tells mom that B is being held, and demands $100,000 for
her release. Meanwhile, B is enjoying a wonderful dinner and is actually winning the
Scrabble games. Has B been involuntarily confined?

Law Fact(s) Reasoning Statement

8. Nick invites his girlfriend, Michelle, to go to the ARC staff party at the ARC director’s
house. On the way, Michelle becomes sick to her stomach. She falls asleep in the car. Nick
covers her up with a warm blanket, locks the car doors, and goes in to the party. Michelle
sleeps all the way through the party. When Nick leaves the party and gets back in the car,
she is still fast asleep. Has Michelle been involuntarily confined?

Law Fact(s) Reasoning Statement

186
9. While waving a gun, Joe tells Maybelle that he is going to kill her if she leaves the
bedroom. He goes out into the living room and listens to the big game on the big-screen TV.
Maybelle can hear him cheering for his favorite team throughout the three-hour game. Has
she been involuntarily confined?

Law Fact(s) Reasoning Statement

10. Suppose that a boy of seven, playing with a bow and arrow, aims at a girl of five, and
hits her, and she is injured. Is he liable for battery?

Law Fact(s) Reasoning Statement

187
11. Can a two-year-old child who bites an infant be liable for battery?

Law Fact(s) Reasoning Statement

12. Defendant is riding a horse, which runs away with him and runs the plaintiff down. Is
Defendant liable for battery?

Law Fact(s) Reasoning Statement

188
13. Did Sleeping Beauty have a cause of action for battery against Prince Charming?

Law Fact(s) Reasoning Statement

14. Does a plaintiff have a cause of action for battery against a doctor for an unauthorized
surgical operation to remove a non-life-threatening mole on his left forearm while under
anesthetic for a scheduled tonsillectomy?

Law Fact(s) Reasoning Statement

189
190
Week Eight:

Writing Legal Analysis

191
192
Week 8 Pre-Reading Questions
(1) What is a “call of the question”?

(2) What should you look for when deciphering the call of the question?

(3) What types of analysis might the call of the question ask you to perform?

(4) What steps can help you to plan the answer to a long essay question?

(5) What is a “trigger”? Why does it matter?

(6) What are some variations on the ERFRC pattern? When would you use them?

193
194
Reading the “Call of the Question”

The call of the question is the question, statement, or series of statements at the very end
of an essay question, after the fact pattern. It tells you what you are supposed to write about.
Before you read the facts of an essay question, first read and analyze the call of the question,
looking for three things:

1. Does the question specify which law you should discuss? If so, what is it? If the
question told you what legal concept to discuss, then the outline of the law and rules
for that area of the legal concept should pop into your head. As you read the facts, be
thinking about what law each fact goes with.

2. Does the question tell you that any particular person, thing, or event is the star in
the fact pattern? If so, who or what? If the question told you to focus on a particular
person, thing, or event, that is what you look for as you read the fact pattern. Once
you find him, her, or it in the fact pattern, look for the details that surround what that
person did or what happened. Match those details up with the elements they go with.

3. What are the key words in the question? That is, what kind of analysis are you being
asked to do to answer this question? There are four possibilities.

One-sided arguments:
(A) Explain only the prosecutor’s or plaintiff’s arguments. (Example: “How can the
prosecutor argue that defendant cannot use self-defense in this case?”)
(B) Explain only the defense’s arguments. (Example: “What is the defendant’s best
argument why he cannot be guilty of this crime?”)
Two-sided arguments:
(C) Explain both sides’ arguments and reach a conclusion. (Example: “Discuss and
decide.”) Usually, you give both sides’ arguments when the facts can be interpreted in
different ways so that the parties could disagree about what the facts really prove. But
sometimes the facts can only be interpreted in one way. If the professor writes a
particular fact so that there is only one interpretation and there is not an argument for
both sides, then don’t worry about making a counterargument for that fact.
Other:
(D) Usually, the “other” category is when the professor is simply asking for
information and is not asking you to do legal analysis. Questions like this are very
similar to the questions you probably had on college exams that simply asked you to
regurgitate information. You will see very few questions like this on your exams.
(Example: “List the elements of battery.”)

Once you have read and analyzed the call of the question, you should know what you are
supposed to do and what you are looking for. The question always contains key words. Pay close
attention to them and answer the question you are being asked.

195
Here is a picture that you can Key words
use to help you remember the
process:
Law Facts

Example: The call of the question says, “What is Joe’s best argument why he cannot be found
guilty of burglary?”

Key words: best argument, cannot be found guilty (one-sided – defend only)

Law Facts

Burglary Joe

Example: The call of the question says, “What intentional torts has Joe committed? Discuss and
decide.”

Key words: discuss and decide (two-sided analysis)

Law Facts

Intentional torts Joe

Example: The call of the question says only, “Discuss and decide.”

Key words: discuss and decide (two-sided analysis)

Law Facts

(Not specified) (Not specified)

196
Planning the Answer to a Long Essay Question
1. Find out how much time is allotted for that question. Planning your answer will
take from one-quarter to one-third of that time.

2. Read and analyze the call of the question. (See section on reading the call of the
question.)

3. If the call of the question tells you what law to discuss, pull your outline of that law
up in your head. Then read the fact pattern carefully, matching details of the events in
the fact pattern with the parts of the law.

4. If the call of the question does not tell you what law to discuss:

a. Read the facts looking for conflicts or events. A conflict is when something
happens that will make somebody angry. Conflicts are present mostly for
intentional torts and criminal law. An event is something that happens; the
event might make somebody angry, but often it is just one step in a series of
occurrences. Sometimes it only takes a few words to describe an entire event.
Most subjects are tested using a series of events (like Contracts, for example).

b. Think about what the parties’ objectives are in that part of the fact pattern.
Ask yourself this: “If I were the lawyer, and that person came into my office
and told me that story, and I asked that person what he wanted, what would he
say?” It might be money; it might be to put someone in jail or stay out of jail;
it might be to pay less in taxes; it might be to make someone do something.

c. For each conflict or event, brainstorm what legal theories might be


present that might help the parties reach their objectives. Write down all
the legal theories that might pertain to the conflict or event.

This step requires the most pre-exam preparation. To be able to brainstorm


the legal theories, you must first understand what “triggers” a legal theory.
That is, what kind of situation has to be present before you can use that legal
theory? If the facts “trigger” the theory, then you must talk about it (unless
the call of the question tells you not to). If the facts do not “trigger” the legal
theory, then you do not talk about it.

d. Match the details of the facts with the pieces of the law they go with. Pay
attention to the details. And pay particular attention to modifying words like
adjectives and adverbs. Using modifying words is how the professor tells you
that an element is met, is not met, or is arguable.
e. Begin writing your answer in the ERFRC pattern.

197
ERFRC – ERFRC – ERFRC …

Basic Pattern: Sample format for one ERFRC for a one-


Issue Statement sided argument:
E – Element The issue is …
R – Rule (definition) (1) The first element is … (2) This means …
F – Facts that are relevant to the element (3) Here, in the facts … (4) Because …
R – Reasoning (5) Therefore, this element is met (or not met).
C – Conclusion for that element

Other Patterns: There are two situations where you need to use ERFRFR…C.
(1) If multiple facts fit one R, then use ERFRFR…C. Keep doing FR (facts and reasoning)
until you have presented all the facts and reasoning. Then conclude whether that element has
been met or not met (C).

Issue Statement Sample format for one ERFRC when there


E – Element are multiple facts
R – Rule (definition) The issue is …
F – Facts that are relevant to the element (1) The first element is … (2) This means …
R – Reasoning (3) Here, in the facts … (4) Because …
F – More facts that are relevant to the element (5) The facts also say … (6) Because …
R – Reasoning (7) Finally, the facts say … (8) Because …
…. (9) Therefore, this element is met (or not met).
C – Conclusion for that element

(2) If both sides have arguments that you need to explore, you also use ERFRFR…C. Do
one side’s FR (facts that support that side and reasoning); then do the other side’s FR (facts that
support that side and reasoning); then conclude with an explanation of which side’s argument is
stronger.

Issue Statement Sample format for one ERFRC for a two-


E – Element sided argument:
R – Rule (definition) The issue is …
F – One side’s facts (1) The first element is (2) This means …
R – Reasoning (3) Here, in the facts … (4) Because …
F – Other side’s facts (5) The other side might argue …
R – Reasoning (6) Because … (7) Overall the __ side is
C – Conclusion for that element stronger because …

Repeat the ERFRC pattern as many times as you have elements to analyze.

When you have finished all the ERFRCs, answer the question that the issue stated. Have all
the elements been proved?
C = Answer to the question you asked in the issue statement.

198
Question One

(15 Minutes)

The State of Cooley has codified its burglary offenses. Section 750.110 of the Cooley Penal
Code states:

“Any person who breaks and enters with the intent to commit a felony or a larceny therein a tent,
hotel, office, store, shop, warehouse, barn, granary, factory, or other building, structure, boat,
ship, truck, or railroad car shall be guilty of a felony punishable by imprisonment for not more
than 10 years.”

On the evening of April 2, 1999, Trent Neff cut his way through a ten-foot-high chain-link fence
topped by razor wire. The fence surrounded a recycling company located in an isolated corner of
Cooley County. Once he was through the fence, he approached a tractor-trailer truck containing
glass bottles. The back door of the truck was standing wide open. He walked right in, filled up a
trash bag with bottles, and then walked back out the open door. As he was crawling back
through the hole he had cut in the fence, he was arrested. He later admitted to police that he had
intended to steal the bottles and sell them back to the recycling company.

Briefly discuss Neff’s argument(s) why he cannot be successfully prosecuted for a violation
of Section 750.110.

Question Two

(15 Minutes)

The state of Michissota has enacted a statute that reads as follows:

“If a dog bites a person, without provocation, while the person is on public property, or lawfully
on private property, including the property of the owner of the dog, the owner of the dog shall be
liable for any damages suffered by the person bitten, regardless of the former viciousness of the
dog.”

Jim Hawkins had decided to spend the day on the public beach, looking for buried treasure with
his metal detector. After a fruitless search, he started to walk back to his car. When he reached
the parking lot, he noticed a small anchor hanging from the tree of the local private boat club
owned by John Silver. Jim jumped the fence and began scanning the ground under the tree with
his detector. As he scanned, he thought he heard a faint beep, so he turned the speaker up as
loud as it could go. Unknown to Jim, the speaker emitted a high-pitched whine that irritated
Silver’s dog. The dog jumped off the patio and chomped down onto Jim’s leg, causing severe
injury to his calf muscle.

Briefly discuss Silver’s best argument(s) why he shouldn’t be held liable under this statute.

199
200
“Oh Nuts”

Penny and David went on a blind date. Penny’s friend Nina had fixed them up as a gift for
Penny’s 21st birthday because Nina felt that David was Penny’s type. Dinner went fine, but
unbeknownst to Penny, the enchiladas she had eaten had been deep-fried in peanut oil. After
going to a movie, David took Penny to her apartment. Penny was feeling strange and she started
to develop trouble breathing. As it turns out, Penny was having an unexpected allergic reaction
to the peanut oil. David gave her two strong antihistamines because he recognized that she was
in distress. The reaction subsided, but Penny fell sound asleep. David, feeling as if the date had
ended too quickly, decided to try to wake Penny and ask if they could have sex before he went
home.

David shook Penny several times to wake her. She groaned and opened her eyes, so David asked
her to have sex with him. She barely managed to mutter, “Yeah, I gesso, waddever.” Then she
drifted back into an antihistamine-induced sleep.

The next morning, after realizing that David had had sex with her the night before, she called the
police, who quickly showed up at her apartment. Penny filed a criminal complaint, and the police
later arrested David at his apartment.

In your opinion, will the prosecutor win if she charges David with third-degree criminal
sexual conduct? Does David have any defenses? Discuss and decide.

**********

MCR §750.520d. Third-Degree CSC

1. A person is guilty of criminal sexual conduct in the third degree if the person engages in
sexual penetration with another person and if any of the following circumstances exists:

A. That other person is at least 13 years of age and under 16 years of age.

B. Force or coercion is used to accomplish the sexual penetration.

C. The actor knows or has reason to know that the victim is mentally incapable,
mentally incapacitated, or physically helpless.

2. Criminal sexual conduct in the third degree is a felony punishable by imprisonment for
not more than 15 years.

201
Sec. 520a. As used in this chapter:

(a) "Actor" means a person accused of criminal sexual conduct.


(b) "Developmental disability" means an impairment of general intellectual functioning or adaptive behavior
which meets the following criteria:
(i) It originated before the person became 18 years of age.
(ii) It has continued since its origination or can be expected to continue indefinitely.
(iii) It constitutes a substantial burden to the impaired person's ability to perform in society.
(iv) It is attributable to 1 or more of the following:
(A) Mental retardation, cerebral palsy, epilepsy, or autism.
(B) Any other condition of a person found to be closely related to mental retardation because it
produces a similar impairment or requires treatment and services similar to those required for a
person who is mentally retarded.
(c) "Intimate parts" includes the primary genital area, groin, inner thigh, buttock, or breast of a human being.
(d) "Mental health professional" means that term as defined in section 100b of the mental health code, 1974 PA
258, MCL 330.1100b.
(e) "Mental illness" means a substantial disorder of thought or mood which significantly impairs judgment,
behavior, capacity to recognize reality, or ability to cope with the ordinary demands of life.
(f) "Mentally disabled" means that a person has a mental illness, is mentally retarded, or has a developmental
disability.
(g) "Mentally incapable" means that a person suffers from a mental disease or defect which renders that person
temporarily or permanently incapable of appraising the nature of his or her conduct.
(h) "Mentally incapacitated" means that a person is rendered temporarily incapable of appraising or
controlling his or her conduct due to the influence of a narcotic, anesthetic, or other substance administered
to that person without his or her consent, or due to any other act committed upon that person without his or
her consent.
(i) "Mentally retarded" means significantly subaverage general intellectual functioning which originates
during the developmental period and is associated with impairment in adaptive behavior.
(j) "Nonpublic school" means that term as defined in section 5 of the revised school code, 1976 PA 451, MCL
380.5.
(k) "Physically helpless" means that a person is unconscious, asleep, or for any other reason is physically
unable to communicate unwillingness to an act.
(l) "Personal injury" means bodily injury, disfigurement, mental anguish, chronic pain, pregnancy, disease, or
loss or impairment of a sexual or reproductive organ.
(m) "Public school" means that term as defined in section 5 of the revised school code, 1976 PA 451, MCL
380.5.
(n) "Sexual contact" includes the intentional touching of the victim's or actor's intimate parts or the intentional
touching of the clothing covering the immediate area of the victim's or actor' s intimate parts, if that
intentional touching can reasonably be construed as being for the purpose of sexual arousal or gratification,
done for a sexual purpose, or in a sexual manner for:
(i) Revenge.
(ii) To inflict humiliation.
(iii) Out of anger.
(o) "Sexual penetration" means sexual intercourse, cunnilingus, fellatio, anal intercourse, or any other
intrusion, however slight, of any part of a person's body or of any object into the genital or anal openings of
another person's body, but emission of semen is not required.
(p) "Victim" means the person alleging to have been subjected to criminal sexual conduct.

202
Week Nine

Issue Spotting

203
204
Planning the Answer to a Long Essay Question
Contracts
Sandra loved to collect perfume bottles. She had so many that she was running out of
room in her home, so she decided to sell some of her least favorite ones. On Saturday, March 2,
she put an ad in her local newspaper.

FOR SALE: twelve antique perfume bottles, $50.00 each. Six are Italian art glass,
from the 1920’s. Three are pink, two are green, and one is yellow. The other six
are French, from the 1930’s, in varying shades of burgundy. If you are interested,
come to my house at 200 Main Street between noon and 5:00 p.m. on Saturday,
March 16. Ask for Sandra. First come, first served.

Bob saw the ad on March 2. He immediately called Sandra and told her he wanted to buy
all the perfume bottles, but he wanted a bulk discount for taking them all off her hands. He
would pay $40.00 per bottle. At that moment, Sandra’s cat, Daniel, started howling. Sandra
apologized, explained that her cat was in distress, and said she would have to go. Both Bob and
Sandra hung up.

The next day, Bob called back. He told Sandra that he still wanted to buy the perfume
bottles for $40.00 each. However, he had some unexpected bills that month, and he wasn’t sure
that he was going to have enough money after all. He offered to give Sandra $20.00 if she would
agree not to sell the perfume bottles to anyone else for 5 days. Sandra said she would agree to
that. Later that day, Bob and his girlfriend decided to elope. They jumped on a plane for Las
Vegas, and Bob forgot to send Sandra the $20.00. When he came back three days later, he called
Sandra and said that he wanted to buy the bottles that day. By now, Sandra figured that Bob was
some kind of goofball, so she told him to get lost.

Carrie didn’t subscribe to the paper, but she heard about the ad from a co-worker. She
wrote Sandra a letter on March 8:

Dear Sandra:

I love Italian art glass. I’ll buy the yellow one, sight unseen.
Please pack it up very carefully and send it to me, and I will send you a
check. I live at 775 Elm Street. Thanks.

Signed, Carrie

Sandra received the letter on March 9 and wrote back as follows:

205
Dear Carrie:

I’m not sure I that I still want to sell that one. How about buying
one of the green ones instead? Let me know by March 12th.

Signed, Sandra

The U.S. Postal Service lost Sandra’s properly addressed, stamped letter, and Carrie
never received it.

Joe was on vacation in Florida when the ad came out in the newspaper. He didn’t see the
newspaper until the morning of March 16. He went straight over to Sandra’s house, arriving at
11:00, and rang the doorbell. However, Sandra wasn’t home. She was at the grocery store,
buying cat food. Joe hung around for a few minutes, got bored, and wrote a note to Sandra:

Dear Sandra:

I came to your house, but you weren’t there. I’ll buy all of the
perfume bottles at the price stated in your ad. Call me at 555-1878 and let
me know when to come by to pick them up. Don’t sell them to anyone
else!

Signed, Joe

Sandra got home at noon, saw Joe’s note, and was about to call him when Abigail came
by and bought all twelve perfume bottles for $50.00 apiece.

Bob, Carrie, and Joe are all upset. They all think that they had a deal with Sandra.
Are they right? Discuss and decide. Do NOT discuss the UCC for this question; instead,
discuss only common-law principles.

206
Planning the Answer to a Long Essay Question
Intentional Torts
“Take Me Out to the Ballgame”
90 minutes

Jim loved to play softball. He and a bunch of friends would get together almost every
Saturday so that they could play. He and his friends would form one team (they called
themselves “The Old Guys”); the other team consisted of whoever else showed up (the other
team was always designated “The Drunken Bums”).

Last Saturday, Jim was playing shortstop. The Drunken Bums already had one person,
Macho Man, on first base. The batter, Ugly Smith, managed to hit the ball pretty hard.
Unfortunately, it flew right to Jim, and Jim caught it. That meant that Smith was out. Macho
Man ran toward second base. That turned out to be a mistake because Jim quickly tossed the ball
to the first baseman, Chris Jones. Macho saw this and quickly turned around. He ran toward first
base as fast as he could. But before he could get there, Chris caught the ball and stepped on first
base.

Macho saw what was happening; he knew that as soon as Chris’s foot touched the base,
he was going to be out anyway, so he decided to do as much damage as possible. He started to
slide into first base (even though he couldn’t possibly beat the throw to first base) and as he slid,
he aimed his feet at Chris’s ankle. He hit Chris with tremendous force, and Chris’s ankle
snapped. It was broken.

Meanwhile, as Ugly walked off the field, he shouted angrily, and loudly, at Jim, “You
jerk! I told you that the next time you did that, I was going to tear your heart out! That time is
now!” As he reached his buddies, he snarled, “I’m sick of being beaten by those Old Guys! I’m
going to beat that Jim to a pulp!”

While Ugly was saying these things, Chris was writhing on the ground. He was in severe
pain. Macho was standing over him, grinning. Suddenly he started to pull his leg back. Buzz
Ground, an Old Guys player, thought that Macho was going to kick Chris. (Actually, Macho was
only going to stretch his leg muscles out – after sliding into first base, his leg had started to
cramp up.) Buzz grabbed a baseball bat and ran over as fast as he could. Coming up behind

207
Macho, he raised the bat to strike Macho in the head. However, as he drew the bat back, Gene
Green grabbed the bat at the last second and stopped him. Gene was an Old Guys player who had
rushed up to help Chris. Macho had seen Buzz swing the bat at him; he had ducked, but if Gene
hadn’t grabbed the bat, Macho wouldn’t have been able to get out of the way in time.

Then everybody piled out onto the field, but Jim managed to keep them from fighting. He
loaded Chris into his car and took him to a nearby hospital.

At the hospital, Chris was in too much pain and shock to sign the consent form. He gave
the doctor express oral consent to fix his broken ankle. Then he passed out. Shortly afterward, he
was wheeled into the operating room where the doctor not only repaired the broken ankle, but
also removed a cyst from Chris’s left wrist. (The cyst was about an inch thick and two inches in
diameter; it was infected, so it had been causing Chris some pain, but it wasn’t dangerous. Chris
had an appointment with his own doctor to get the cyst removed the following Monday.)

Discuss the rights and liabilities of all the parties.

208
Planning the Answer to a Long Essay Question
Contracts
Practice Issue-Spotting
“Sibling Rivalry”
55 minutes

Objective: To practice issue-spotting in a fact pattern containing a series of events (Contracts)

Brother (B) and Sister (S) over the years had admired a brass lamp at their Parents’ (Ps)
home. During many conversations, S had told B that if S “got” the lamp in Ps’ will, S would give
the lamp to B because of the many kindnesses B had shown S in the past. Each time, B said,
“You are such a wonderful person! I accept.” Ps died, leaving the lamp (as well as Ps’ clock) to
S in Ps’ will. However, when B reminded S of S’s promise, S replied: “Grow up! That lamp was
made by Tiffany and is worth thousands. If you want it, you’ll have to buy it.” A (an attorney)
advised B that S legally had to turn the lamp over to B. Notwithstanding this advice, on March 1,
B telephoned S and said B might be willing to pay for the lamp and quoted a price of $8,000 for
the lamp if S would consider selling Ps’ clock as well. However, S never responded and hung up.
A few days later, on March 3, S signed and mailed a personal note to B, stating that S was
willing to sell the lamp and clock for $10,000, but that the deal was only open for 7 days from
the date of S’s note. B received S’s note on March 7th. B wanted the lamp and clock, but S’s
attitude really bothered B. Consequently, on March 10th, B mailed a letter to S that said, “Fine, I
agree to your terms: the lamp and clock for $10,000, provided you personally bring them to me,
fall on your knees, and apologize for being so hateful.” B’s letter was lost by the post office and
was never delivered. Because S did not hear from B after 7 days, S informed B that they did not
have a deal on the lamp and clock.

Discuss all issues, and include in your answer a discussion of:


(1) Whether A was correct that S legally had to turn the lamp over to B;
(2) Whether S and B have an enforceable contract for the lamp and clock; and
(3) An explanation of how your answer for (2) would change (if at all) if S were a
merchant.

209
Planning the Answer to a Long Essay Question
Issue-Spotting Intentional Torts
“Ray, Jill, and the Hotdog Cart!”
Ray Allen was out for a drive, cruising and listening to some blues. As he approached a
railroad crossing, he saw that a car was stalled on the tracks and a train was approaching. The
train hit the back of the stalled car, making it spin around and crash into a ditch. Ray jumped out
of his car and ran to the other car to see how the driver was.

Suddenly, the car caught on fire. The driver, Jill Hurst, was unconscious. Ray tried to pull
her out, but she was stuck somehow. He was desperate to get her out and yanked hard on her
shoulder. He pulled her out of the car, but he had to sprain her shoulder to do so.

Jill was now barely conscious and complaining of severe pain, so Ray decided to rush her
to the hospital. He lifted her into the back seat and took off. In his haste as he sped through the
city, he turned the wrong way down a one-way street, right into the path of an oncoming taxicab.
To avoid hitting the cab, Ray swerved onto the sidewalk and crashed into a portable hot-dog cart.

1. If Jill sues Ray, for what intentional tort will Jill have the best chance of
winning? Explain thoroughly. What privileges (defenses) will Ray argue
in response, and what will be the likely outcome? Explain thoroughly.

2. If the owner of the hot-dog cart sues Ray, what intentional tort(s) is he
likely to claim? What are his chances of success? Explain thoroughly.
What privileges will Ray argue in response, and what will be the likely
outcome? Explain thoroughly.

210
Writing Practice

211
212
Sample Essay Question
“Jimmy and the Petunias”
25 minutes

Jimmy is standing by the front of his house, watering his mother’s new petunia plants.
Jimmy’s mom has just put in the new bed of petunias to decorate the very front corner of the
house. Jimmy spies an elderly man walking by the front of the house on the sidewalk. The
sidewalk is more than 50 feet from the petunia bed. Jimmy squirts the gentleman with the water
from the hose, knocking his glasses off. The glasses fall to the sidewalk and are broken.

What is the best cause of action the elderly gentleman has? What are his chances of
winning? Discuss and decide.

213
Instructions for Sample Essay
“Jimmy and the Petunias”

Objective:

To practice the legal-analysis pattern

Procedure:

1. Read and analyze the call of the question. What is it asking you to do?

2. In the “law” column of the T-chart (see “Plan Your Answer”), write down the elements
of the intentional tort you have chosen. Then read the facts carefully, looking for the
details of the events taking place. Pay close attention to modifying words like
adjectives and adverbs.

3. When details in the fact pattern “fit” one of the elements of the tort you have chosen,
make a note on the exam question or in the “facts” column of the T-chart to remind
yourself which facts go with which elements.

4. If you wish, you can use the template below to help you organize your answer. Please
use full sentences. Insert all the elements, the rules that explain what those elements
mean, the facts that “fit” each element, the reasoning that explains why those facts do or
do not prove each element, and a conclusion on whether each element has or has not been
met. (ERFRC, ERFRC, ERFRC . . . C) When you run out of elements, go to the C at
the end. Have all the elements been proved? Then the C is that the elderly gentleman
will win. If an element has not been proved on these facts, then the C is that the elderly
gentleman will lose.

5. Evaluate your answer using the Evaluation Sheet you will be given. Also, compare it
with the sample answer.

214
Plan Your Answer
“Jimmy and the Petunias”

Law Facts

215
“Jimmy and the Petunias”

You will not necessarily use all of the spaces in this template.

Issue:
E: element:

R: rule

F: facts

R: reasoning

Conclusion:

E: element

R: rule

F: facts

R: reasoning

Conclusion:

E: element

R: rule

216
F: facts

R: reasoning

Conclusion:

E: element

R: rule

F: facts

R: reasoning

Conclusion:

C:

217
218
Instructions for Sample Essay
Intent to Contract
“Little Red Corvette”

Objective:

To practice the legal-analysis pattern

Procedure:

1. Read and analyze the call of the question. What is it asking you to do?

2. In the “law” column of the T-chart (see “Plan Your Answer”), write down the rules
required to prove intent to contract (from Lucy v Zehmer). (A blank t-chart has been
provided. Hint: According to the Restatement § 71, reprinted in the case, there is both a
general rule and an exception.) Then read the facts carefully, looking for the details of
the events taking place. Pay particular attention to modifying words.

3. When details in the fact pattern “fit” one of the rules, make a note on the exam question
or in the “facts” column of the T-chart to remind yourself which facts go with which
pieces of law.

4. We did not give you a template this time. If you wish to use a template, you can write one
out.

Here is how your answer should be organized:

State the general rule 2 points


Facts and reasoning that support Joe 4 points
Facts and reasoning that support Ed 4 points
Reach an appropriate conclusion 1 point

State the exception 1 point


Facts and reasoning that support Joe 2 points
Facts and reasoning that support Ed 2 points
Reach an appropriate conclusion 1 point

Reach an appropriate ultimate conclusion 2 points

Points awarded: __________


Divided by 19 =

Percentage: ______________

219
Sample Brief: Lucy v Zehmer, Va. Supreme Court
Issue: Whether a person is deemed to have intended to contract when he acted like he was serious but secretly meant it as a joke

Reasoning Law Facts

Intent to contract: Procedure: Trial court held that Ps couldn’t show


that they were entitled to specific performance;
P’s appealed
To decide whether a person has intent
to contract:
D and P negotiated for the sale of farm for 40
-The things D did were the kind of • Look at the person’s outward minutes, then D started to write a sales agreement,
things that a person would do if the expressions of intent but P and D rewrote so that it said “we” (D and
D’s wife) both agreed to sell the land. D and D’s
person were serious about selling the wife and P all signed the paper, which said that
land. Because D’s actions would • The law imputes intent that Ds would sell the farm for $50,000, title
cause a reasonable person to think he corresponds to the reasonable meaning satisfactory to the buyer. P took the paper and
was serious, intent to contract is of a person’s words or acts (objective offered $5 to seal the bargain, which D refused.
imputed to him because he test)

220
outwardly seemed to accept the deal. D claimed he was drunk and joking and did not
-It doesn’t matter whether it was a The secret undisclosed intent is mean to make a contract; when D told his wife to
serious offer by P and a serious irrelevant sign, he whispered, so that P could not hear, that it
acceptance, or whether it was a was meant as a joke.
serious offer and a secretly joking
acceptance. D’s secret intent doesn’t
matter.

P actually believed that D was Unless the other person knows that the P and D were both drinking, but D didn’t drink so
serious (and he was justified in person doesn’t have the intent to much that P thought D didn’t know what he was
doing. D never took the paper back after he
thinking so), and P had no way to contract. signed it. So P thought D was serious and moved
know that D was not serious. forward with the deal: he got an attorney to
examine the title and arranged to get the money.
Sample Essay Question
Intent to Contract
“Little Red Corvette”

Joe and Ed had been enemies for 20 years. Ed knew that Joe was having business
problems and was really hard up for cash. One evening, Joe was drinking at the Drop-Inn
Tavern. Ed was there too: he saw Joe drink six 12-ounce beers and six shots of bourbon
in the space of about three hours. Ed was drinking Coca-Cola. When Joe started to act
quite tipsy, Ed approached Joe and told him that he had long admired Joe’s beautiful
classic Corvette car. Ed and Joe both knew that the car was worth about $25,000. Ed
offered to buy it for $15,000, and Ed also offered to throw in two bushels of tomatoes
from his garden just to “sweeten the deal.” Joe started to laugh. He said, “Yeah, right.
You bet. You’re just the person I want to sell my sweet Corvette to.” Joe took a cocktail
napkin and wrote, “I hereby agree to sell for $15,000 my sweet Corvette (which is worth
$25,000) to Ed, the jerk. Oh, and he promises to give me tomatoes, too. Wow!” Joe
signed his name on the cocktail napkin and, laughing uproariously, threw it on the bar. Ed
took the napkin and left.

The next day, Ed went to Joe’s house and tendered a check for $15,000. He also
brought along two bushels of tomatoes. Joe, suffering from a hangover, angrily refused to
accept either.

Ed has sued Joe for breach of contract. Joe wants to argue in response that
he had no intent to form a contract. What arguments can Joe make? What
arguments can Ed use to respond? Who do you think should win? Why?

221
Plan Your Answer
Sample Essay Question
Intent to Contract
“Little Red Corvette”

Law Facts

222
Instructions for Sample Essay
Trespass to Land
“Psychics on Drugs”

Objectives:
to become familiar with the legal-analysis pattern
to practice the legal-analysis pattern

Procedure:
1. Read and analyze the call of the question. What is it asking you to do?

2. In the “law” column of the T-chart (“Plan Your Answer”), write down the
elements of trespass to land. Then read the facts carefully, looking for the
details of the events taking place. Pay particular attention to modifying
words – adjectives and adverbs.

3. When details “fit” one of the elements of trespass to land, make a note on the
exam question or in the “facts” column of the T-chart to remind yourself
which facts go with which elements.

4. If you wish to use the template, fill it in. Insert all the elements, the rules that
explain what those elements mean (for those elements that have rules), the
facts that “fit” that element, the reasoning that explains why those facts do or
do not prove that element, and a conclusion on whether that element has or
has not been met.
5. When you run out of elements, go to the C (the ultimate conclusion) at the
end. Have all the elements been proved? Then the C is that Leah can win
the trespass to land case. If an element has not been proved on these facts,
then the C is that Leah will lose.

6. If you do not wish to use the template, just write the answer out in paragraph
form. Be sure to follow the ERFRC pattern. Remember, you will not fill out a
template like this when you take the actual exam; the purpose of the template
is to get used to organizing the information in the legal-analysis pattern.

Here is how your answer should be organized:

Entry onto plaintiff’s land:


Element 1 point
Rules 2 points
Facts 2 points
Reasoning 2 points
Conclusion 2 points

223
Intent to enter the land:
Element 1 point
Rule 1 point
Facts 1 point
Reasoning 1 point
Conclusion 1 point

Unpermitted entry:
Element 1 point
Rule 1 point
Facts 1 point
Reasoning 1 point
Conclusion 1 point

Ultimate conclusion

Extra points for damages analysis 4 points

Points awarded: __________


Divided by 23 =

Percentage: _____________

224
Trespass to Land
Sample Essay Question
“Psychics on Drugs”

Leah rents a house in a subdivision in East Lansing, Michigan. She is very

interested in both spirituality and animals. So when two members of the Church of the

Kitty-Cat Psychics came to her house to tell her about their beliefs, she was intrigued

enough to let them come in. The two Church members, Abe and Ben, left their large,

heavy bags of leaflets on the front porch, went in, and told her all about the Church.

They pressured her intensely to make a donation. Their insistence quickly became

annoying. After an hour, Leah had had enough, and she asked them to please leave. As

they started out the door, Leah shouted after them, “Take your stuff and don’t come

back!”

Leah didn’t know it, but Abe and Ben, in addition to being fanatical Kitty-Cat

Psychics, were also drug abusers. They were carrying very large bags of leaflets, but

inside the bags they were carrying various kinds of illegal pills. They were tired of

hauling the heavy bags around the neighborhood, but they hadn’t yet been to all the

houses. So they each grabbed a handful of leaflets and stashed the bags in the bushes at

the front of Leah’s house. Then they went on to the next house.
An hour later, Leah’s six-year-old son, Jimmy, got home from school. He spotted

the bags in the bushes and went to investigate. Finding the pills and thinking they were

candy, he quickly ate half a bottle. An hour later, he started acting very strangely, so

Leah rushed him to the hospital where doctors pumped his stomach.

Leah is furious. She wants to sue Abe and Ben for trespass to land. If she does, can
she win? What types of things can she recover damages for? Discuss and decide.

225
Plan Your Answer
Sample Essay Question
Trespass to Land
“Psychics on Drugs”

Law Facts

226
Template for Sample Essay
Trespass to Land
“Psychics on Drugs”

You will not necessarily use all of the spaces in this template.

Issue:
E: element:

R: rule

F: facts

R: reasoning

Conclusion:

E: element

R: rule

F: facts

R: reasoning

Conclusion:

E: element

R: rule

F: facts

R: reasoning

Conclusion:

E: element

R: rule

F: facts

227
R: reasoning

Conclusion:

E: element

R: rule

F: facts

R: reasoning

Conclusion:

E: element

R: rule

F: facts

R: reasoning

Conclusion:

E: element

R: rule

F: facts

R: reasoning

Conclusion:

E: element

R: rule

F: facts

R: reasoning

Conclusion:
C:

228
Call-of-the-Question Exercise
Burglary and . . . .
“David Ate a Sandwich – So What?”

Objectives:
Understand how crucial analyzing the call of the question is.

Practice analyzing the call of the question.

Procedure:
1. Following are two essay questions. The facts are exactly the same, but the calls of the
question are different. For each one, read and analyze the call of the question. What is
each one asking you to do?

2. Elements of two common-law crimes have been provided for you. In the “law” column
of the T-chart (“Plan Your Answer”), write down the elements and rules for the crime(s)
that are relevant for the first question. Then read the facts carefully, looking for the
details of the events taking place. Pay attention to the modifying words – adjectives
and adverbs.

3. When details “fit” one of the elements or rules for the crime you are analyzing, make a
note on the exam question or in the “facts” column of the T-chart to remind yourself of
which facts go with which elements.

4. Now look at the call of the question for the second essay. In the “law” column of the T-
chart, write down the elements and rules that are relevant for the crime(s) that are
relevant for the second question. Then read the facts carefully, looking for the details of
the events taking place.

5. When details “fit” one of the rules or elements for the crime(s) for the second question,
make a note on the second exam question or in the “facts” column of the T-chart.

6. Write out the answer for each question. Be sure to use the ERFRC pattern.

Here is how your answer should be organized:

Question #1
Choose the correct crime 2 points
First element, rule, facts, reasoning, conclusion 6 points
Second element, rule, facts, reasoning, conclusion 6 points
Third element, rule, facts, reasoning, conclusion 6 points
Fourth element, rule, facts, reasoning, conclusion 5 points
Fifth element, rule, facts, reasoning, conclusion 4 points
Sixth element, rule, facts, reasoning, conclusion 6 points
Give an appropriate ultimate conclusion 1 point

229
Question #2
Choose the correct argument (the correct element) 2 points
State the element, rule, facts, reasoning, and conclusion 6 points
Do not analyze what the question did not ask about 3 points

Points awarded: __________


Divided by 47 =

Percentage: _____________

230
Fact Patterns for Call-of-the-Question Exercise
Burglary and . . . .
“David Ate a Sandwich – So What?”

Question #1:

David Alexander is a homeless person. He was walking down a road at about


10:00 p.m. in late November, and it was getting cold out. So he went up to a house that
had no lights on. He tried the closed front door. It was unlocked. He went inside so that
he could get warm. There was nobody home.

Once inside, he started feeling hungry. He opened the refrigerator and saw food
and beer. He made himself a sandwich and drank two beers. Then he fell asleep.

Around 2:00 a.m., the owners came home. They saw David asleep on the couch,
so they called the police. David was arrested.

What crime under the common law is David guilty of? Why?

Question #2:

David Alexander is a homeless person. He was walking down a road at about


10:00 p.m. in late November, and it was getting cold out. So he went up to a house that
had no lights on. He tried the closed front door. It was unlocked. He went inside so that
he could get warm. There was nobody home.

Once inside, he started feeling hungry. He opened the refrigerator and saw food
and beer. He made himself a sandwich and drank two beers. Then he fell asleep.

Around 2:00 a.m., the owners came home. They saw David asleep on the couch,
so they called the police. David was arrested.

What is David’s best argument why he is not guilty of common-law burglary?

(See the next page for elements of crimes.)

231
Elements and Rules for Burglary and Larceny
Call-of-the-Question Exercise
“David Ate a Sandwich – So What?”

Note: If any of these elements or rules differ from what your professor has
explained to you, use the elements or rules that your professor gave you!!

Common-Law Burglary Common-Law Larceny

- breaking (applying some force, no matter - trespassory (deprived the victim of the
how slight, to create or enlarge an opening) right to physical possession of the property
without lawful consent)
- entry (the person, or part of the person, or
an extension of the person intrudes into the - taking (getting physical control over the
interior) item)

- dwelling house (currently used for - carrying away (any movement of the
human habitation; no one need be item)
physically present)
- personal property (moveable property
- of another (D has no permission to enter) that was not excluded under common law
as being subject to larceny: for example,
- in the nighttime (between dusk and land was excluded)
dawn)
- of another (D has no right to possess the
- with the intent to commit a felony once property)
inside (D must have this intent at the
moment he breaks and enters) - with intent to permanently deprive the
owners of their property

232
Sample Essay Question
Reading Criminal Statutes
“Should She Pay the Ticket?”

The State of Cooley has a number of statutes that govern people who drive on
public roads. One of them reads as follows:

The driver of a vehicle overtaking or meeting a school bus which has


stopped and is displaying 2 alternately flashing red lights located on the
same level shall bring the vehicle to a full stop not less than 20 feet from
the school bus and shall not proceed until the school bus resumes motion
or the visual signals are no longer actuated. At an intersection where
traffic is controlled by an officer or a traffic stop-and-go signal, a vehicle
need not be brought to a full stop before passing a stopped school bus, but
may proceed past the school bus at a speed not greater than is reasonable
and proper but not greater than 10 miles an hour and with due caution for
the safety of passengers being received or discharged from the school bus.
The driver of a vehicle who fails to stop for a school bus as required by
this section or who passes a school bus in violation of this section is
responsible for a civil infraction.

Last week, Joni Mitchell was driving to work. She was a little late, so she was
driving about 55 miles per hour in a 45-mile-per-hour zone. The road she was driving on
was a two-lane highway. As she came around a curve, she saw a school bus ahead of her,
driving in the same direction as she was. When she was about 150 feet behind the bus,
the bus started to slow down and pull over to the shoulder. Neither the bus nor Joni was
at an intersection; the bus was simply stopping in front of some houses to pick up some
school children. As the bus moved onto the shoulder, the bus driver activated the
alternating flashing red lights (the lights were on the same level, directly across from
each other). Because Joni was in a hurry, she pulled into the left lane and passed the bus
just after the bus came to a full stop. She had slowed down somewhat, so she was
moving at approximately 35 miles per hour. Once she was past the bus, she speeded up
again.
A police officer saw the whole thing. He pulled Joni over and wrote her a ticket.
The ticket charged her with a civil infraction for violating the statute printed above.

Joni has come to you, asking for your advice. She wants to know whether these
facts give her some hope of fighting the ticket or whether she should just pay it.

233
Reading Criminal Statutes
“Should She Pay the Ticket?”

Objectives:
Practice breaking statutes down into their individual pieces and seeing what the pieces
are and how they fit together.

Practice essay writing.

Procedure:
1. Read and analyze the call of the question. What is it asking you to do?

2. Find the statute in the fact pattern. Break it down so that you can see the separate
ideas that are present there. Organize the pieces so that you can see how they fit
together. This statute is the law that governs this question – that is, it gives the
elements. Make yourself a T-chart and write the pieces of law in the left-hand side.

3. Read the facts carefully, looking for the details of the events taking place. When
details “fit” one of the elements or rules, make a note on the exam question or in the
“facts” column of the T-chart to remind yourself of which facts go with which
elements.

4. Write out the answer. Be sure to use the ERFRC pattern.

Here is how your answer should be organized:

Find all the elements required for the statute to apply

List each requirement, explain it, and give the facts, 12 points
reasoning, and conclusion

If the statute applies, find the requirements for what


the driver must do

For each requirement, explain it, and give the facts, 6 points
reasoning, and conclusion

Reach an appropriate ultimate conclusion 1 point

Do not analyze parts of the statute that do not apply 2 points

Points awarded: __________


Divided by 21 =

Percentage: _____________

234
Sample Essay Question
Interpreting and Applying Statutes
“The Case of the Smoking Car”

Al Smith lives in Cooleyville in Michiana. He owns an old car – really old. Not

only is it old, but Al is one of those people who is inept when it comes to mechanical

things. He can barely change a light bulb, so he is certainly not capable of fixing

something as complicated as a car.

For the last six months, the car’s condition has been getting worse and worse.

Every time Al drives his car and presses down on the gas pedal, the car spews out large

clouds of blue smoke and other noxious vapors. Drivers behind him never tailgate him: if

they do, they can’t breathe.

Last Tuesday, Deputy Bud Lite was in his patrol car, driving behind Al. Al

stopped at a stop sign. When he pressed down the accelerator to get the car moving, the

usual large cloud of smoke billowed out, enveloping the patrol car in fumes. Deputy

Lite, choking, pulled Al over and arrested him.

The state of Michiana has a statute that states as follows:

Any person who owns, operates, or has in his possession any motor
vehicle that is equipped with a device for producing excessive smoke or
gas, or is so equipped as to permit oil or any other chemical to flow into or
upon the exhaust pipe or muffler of such vehicle, or is equipped in any
other way to produce smoke or dangerous or annoying gases from any
portion of the vehicle, other than the ordinary gases emitted by the exhaust
of an internal combustion engine under normal operation, shall be guilty
of a felony.

Al has been charged with violating this statute. Discuss and decide whether he should be
found guilty.

235
Instructions
Interpreting and Applying Statutes
“The Case of the Smoking Car”

Objectives:
1. Practice breaking statutes down into their individual pieces to see what the pieces are
and how they fit together

2. Practice essay writing

Procedure:
1. Read and analyze the call of the question. What is it asking you to do?

2. Find the statute in the fact pattern. Break it down so that you can see the separate ideas that
are present there. Organize the pieces so that you can see how they fit together. This statute
is the law that governs this question – that is, it gives the elements. Make yourself a T-chart
and write the pieces of law in the left-hand side.

3. Read the facts carefully, looking for the details of the events taking place. When details “fit”
one of the elements or rules, make a note on the exam question or in the “facts” column of
the T-chart to remind yourself of which facts go with which elements.

4. Write out the answer. Be sure to use the ERFRC pattern.

Here is how your answer should be organized:

Find the first element, explain it, and give the facts, 2 points
reasoning, and conclusion

Find the second element, explain it, and give the facts, 9 points
reasoning (including counter-arguments, if any)
and conclusion

Find the third element, explain it, and give the facts, 4 points
reasoning (including counter-arguments, if any)
and conclusion

Reach an appropriate conclusion 1 point

Points awarded: __________


Divided by 16 =
Percentage: _____________

236
Instructions for Sample Essay
Trespass to Land
“Pete Hates Football”

Objective:

To practice the legal-analysis pattern.

Procedure:

1. Read and analyze the call of the question. What is it asking you to do?

2. In the “law” column of the T-chart (see “Plan Your Answer”), write down the elements
of trespass to land. Then read the facts carefully, looking for the details of the events
taking place. Pay close attention to modifying words like adjectives and adverbs.
Also, the call of the question asked you what other intentional tort Pete could sue for.
So as you read, think about what other intentional tort might be present in this situation.
(Look for triggers.)

3. When details in the fact pattern “fit” one of the elements of trespass to land, make a
note on the exam question or in the “facts” column of the T-chart to remind yourself
which facts go with which elements. Also, write down the elements of the other cause
of action and which facts go with each element.

4. Fill in the template below. Insert all the elements, the rules that explain what those
elements mean, the facts that “fit” that element, the reasoning that explains why those
facts do or do not prove that element, and a conclusion on whether that element has or
has not been met. When you run out of elements, go to the C at the end. Have all the
elements been proved? Then the C is that Pete can win the trespass to land case. If an
element has not been proved on these facts, then the C is that Pete will lose. Then
begin a new issue with the second cause of action and repeat the ERFRC until you run
out of elements.

You will not fill out a template like this when you take the actual exam; the purpose of
the template is to get you used to organizing information in the legal-analysis pattern.

Here is how your answer should be organized:

State Issue I 1 point


Analyze rule on whether minors can be held liable 2 points
Conclusion 1 point
Elements/rules for trespass 8 points
Appropriate facts for each element/rule 5 points

237
Appropriate reasoning for each element/rule 5 points
Appropriate conclusion for each element/rule 5 points
Appropriate overall conclusion 1 point

State Issue II
Choose appropriate cause of action 1 point
Elements/rules 4 points
Appropriate facts for each element/rule 3 points
Appropriate reasoning for each element/rule 3 points
Appropriate conclusion for each element/rule 3 points
Appropriate overall conclusion 1 point

Points awarded: __________


Divided by 44 =

Percentage: _____________

238
Trespass to Land
Sample Essay Question
“Pete Hates Football”

Alan, age 11, lived in a house to the right of Pete’s house. Carr, age 12,
lived in a house to the left of Pete’s house. Alan and Carr were tossing a football
back and forth in Alan’s yard. To make the game more fun, Alan stayed in his
yard and Carr walked out to the street, down the street in front of Pete’s yard, and
up into his own yard. They threw the football back and forth across Pete’s yard.

Carr’s yard Pete’s yard Alan’s yard

Carr’s Pete’s Alan’s


house house house

Carr Alan

Street

Alan and Carr didn’t know that Pete was home. They threw the football
back and forth a few times, when suddenly Alan’s grip on the football slipped and
the ball flew toward Pete’s front porch. At that same moment, Pete walked out
onto the porch to pick up his newspaper. The ball hit Pete right in the face,
breaking Pete’s nose.

Pete is really annoyed at Alan and Carr and has come to you for advice. He
wants to know whether he can successfully sue the boys for trespass. Can he?
Is there any other intentional tort that he could win on against one or both of
the boys? Explain.

(Credit to Dean Timmer, who created this fact pattern for a previous exam.)

239
Plan Your Answer
Trespass to Land
“Pete Hates Football”

Facts
Law

240
Instructions for Sample Essay Question
“Here’s a Little Ditty ‘Bout Paula and Diane”

Objectives:
Understand how crucial analyzing the call of the question is.
Practice analyzing the call of the question.
Practice writing in the legal analysis format.

Procedure:
1. Read “Reading the Call of the Question”

2. On the next page, you will find an essay question, divided into two parts. First, read
and analyze the first call of the question. What is it asking you to do? What is the
problem you must solve? What kind of analysis should you do? If you are asked to
defend Diane, you will write only about the elements that Paula cannot prove. If you
are asked to give Paula’s arguments, you will have to analyze all the elements.
Which are you supposed to do?

3. In the “law” column of the T-chart (“Plan Your Answer”), write down the elements
and rules for the tort that is being asked about. Then read the facts carefully, sentence
by sentence, looking for the details of the events taking place. Pay attention to the
details of what is happening. The details that describe what is occurring are the
details you will use in your analysis.

4. When details “fit” one of the elements or rules for the tort you are analyzing, make a
note on the exam question or in the “facts” column of the T-chart to remind yourself
of which facts go with which elements.

5. Write out the answer. Be sure to use the ERFRC pattern, and be sure to answer the
question you were asked.

6. Now read the second call of the question. Again, for each element, find the facts that
“fit.”

7. Write out the second answer. Again, be sure to use the IRAC pattern, and be sure to
answer the question you were asked.

Here is how your answer should be organized:

Question 1:
Analyze only what the call of the question tells you to analyze 2 points
Issue 1 point
Element(s) 2 points
Rule(s) for each element you analyze 2 points
Fact(s) for each element you analyze 2 points

241
Reasoning for each element you analyze 4 points
Conclusion(s) 2 points
Ultimate conclusion (the answer to the question you were asked) 1 point

Question 2:
Analyze only what the call of the question tells you to analyze 2 points
Issue 1 point
First element, rule, facts, reasoning, conclusion 6 points
Second element, rule, facts, reasoning, conclusion 6 points
Third element, rule, facts, reasoning, conclusion 5 points
Ultimate conclusion (the answer to the question you were asked) 1 point

Points awarded: __________


Divided by 37 =

Percentage: _____________

242
Sample Essay Question
“Here’s a Little Ditty ‘Bout Paula and Diane”

Diane learned that her husband has been having an affair with a woman
named Paula. So Diane decided to kill Paula. To that end, Diane bought a
handgun and devised a plan to hide in Paula’s garage and shoot Paula when she
returned home from work one day. On the day of the planned shooting, Diane
placed her handgun in the glove box of her car and began driving to Paula’s house.
(1) If Paula were to discover Diane’s plan at this point and sue her for
battery, how could Diane argue that she is not liable?

The story continues. As she neared the house, Diane grew increasingly
preoccupied with her plan and ended up running a stop sign and hitting and killing
a pedestrian in the crosswalk. The pedestrian, it turned out, was Paula. Upon
discovering the identity of the pedestrian, Diane promptly celebrated. (2) Paula’s
estate sues Diane for battery. Discuss and decide.

243
Introduction to Law
Plan Your Answer
“Here’s a Little Ditty ‘Bout Paula and Diane”

Law Facts Facts


(Elements and rules) (Question 1) (Question 2)

244
Instructions for Sample Essay
“And the Food’s Bad, Too!”

Objective:
To practice the legal-analysis pattern

Procedure:
1. Read and analyze the call of the question. What is it asking you to do?

2. In the “law” column of the T-chart (see “Plan Your Answer”), begin by noting
the rule about whether minors can be held liable for intentional torts. Then
write down the elements of the intentional torts the question asks about. Then
read the facts carefully, looking for the details of the events taking place. Pay
close attention to modifying words like adjectives and adverbs.

3. When details in the fact pattern “fit” one of the elements of one of the torts,
make a note on the exam question or in the “facts” column of the T-chart to
remind yourself which facts go with which elements.

4. Write your answer. First address the rule about minors. Then write about one
tort at a time. If you wish, you could make a template to help you organize
your answer (label the page for issue, elements, rules, facts, reasoning,
conclusion, etc.) Please use full sentences. Insert all the elements for the first
tort, the rules that explain what those elements mean, the facts that “fit” that
element, the reasoning that explains why those facts do or do not prove that
element, and a conclusion on whether that element has or has not been met.
(ERFRC . . . C) When you run out of elements, go to the C at the end. Have
all the elements been proved on the first tort? Then the C is that Candace will
win. If an element has not been proved on these facts, then the C is that
Candace will lose. Then write the answer for the second tort.

Here is how your answer should be organized:

Issue 1 1 point
Elements, rules, facts, reasoning, conclusions 21 points

Issue 2 1 point
Elements, rules, facts, reasoning, conclusions 33 points

Points awarded: __________


Divided by 56 =
Percentage: _____________

245
Sample Essay Question
“And the Food’s Bad, Too!”

Candace was the tyrant of Bunkhouse 2 at Camp Wannagohome, and the other
eleven-year-old girls were sick of her. As Candace slept, Simone lifted up Candace’s
long braid and gestured with the scissors in her other hand as though she were going to
cut off the braid. The other girls stifled their giggles, but Simone didn’t actually cut off
the hair.
Later, as Candace continued to sleep, the other girls decided to row across the
lake to visit the boys’ camp. They tied Candace to her bunk with their macrame and
lanyard kits and pinned the door shut. Two hours later, they returned to find Candace
still sleeping peacefully. They took a Polaroid photo, pinned it above her bunk, and then
released her. When Candace awoke and found the picture, she was furious.

Candace’s parents filed a battery and false imprisonment suit on her behalf against
the campers. Discuss and decide.

246
Instructions for Sample Essay
“Good Sports Get Cheated”

Objective:
To practice the legal-analysis pattern

Procedure:

1. Read the call of the question. What is it asking you to do?

2. In the “law” column of your T-chart (“Plan Your Answer”), write down the rules
required to prove that an offer was made. (Hint: UCC 2-205 may be helpful here.)
Then carefully read the facts, looking for the details of the events taking place. Pay
particular attention to the modifying words.

3. When the details “fit” one of the rules, make a note on the exam question sheet or in
the “facts” column of the T-chart to remind yourself which facts go with which pieces
of law.

4. We did not give you a template this time. If you wish to use a template, you can write
one out.

5. Write out the answers to each call of the question from parts 1 and 2.

UCC 2-205. Firm Offers.


An offer by a merchant to buy or sell goods in a signed writing which by its terms gives
assurance that it will be held open is not revocable, for lack of consideration, during the
time stated or if no time is stated for a reasonable time, but in no event may such period
of irrevocability exceed three months; but any such term of assurance on a form supplied
by the offeree must be separately signed by the offeror.

247
Part #1
“Good Sports Get Cheated”

All Things Football, a wholesaler distributor of football equipment and clothing,


sent the following correspondence by mail, dated September 15, 2002. Good Sports, a
local sporting goods store, received the correspondence on September 16, 2002.

“All Things Football hereby offers to sell Good Sports, Inc. 500 synthetic
footballs, Item CHPFTBL, for $6.00 each. This offer will remain open until
November 15, 2002.

/s/
Ima Football Fan, President”

On October 1, 2002, All Things Football sent the following correspondence to


Good Sports, Inc by mail.

“Sorry, but our costs have increased on our synthetic footballs, Item
CHPFTBL, and we are unable to supply this item for $6.00. The new
price is $12.00 each. Thanks for your understanding!

/s/
Ima Football Fan, President”

On November 1, 2002, All Things Football received the following order from
Good Sports, Inc, by fax:

“Please ship 500 synthetic footballs, Item CHPFTBL, immediately.


Enclosed is $3,000.00 (500 x $6/football).”

After receiving the money, All Things Football shipped 250 footballs to Good
Sports, Inc, claiming the footballs were $12/football.

When Good Sports faxed its order on November 1, was there a valid offer, and if
so, was it for $6/football or $12/football? Please explain your answer in essay format.

Do not address the Mailbox rule or Acceptance in answering this essay.

248
Part #2
“Good Sports Get Cheated (Again)”
On September 16, 2002, Good Sports, Inc. received the following
correspondence, dated September 15, 2002, from All Things Football:

“All Things Football hereby offers to sell Good Sports, Inc. 500 synthetic
footballs, Item CHPFTBL, for $6.00 each. We will hold this offer open
for you until March 13, 2003, if you send us $50.00 now.

/s/
Ima Football Fan, President”

On September 18, 2002, Good Sports, Inc. sent All Things Football $50.00. All
Things Football received the money on September 19, 2002.

On October 1, 2002, All Things Football sent the following correspondence to


Good Sports, Inc by mail.

“Sorry, but our costs have increased on our synthetic footballs, Item
CHPFTBL, and we are unable to supply this item for $6.00 each. The
new price is $12.00 each. Thanks for your understanding!

/s/
Ima Football Fan, President”

On January 1, 2003, All Things Football received the following order from Good
Sports, Inc, by fax:

“Please ship 500 synthetic footballs, Item CHPFTBL, immediately.


Enclosed is $3,000.00 (500 x $6/football).”

After receiving the $3,000, All Things Football shipped 250 footballs to Good
Sports, Inc, claiming the footballs were $12/football.

When Good Sports faxed its order on January 1, was there a valid offer, and if so,
was it for $6/football or $12/football? Please explain your answer in essay format.

Do not address the Mailbox rule or Acceptance in answering this essay.

249
Instructions for Sample Essay
“A Swing and a Miss”

Objective:

To practice the legal-analysis pattern

Procedure:

1. Read and analyze the call of the question. What is it asking you to do?

2. In the “law” column of the T-chart (see “Plan Your Answer”), write down the
elements of battery. Then read the facts carefully, looking for the details of the
events taking place. Pay close attention to modifying words like adjectives
and adverbs.

3. Note that the T-chart has two “facts” columns. You can write the facts for each
defendant next to the elements, since they have each done something slightly
different. When details in the fact pattern “fit” one of the elements of battery,
make a note on the exam question or in the appropriate “facts” column of the T-
chart to remind yourself which facts go with which elements for each defendant.

4. If you wish, you can use the template below to help you organize your answer.
Please use full sentences. Insert all the elements, the rules that explain what
those elements mean, the facts that “fit” that element, the reasoning that explains
why those facts do or do not prove that element, and a conclusion on whether that
element has or has not been met. (ERFRC . . . C) When you run out of
elements, go to the C at the end. Have all the elements been proved? Then the
C is that Pia will win. If an element has not been proved on these facts, then the
C is that Pia will lose.

250
Sample Essay Question
Oakland Campus
“A Swing and a Miss”

Pia was curled up comfortably in her hammock. Dante, a jilted lover, saw
her and swung the hammock vigorously back and forth by grasping the rope that
led to the tree, despite Pia’s request that he stop.

After swinging the hammock for a minute or two, Dante left. Pia’s
neighbor, Dirk, who had a mean streak and thought it was amusing when Dante
was swinging Pia back and forth, came out and did the same. He was a large,
strong man, and he yanked the hammock rope forcefully. Pia fell from the
hammock to the ground.

Pia wants to sue both Dante and Dirk for battery. Discuss and decide.

251
Plan Your Answer
“A Swing and a Miss”

Law Facts Facts


(for Dante) (for Dirk)

252
“A Swing and a Miss”

You will not necessarily use all of the spaces in this template, depending on
how you “slice” the elements.

Issue:
E: element:

R: rule

F: facts

R: reasoning

Conclusion:

E: element

R: rule

F: facts

R: reasoning

253
Conclusion:

E: element

R: rule

F: facts

R: reasoning

Conclusion:

E: element

R: rule

F: facts

R: reasoning

Conclusion:

C:

254
Answer Guides

255
Issue guide
Planning the Answer to a Long Essay Question
Contracts

There may be some additional issues that are not listed here.

Bob’s first telephone call


1. Was the ad an offer?
2. Was it communicated?
3. Was Bob’s telephone call an acceptance?
4. Was Bob’s telephone call a rejection of Sandra’s offer?
5. Was Bob’s telephone call a counteroffer?
6. Did Sandra accept?
7. Did the counteroffer lapse?
8. Does the mailbox rule apply?
9. Did this all constitute a contract?
Bob’s second telephone call
10. If the ad was an offer, was Bob’s telephone call an offer?
11. Did Sandra accept?
12. Did Sandra and Bob have a contract?
13. Was there an option?
14. Did Sandra and Bob have an option contract?
15. Does it matter that the $20.00 was never paid?
16. How should the days be counted?
17. Did Bob have the right to buy the bottles when he got back from Las Vegas?
Carrie
18. If the ad was an offer, was Carrie’s letter an acceptance?
19. Was Carrie’s letter a counteroffer?
20. Does the mailbox rule apply to Carrie’s letter?
21. Did Sandra accept?
22. Did Sandra make a new offer or a counteroffer?
23. Did Sandra’s letter create an option?
24. Did Sandra’s letter create an option contract?
25. Was Sandra’s letter effective under the mailbox rule?
26. Did Sandra and Carrie have a contract?
Joe
27. Was the ad an offer, and when was it effective as to Joe?
28. If the ad was an offer, did Joe effectively accept?
29. If Joe did not effectively accept, did he make a counteroffer?
30. Did Sandra accept the counteroffer?
31. Does Sandra’s subjective intent to accept help Joe?
32. Did Sandra and Joe have a contract?

256
Issue Guide
Intentional Torts
“Take Me Out to the Ballgame”

Parties Tort Defenses

Chris v Macho Assault Did Chris see it coming?


Battery

Jim v Ugly Assault No present apparent ability


Ugly was walking away
Mere words are not enough

Macho v Buzz Assault Defense of Others

Buzz v Gene Battery Defense of Others

Chris v Doctor Battery Consent

257
Issue Guide
Contracts
“Sibling Rivalry”

Checklist for Essay Question


28 Issues @ 5 points apiece = 140 points

1. Define Offer (Students: Prof Cox requires his students


2. Define Acceptance to begin essay answers by defining all
3. Define Consideration these terms: other professors do not
4. Define Promissory Estoppel require this and do not want this.)
5. Define Moral Obligation
6. Define Material Benefit Rule
7. Does UCC apply?

Offer/Acceptance #1:

8. Over the years, did S make Offers to B?


9. Over the years, did B accept S’s offers?
10. Is consideration present: Is past consideration bargained for?
11. Do social promises between family members have value?
12. Do S and B have an enforceable contract for the lamp?

Offer/Acceptance #2:

13. Was B’s quote (3/1) an Offer?


14. In any case, did the quote lapse when B hung up the telephone?
15. Did S make an offer on 3/3?
16. Was S’s 3/3 Offer an option, an option contract, or a Firm Offer (UCC §2-205)?
17. How do you count the days?
18. On 3/6, did B accept S’s Offer or make a counteroffer? (UCC §2-207)?
19. Did B change a dickered term?
20. Did B use “proviso” language?
21. Is the contract “between merchants” or not “between merchants”?
22. Are the additional terms part of the contract or only proposals for addition to the
contract?
23. Under the mailbox rule, was B’s Acceptance effective when posted?
24. If S and B do have a contract, what is the consideration?
25. Do S and B have an enforceable contract for the lamp and the clock?

If S were a merchant:

26. Was S’s 3/3 Offer an option or a Firm Offer (UCC §2-205)?
27. Would the mailbox rule apply to B’s 3/6 Acceptance?
28. Do S and B have an enforceable contract for the lamp and clock?

258
Issue Guide
Intentional Torts
“Ray, Jill and the Hot-Dog Cart!”

Battery Defenses Implied consent


Emergency doctrine

Conversion Defense Private necessity

Trespass to Chattels

259
Sample Answer
Problem 2

The question is whether Francis is entitled to the car.

To decide whether a person has the intent to contract, courts use an objective test.
They look at the person’s outward expressions of intent and impute the intent that
corresponds to the reasonable meaning of the person’s words or acts. Secret, undisclosed
intent is irrelevant. Here John (J) bought a new car for $15,000, but he told Francis (F)
that he would sell it for $1,500 because the car had stranded him on the first day. Even
though J was mad at the car, which might make him do something rash, a reasonable
person would not think this was an offer because nobody would be willing to take a
$13,500 loss after owning the car for just one day. Furthermore, all this took place at a
bar, and J was drinking. Though people do sometimes do serious business deals in bars
while drinking, this is another factor that would tend to show that he didn’t mean it,
because offers in bars are more likely to be jokes. Finally, everybody in the bar laughed
when F ran out of the bar. This shows that everybody in the bar knew J wasn’t serious.
Even though the “reasonable” person doesn’t mean anybody in particular, the fact that
everybody in the bar knew that J wasn’t serious shows that the reasonable person
wouldn’t think so either. So the rule is almost certainly satisfied.

There is an exception to the rule, which states that the court will look at a person’s
subjective intent if the other party actually knew that person’s subjective intent, even if a
reasonable person would not know. Here, F was “feeble” and had a very low I.Q. This
shows that he would less likely be able to understand that J was pulling his leg – he
would be more likely to take J literally. Furthermore, he went and got the car, had it
towed to his home, and got and gave J the $1,500. Because he did everything as J
suggested, and because he put himself through a lot of trouble, he must have thought that
J was serious. Otherwise, he wouldn’t have done all that. So the exception doesn’t seem
to apply.

[The call of the question is whether F would get the car, so if you stuck to the
call of the question, you might be able to stop here. Contracts professors tend to
take this a step further, however. To show that other issues could lurk in these facts,
they often engage students in a discussion of whether F can get anything. So you
could continue the answer in the following way.] On the other hand, J may well have
known that F thought J was serious. Here, the facts say that F was “known by all to have
a very low I.Q.” Furthermore, F knew J by name. So J is probably one of the people who
knew that F had a low I.Q. because they obviously knew each other. So if J knew that F
thought J was serious, J could bear some responsibility for the expectations that he
created in F’s mind that F relied on. Even though no court would award the car to F, F
would probably be able to get something. [Discussion of possible damages is omitted.]

F is not entitled to the car.

260
Intro to Law
Answers to Multiple Choice Questions

Please remember this: these materials were taken from publications that were not
written by your professor. If you see any analysis here that conflicts with what your
professor told you, follow what your professor told you.

Earlier in the semester, we discussed how to write an essay exam. The pattern
that you were to follow was to first write out the law (the rule or rules, definitions,
subrules, etc.), then state the facts that are relevant under that law, and then
explain why the facts might, or might not, fulfill that law (the because/therefore
part of the analysis). As you read these answers, please note that they follow that
pattern. The point is that all legal analysis follows that pattern.

1. The answer is C. In this question, it was easy to spot the concepts being tested:
the options told you that the concepts were assault and battery. Assault is when
the defendant intends to cause either (1)(a) contact or (b) apprehension of contact,
and (2) defendant does something (3) that in fact causes the plaintiff to apprehend
that a harmful or offensive contact is about to occur. Here, Dan intended to
swerve his car at Pete so that he could frighten Pete. Therefore, Dan intended for
Pete to apprehend a contact, and Pete did apprehend that a harmful contact was
about to occur. Therefore, Dan committed an assault.

(Please note: for battery, there is a split among professors about what intent the
defendant has to have: some professors say that the defendant has to intend a
harmful or offensive contact or apprehension of a harmful or offensive contact.
Other professors say that defendant need only intend to make a contact or cause
apprehension of a contact, and the contact has to turn out to be harmful or
offensive; the defendant need not intend to harm or offend. So adjust your
analysis accordingly. For this fact pattern, it wouldn’t matter which view you use
because Dan did intend to cause Pete to feel apprehension of a harmful or
offensive contact because he wanted to frighten Pete by swerving his car at Pete.
That would be enough under either view.) Battery is when (1)(a) the defendant
intends to cause a [harmful or offensive] contact or (b) a reasonable person in
defendant’s position would realize that a [harmful or offensive] contact was
substantially certain to occur or (c) defendant intends to cause apprehension of
[harmful or offensive] contact (transferred intent), and (2) defendant does
something that in fact causes the harmful or offensive contact. Here, even if Dan
didn’t intend to cause a contact and even if a reasonable person wouldn’t realize
that a contact was substantially certain to occur, the facts say that Dan intended to
frighten Pete. Therefore, at the very least, he intended for Pete to apprehend a
contact. When Pete swerved to avoid Dan’s car, Pete ended up suffering a
harmful contact because his car went into the ditch, Pete’s body suffered a violent
contact with the inside of his car, and that contact injured him. Therefore, Dan
can be liable for both assault and battery.

261
2. The answer is B. This question tested the concept of self-defense. Self-defense is
a privilege to use reasonable force to protect oneself against a threatened tortious
contact or confinement. There is a split among the states: some don’t require
retreat before using deadly force; others hold that it is never reasonable to use
deadly force when it is reasonably safe to retreat. But even in the states that don’t
require retreat, whether Dave has the privilege is likely to depend on whether he
knew or should have known that he could safely retreat because that will show
whether the amount of force used was reasonable. If Dave could not have known
that he could safely and easily retreat, the reasonable person in his situation might
have considered it necessary to use a knife because of the extreme danger facing a
person who has a pistol pointed at him. On the other hand, for self-defense,
“reasonable force” means the force that would have appeared necessary to a
reasonable person in Dave’s situation. If Dave knew or should have known that
he could safely and easily retreat without sustaining harm, the reasonable person
in his situation would probably not have considered it necessary to use any force
at all in self-defense. So B makes it possible for Pat to win the case.

A doesn’t make it possible for Pat to win the case because even if Dave was using
deadly force, Pat was using deadly force, too. Deadly force may be reasonable in
self-defense if the person using it is threatened by what reasonably appears to be
deadly force. Since Pat was threatening Dave with a pistol, Dave’s using a knife
– even if it was deadly force – may have been privileged in self-defense. So A
isn’t likely to help Pat. C isn’t likely to help Pat either. The initial aggressor
usually isn’t privileged to use self-defense. However, if the person the aggressor
is threatening responds by using excessive force and escalating the danger of the
situation, the initial aggressor regains the privilege and can defend himself against
the excessive force. So if Dave initiated the aggression by slapping Pat, then Pat
was privileged to use reasonable force against the possibility that Dave would slap
him again. Thus, if the force with which Pat threatened Dave was reasonable, it
would have been privileged, and, therefore, not tortious. Then Dave would not
have been privileged to use force to defend himself against Pat. That is the
general rule: an initial aggressor is not privileged to use force to defend himself
against a reasonable response to his aggression. However, this case involves an
exception to the general rule. If Pat’s using a pistol was unreasonable (that is,
excessive force), then Pat’s actions were not privileged by self-defense. At that
point, the contact that Pat was threatening Dave with would have been tortious,
and Dave would have been privileged to defend himself against it by using
reasonable force. And because the force that he was defending against was
deadly force (a pistol), he would be privileged to use deadly force (a knife). So
even if Dave was the initial aggressor, he might have been privileged to use the
knife to defend himself against Pat’s using a gun. So that’s why C is incorrect.
Finally, D is also incorrect. It doesn’t matter what Pat knew. It only matters
what Dave knew. Reasonable force depends on what the reasonable person in the
defendant’s position would have considered necessary, so Dave’s privilege
depends on how Dave perceived or should have perceived the threat that Pat was

262
confronting him with. Even if Pat knew that the pistol was not loaded, Dave’s
using a knife might have been reasonable if Dave believed that the pistol was
loaded. So D won’t help Pat.

3. The answer is D. The concept being tested here is assault. Assault is when the
defendant intends to cause either (1)(a) contact or (b) apprehension of contact,
and (2) defendant does something (3) that in fact causes the plaintiff to apprehend
that a harmful or offensive contact is about to happen to her. The point is that the
plaintiff has to believe that something is about to happen to her, not to someone
else. Here, Joan had no reason to believe that Galen was going to do anything to
her. Therefore, there’s no assault.

All four options, A, B, C, and D, contain the word “because.” Therefore, you
must evaluate the statement that comes after the word “because” and decide
whether it is factually and legally true. Then you must see whether that statement
can lead to, or justify, the answer in front of the word “because.” D is factually
true (it doesn’t say any legal stuff, so you don’t have to decide whether it’s legally
true); and the facts in D will justify the answer – that Galen will win. C is
factually true, but it is incorrect because those facts don’t justify the answer. It
doesn’t matter whether Joan saw Galen do anything to Betty. Joan could only
claim assault if she saw Galen about to do something to her (Joan). A and B are
wrong for the same reason that C is wrong. A is factually true: Galen did know
that Joan would be frightened. However, those facts won’t justsify the answer
because assault requires more than just wanting to frighten someone. Assault
requires the victim to apprehend an immediate threat to herself. B doesn’t make
sense – it is legally and factually untrue – because to transfer intent, defendant has
to have intended to do something to one person and then ended up doing
something to someone else (for example, defendant intends to hit A, but he misses
and hits B instead). Here, Galen never intended to do anything to Betty, so there
was no intent for Galen to transfer to someone else.

4. The answer is B. The concepts being tested here are self-defense, trespass to
land, and defense of property. Darcy was not entitled to use deadly force to
protect his land, but he was probably entitled to use deadly force to protect
himself. Self-defense is a privilege to use reasonable force to defend oneself
against a threatened contact. Reasonable force is the amount of force that would
appear necessary to a reasonable person under the circumstances. Generally,
courts hold that it is reasonable to use deadly force to defend against what
reasonably appears to be a threat of deadly force. B states that Darcy will win if
Darcy fired at Pat to defend himself against Pat’s threat with the shotgun.
Because B contains the word “if,” you have to assume that the part after the “if” is
true. Therefore, Pat used deadly force to defend himself against deadly force (the
shotgun that was pointed at him). Courts generally hold that that is reasonable;
therefore, B is correct.

263
A contains the word “if,” so you must assume that Darcy did fire “at Pat to defend
his land against a trespass.” Then you must ask yourself whether that fact will
make Darcy win the case. A is incorrect because it is never reasonable to use
deadly force for the sole purpose of preventing a trespass to land or chattel. C is
factually true; however, C is incorrect because the reason given won’t justify the
answer given. The fact that Darcy struck the first blow won’t make Pat win:
Darcy’s initial use of gentle force (Darcy “gently pushed Pat backward”) was
privileged in defense of property, making Pat’s response to Darcy unprivileged
and a threatened battery. D is also factually true; however, D is incorrect because
the fact that Pat was already on the land won’t make Pat win. Pat’s trespass did
not end once he had entered the land. The trespass continued for as long as he
refused to leave. So Darcy was privileged to use reasonable force to defend his
property.

5. The answer is C. The concepts being tested are trespass, implied consent, and
necessity. If, as stated in C, MC had no reason to anticipate that the tests would
cause any of the results that occurred, then MC did not intend to commit the act
constituting trespass. Without intent, there is no cause of action for trespass. A
prima facie case for trespass to land consists of (1) a physical invasion – entry –
of the plaintiff's land by the defendant, without the plaintiff’s permission; (2)
defendant's intent to bring about a physical invasion of the plaintiff's land; and (3)
causation. The intent required is not intent to trespass; thus, mistake as to the
lawfulness of an entry onto another's land is not a defense if defendant intended
the entry onto that particular piece of land. But what is required is the intent to do
the act that constitutes the trespass. Here, MC, in firing the rocket engine, caused
debris to fall onto Homesteader's property. This would be a sufficient physical
invasion for trespass. However, trespass also requires that MC intended to send
this debris onto Homesteader's land. If (as stated in C) MC had no reason to
anticipate that its rocket engine tests would cause the debris to fall onto
Homesteader's property, then MC did not intend to make any entry onto
Homesteader's property (that is, MC acted neither with the goal of sending debris
onto Homesteader's land nor did it know with substantial certainty that such a
consequence would result from its tests of the engine). Therefore, under the
circumstances in C, the element of intent would be missing, so Homesteader
could not show a prima facie case of trespass.

A is wrong because the fact that Homesteader bought the farm knowing that MC
used the adjoining land for engine tests will not allow MC to commit an act that
would otherwise be trespass. The fact that Homesteader knew how MC used the
adjoining property cannot be taken as implied consent to MC's scattering debris
on Homesteader's land because consent requires willingness. Homesteader’s
being willing to live next door to the site doesn’t mean that Homesteader is
willing to have his farm pelted by debris. B is incorrect because a physical
invasion does not require that the defendant personally come onto the land. There
is a trespass if, for example, the defendant floods the plaintiff's land or, as here,
causes debris to settle on the plaintiff's land. D is wrong because the defense of

264
public necessity arises where the public good is threatened with injury, and the
defendant's actions are reasonably and apparently necessary to avoid such injury.
Also, the threatened injury must be substantially more serious than whatever the
defendant did to interfere with the plaintiff's property so that the defendant could
avoid the threatened injury. This defense requires a situation in which immediate
action is required so that the defendant can ward off an imminent threat to the
public good. Testing rocket engines for eventual military use does not rise to this
level because it was not necessary to avoid an impending injury to the public
good. Thus, MC cannot successfully claim that its conduct is privileged as a
public necessity.

6. The answer is B. The concept being tested is ultra-hazardous activity. MC's


exercise of due care will be least helpful to MC. An activity is characterized as
ultra-hazardous or abnormally dangerous if it (1) involves a risk of serious harm
to persons or property; (2) cannot be performed without risk of serious harm no
matter how much care is taken; and (3) is not a commonly engaged-in activity by
persons in the community. When defendant engages in an ultra-hazardous
activity, defendant owes an absolute duty to make the activity safe, and defendant
will be liable for any injuries to persons or property resulting from the activity.
The argument raised in B will not help MC because, though due care and caution
would be relevant in a case based on negligence (which alleges violation of a duty
to act as an ordinary, reasonable person), those factors are irrelevant to an action
based on the defendant's having engaged in an ultra-hazardous activity, for which
there is strict liability. Therefore, for an ultra-hazardous activity, what's relevant
is whether the defendant has breached the absolute duty to make the activity safe,
not whether the defendant has exercised reasonable care.

A is incorrect because the absolute duty to make the activity safe is owed only to
foreseeable plaintiffs, that is, those plaintiffs to whom a reasonable person would
have foreseen a risk of harm under the circumstances. Therefore, if
Homesteader's farm is so far from the test site that a reasonable person would not
have foreseen a risk of the harm that occurred, then Homesteader would not be a
foreseeable plaintiff, and MC would not owe him the absolute duty to make the
engine tests safe. So because A would be helpful to MC, A is incorrect. C also
sets out an argument that would help MC. If MC's tests did not pose a risk of
serious harm to either Homesteader or his property, then the first requirement for
finding that the activity is ultra-hazardous would be missing. D would also be
helpful to MC. The fact that similar tests were carried out in the same locale by
other companies would indicate that MC's testing was a more commonly
engaged-in activity in that area; therefore, the activity would be less likely to be
found ultra-hazardous.

7. The answer is B. The concept being tested is negligence. The distance from the
test site to Homesteader's farm is most helpful to MC. When an action is based
on negligence, the plaintiff is alleging that the defendant has breached its duty of
acting as an ordinary, prudent, reasonable person, and that the breach has actually

265
and proximately caused injury to the plaintiff. No duty is imposed upon a person
to take precautions against events that cannot reasonably be foreseen. Thus, if at
the time of the defendant's conduct, the defendant's act does not create a
foreseeable risk of injury to a person in plaintiff's position, then there's no duty.
If, as B states, Homesteader's farm is so far from the test site that no risk to
Homesteader was foreseeable, then there would be no duty imposed upon MC to
take precautions against damage that could not reasonably have been foreseen.
Thus, MC would not have a duty of reasonable care toward Homesteader, so there
could be no negligence.

A is wrong because the prior instability of the subsurface earth structures will
almost certainly not help MC. That instability would tend to show a lack of
causation. But even assuming that the subsurface structures were unstable, the
facts say that the structures would not have collapsed but for the firing of the
rocket engine. Therefore, MC's conduct was the cause in fact (the actual cause)
of the damage to the water well on Homesteader's farm. Firing the rocket was
also the proximate cause of Homesteader's injury because the damage was the
direct result of MC's conduct. Therefore, it will not help MC to show that the
subsurface structures were already unstable. C is incorrect because, even if MC
exercised reasonable care in selecting the personnel involved in the testing, the
personnel may still have acted negligently in the way in which they conducted the
tests or chose the test site. Then, MC would be vicariously liable for its
employees' negligence because the negligence would have been committed within
the scope of the employment relationship. D is wrong because industry standards
of conduct do not establish a conclusive test for determining whether specific
actions constitute a breach of duty owed to someone. Industry standards are
admissible as evidence of an appropriate standard of care, but they are not
conclusive (after all, it could be that everyone in the industry is acting
negligently).

8. The answer is C. The concept being tested is negligence – specifically,


foreseeability. Daphne is not liable for the damage to the car because a
reasonable person would not have foreseen damage arising from the delay in
getting the pizza. A prima facie case for negligence requires (1) that defendant
had a duty to conform to a specific standard of conduct to protect the plaintiff
against an unreasonable risk of injury; (2) defendant's breach of that duty; (3) that
the breach was the actual and proximate cause of the plaintiff's injury; and (4)
damage. No duty is imposed upon a person to take precautions against events that
cannot reasonably be foreseen. Here, it is true that if Daphne had gotten the pizza
immediately, Paula's car would not have been where it was when Tammy ran into
it. However, mere delay in picking up a pizza while leaving a car properly parked
does not create a foreseeable risk of damage to the car. Therefore, there is no
duty and no breach of that duty. Note that C contains the word “because.” The
statement after the “because” is factually and legally true, and that statement
justifies the answer (that Daphne will win). Therefore, C is the correct answer.

266
B is wrong because the mere fact that Paula's car would not have been in that spot
if Daphne had gotten the pizza immediately is not enough to show negligence.
The damage to Paula's car was unforeseeable under these circumstances (after all,
the car was properly parked in the parking lot, not parked in the middle of the
freeway; therefore, it was unforeseeable that spending a few extra minutes would
cause a risk that hadn’t been there before). Therefore, Daphne did not act
negligently when she did not pick up the pizza immediately. Though the
statement in B is true factually and legally, it doesn’t justify Paula’s winning the
case. Likewise, A is wrong because though it is factually and legally true, it
doesn’t justify Paula’s winning the case. The mere fact that Daphne stopped at
the bookstore, and that Paula had loaned her the car specifically to pick up the
pizza, will not mean that Daphne acted in a way that created an unreasonable risk
of damage to the car. The concept of "exceeding one's authority" might be
relevant when an employer is trying to avoid liability for an employee's tortious
conduct by arguing that the employee acted outside the scope of her employment.
However, Daphne is not an employee. D misstates the family car doctrine;
therefore, it is legally untrue. The doctrine states that an automobile owner is
liable for the tortious conduct of immediate family or household members who
are driving with the owner's permission. So this doctrine could not relieve
Daphne of any liability for negligence. The doctrine would not make Paula liable
instead; all the doctrine does is make the owner liable also. And the doctrine
can't apply anyway, since Daphne and Paula are not related and do not live in the
same household.

9. The answer is D. The concept being tested is negligence – specifically, the duty
owed to a person who is on the defendant’s premises. Because Bill had Carol's
permission to be on Carol's land, Bill is a licensee. A licensee is a person who
enters land with the owner's permission, for his own purpose or business rather
than for the owner's benefit. The owner or occupier of land has a duty to warn a
licensee of a dangerous condition known to the owner or occupier when the
condition creates an unreasonable risk of harm to the licensee and when the
licensee is unlikely to discover the condition. However, the owner or occupier
has no duty to inspect for defects nor to repair known defects. Bill was a licensee
because he entered Carol's land with her permission for his own purpose
(retrieving his wallet) rather than for any benefit of Carol's. The exposed electric
wire created an unreasonable risk of death or serious injury to Bill as he searched
the yard. If Carol knew of the presence and condition of the wire, she should also
have known that it posed an unreasonable risk of harm to Bill and that, because
the wire was obscured by the bushes, Bill was unlikely to discover the wire.
Thus, Carol would have a duty to warn Bill about the wire. (Remember, D
contains the word “unless.” So you must first decide whether the facts above
question 10, by themselves, would mean that Carol isn’t liable. Using just those
facts, she wouldn’t be, because there would be no breach of any duty. Then you
must assume that the facts after the “unless” are true, and ask yourself whether
adding those facts would make Carol liable, and that would be the only way that
she could be liable. Here, the only way that Carol could be liable to a licensee is

267
if (1) she knew of the duty and (2) failed to warn. D tells you to assume that both
of those are true. Therefore, that would make Carol liable.)

A accurately describes the duty that Carol owed to Bill. However, A is wrong
because the mere fact that Carol gave Bill permission to enter her land will not
make her liable for his injuries there. Carol is not strictly liable for injuries to a
licensee; she is only liable for injuries caused by a breach of her duty to warn of
dangerous conditions that she knows about and that the licensee is unlikely to
discover. A would make Carol liable even if Carol did not know about the wire.
Therefore, even though A is factually and legally true, it won’t justify making
Carol liable. B is incorrect because an owner of land does not owe a duty to a
licensee to repair defects or dangerous conditions. So even though B is factually
true, it won’t justify making Carol liable. Likewise, C is incorrect because the
owner of land has no duty to a licensee to inspect for defects or dangerous
conditions. So even when you assume that Carol, in fact, “failed to reasonably
inspect the property and, as a result, was unaware of the dangerous condition of
the wire,” that can’t make her liable because she didn’t have any duty to do any of
that.

10. The answer is C. This question tests two concepts: necessity and the difference
between negligence and intentional torts. Here, Bill is liable for the damage to the
flowers even if he had a privilege to enter Carol's yard. When a person has the
privilege of necessity, the person may interfere with the property of another (1) if
it is reasonably and apparently necessary to do so to avoid threatened injury from
a natural or other force and (2) if the threatened injury is substantially more
serious than the interference with property that the defendant is committing so
that he can avoid the threatened injury. If a case involves private necessity (that
is, the act is solely to protect one person from serious injury, rather than the public
as a whole), the defense is qualified, so that the actor must pay for any damage he
causes. Here, Bill was faced with death or serious injury when the truck almost
hit him. Apparently, the only way to avoid this injury was to jump into Carol's
yard. The threatened injury to Bill was substantially more serious than Bill's
entry into Carol's yard. Thus, Bill was privileged to enter the yard. However,
because Bill acted solely to protect himself, he may be required to pay for the
damage he caused to the flowers. (Note that this is a situation where the correct
answer is one that seems kind of unappealing. C says only that Bill can be held
liable for the damage, but it doesn’t say why. Nevertheless, it has to be the right
answer because the rest of the options have something seriously wrong with
them.)

A correctly states that Bill is liable for the damage, but incorrectly states that he
was not privileged to go onto Carol's land (so A is not legally true). On the other
hand, D is wrong because it concludes that Bill's privilege absolves him of
liability for the damage he caused (so though the statement in D is legally true –
Bill did have a privilege – it doesn’t justify the answer given – that is, that he
won’t have to pay). B is incorrect because Bill's exercise of due care is irrelevant.

268
Carol will be suing Bill on a theory of intentional tort, such as trespass. Although
Bill's actions were privileged, he did intend to perform the act (jumping off the
road) that damaged the flowers. Due care is a concept that applies in negligence
cases, but it does not apply to intentional torts. Therefore, Bill's liability does not
depend on whether he was exercising due care.

11. The answer is A. The concept being tested is negligence – specifically, the duty
owed to a person on the defendant’s premises. If the banana peel had been on the
floor for an hour, the grocery store, through its employees, would have had a
reasonable amount of time to discover it and remove it. Gloria was an invitee, so
the store had the duty to warn her of concealed dangerous conditions known to the
store, as well as the duty to make reasonable inspections to discover dangerous
conditions and make them safe. Because A makes you assume that the banana
peel was on the floor for an hour, the banana peel's presence should certainly have
been discovered through reasonable inspections by store employees, and someone
should have picked up the banana peel to remove the danger of someone’s
slipping and falling. So failure to discover the banana peel within an hour would
breach the duty that the store owed to Gloria. This breach actually and
proximately caused Gloria to fall and hurt herself. Therefore, under the
circumstances in A, the store would be liable for Gloria's injuries.

B is incorrect because it implies that the store can be liable without fault. Mere
presence of the banana peel on the floor will not make the store liable; liability
requires that the store was somehow at fault for the banana peel being on the floor
— for example, that a store employee had thrown the peel on the floor or that the
store employees had not reasonably inspected the premises. So even though the
statement in B is factually true, it doesn’t justify the answer given – that Gloria
should win. C is wrong because the store's liability does not depend on whether
an employee put the banana peel on the floor. If, in the exercise of reasonable
care, a store employee should have discovered and removed the banana peel, the
store will be liable, regardless of who put the peel on the floor. And remember —
the choice says "no, unless a store employee knocked the banana peel on the
floor." So the only way this pick can be correct is if (1) given only the facts in the
fact pattern, the answer would be "no," but (2) if you assume that an employee in
fact knocked the banana peel on the floor, (3) then the answer would be "yes,"
and (4) the only way that the answer can change to "yes" (that is, the only way for
Gloria to win) is if an employee knocked the banana peel on the floor. Because it
doesn't matter who knocked the banana peel on the floor, C has to be wrong. D is
wrong because the store can be liable even if none of its employees actually knew
that the banana peel was on the floor. If there was a reasonable time to discover
the banana peel and no one discovered it, or if no one ever even inspected, then
the store will be liable. Again, the choice says "no, unless a store employee knew
that the banana peel was on the floor." This pick can only be correct if (1) given
only the facts in the fact pattern, the answer would be "no," but (2) if you assume
that an employee actually knew that the banana peel was on the floor, (3) then the
answer would change to "yes," and (4) the only way that the answer can change to

269
"yes" (that is, the only way for Gloria to win) is if an employee actually knew.
But no one has to actually know for the answer to be "yes." Therefore, D is
wrong.

12. The answer is D. This question requires a multi-step analysis. It tests on how the
concepts of intoxication, burglary, and felony murder are related. It also tests
your ability to read and understand the statutes explained in the fact pattern.
First, this jurisdiction’s murder statute says that murder can occur in two ways:
(a) premeditated and intentional or (b) committed during another crime (this is
what’s known as felony murder). Jesse could argue that he did not commit the
first kind of murder because he did not intend to kill the MP (and he could also
argue that he did not act with premeditation). However, if the killing occurred
during a burglary, it could also be murder under the second part of this
jurisdiction's statute. Second, burglary is a specific intent crime; therefore, it
requires a specific intent to commit a crime. Intoxication might negate the
existence of that intent (so it would qualify as a defense under this jurisdiction’s
intoxication statute). So if Jesse did not have the specific intent necessary for
burglary, then he could not have been committing a burglary, and the killing
could not have been under the felony-murder part of the statute. Thus, third, if
Jesse was too drunk to intend to commit burglary, then the only way Jesse could
be convicted of murder is if he acted intentionally or with premeditation because
the felony-murder part of the statute couldn’t apply. D is correct because it sets
out this analysis.

A is incorrect because voluntary intoxication is only a defense when the


intoxication negates intent. A has nothing to do with intent; it only refers to
causation. B is wrong because it states the wrong standard of proof: the
prosecution must prove all elements of the crime beyond a reasonable doubt. C
is wrong because voluntary intoxication may be a defense to the felony-murder
definition of murder.

13. The answer is D. The concepts being tested are (1) attempt and (2) the difference
between, and the legal effect of, mistake of fact and mistake of law. All four
choices in this question have to do with attempt. To convict a defendant for an
attempt, the prosecutor must show that the defendant had the specific intent to
commit the crime and that the defendant engaged in behavior that came very close
to completing the crime (beyond mere preparation). In A, Walter could be found
guilty of attempted murder. He had the specific intent to kill, and the jury could
find that he came close to committing the crime. If it weren’t for his mistake of
fact – that is, whether anyone was in the house – he would have succeeded. And
mistake of fact is not a defense. In B, Yvette could be found guilty of attempting
to obtain property by false pretenses. She had the specific intent to commit the
crime, and she came very close to completing it. (There’s no mistake of fact –
Yvette was kept from succeeding only because the store clerk checked out the
credit card, and the card had been reported stolen, so Yvette wasn’t mistaken
about anything.) In C, Hazel could be found guilty of attempted robbery. She

270
had the specific intent, and the jury could find the necessary proximity. Again,
her mistake of fact – she thought that the people had money, but they didn’t – is
the only thing that prevented her from carrying out the crime. But in D, Farley
could not be guilty of an attempt. Since the activity he was engaging in, and
intending to engage in, was not illegal, he cannot be found guilty of attempt even
if he thought that he was engaging in illegal conduct. This is the doctrine of legal
impossibility, which (unlike factual impossibility) is a defense.

14. The answer is C. False imprisonment requires that the defendant intentionally
confined the plaintiff. The plaintiff is confined when his will to leave a place with
fixed boundaries is overcome in a way that would similarly overcome the will of a
reasonable person in the plaintiff’s situation. Since Rezzie was not prevented from
leaving, he was not confined (that is, he was not imprisoned).

In a false imprisonment case, “intent” means a desire or knowledge that the


defendant’s act will result in a confinement of the plaintiff. (B) contains the wrong
definition of intent. The intent required is not that the defendant knew that the
plaintiff’s rights were being violated. Thus, if the acts of Global’s employees had
resulted in a confinement of Rezzie, the fact that they did not know that they were
violating Rezzie’s rights would not provide them with an effective defense. (A) is,
therefore, wrong. (B) is wrong because “unreasonableness” is not an element of
false imprisonment, so it doesn’t matter whether Global can prove that or not. Even
if it is true that Global’s employees were reasonable, it is not relevant. (D) is wrong
because damage is not an element of false imprisonment, so even it if is true that
Rezzie suffered no damage, it isn’t relevant.

15. The answer is A. The element (A) states – the extreme and outrageous conduct – is
the linchpin of an emotional distress claim. In fact, where creditors are concerned,
a pattern of abuse, hounding, and extreme conduct can form the basis for an
emotional distress claim even if the plaintiff really owes the money! (This is why
(D) is wrong.) And because (A) contains an “if,” it requires you to take it as true
that the behavior was extreme and outrageous. (B) is wrong because it addresses an
element that isn’t part of an emotional distress claim. Even if going onto Poe’s
property contributed to the outrageousness, it isn’t necessary. The same behavior
would be outrageous even if it happened in a restaurant. (C) isn’t the best answer
because physical injury is not absolutely required. The damage can consist of
severe emotional distress beyond what the reasonable person could take. Because
this answer requires you to find that the only way Poe can win is to suffer physical
harm, it’s not the best answer.

16. The answer is B. Here, the element that is most in question is intent, and (B)
requires you to take as true that Poe knew with substantial certainty that the door
would contact something connected to Kane’s person – the bullhorn. (A) is a good
example of a pick that is true but not relevant. For one thing, it does not address the

271
central issue in the battery case: intent. Also, asking a person to leave is not an
element of battery, so it is not relevant. (C) is also wrong. “Extreme and
outrageous” is an element of IIED, not of battery, so this pick is not relevant to
what is being asked. (D) is wrong: it states that Kane shouldn’t win because Poe
doesn’t owe the money. But, first of all, it’s not necessarily true that Poe doesn’t
owe the money. Second, owing or not owing money has nothing to do with any
element of battery, which is what the question asks about.

17. The answer is A. (A) is correct because Gruner committed all the elements of
battery – he intended to cause the contact with Skipper because he kept throwing
rock after rock, coming closer and closer, until he finally achieved his objective of
hitting Skipper. There was a contact, and it was at least offensive, and harmful, too
– rocks are hard and heavy, so at the very least, getting hit by a rock causes pain.
(B) is wrong because an element is missing: assault requires that the victim
apprehend an imminent contact. Here, Skipper was asleep, so he did not know that
Gruner was throwing rocks at him, so that element is missing. (C) is wrong because
a minor can be liable if the minor in fact had the intent required, which Gruner did.
He wanted to hit Skipper with the rock. (D) is wrong because Skipper doesn’t have
to sue Gruner’s parents to win. Skipper can just sue Gruner.

18. The answer is B. A battery claim requires that defendant intended to cause a
harmful or offensive contact with another person. The intent can be proved either
by showing that defendant’s purpose was to cause that contact, or that defendant
knew that the contact would result. Battery also requires that defendant’s act
caused a contact to occur, and that the contact was harmful or offensive. (B) is the
right answer because the question asks which of the picks contains elements of
battery, and (B) is the only one that does. (A) says that Customer would have to
prove severe bodily harm, but that is not an element. Similarly, (C) says that
Customer would have to prove severe emotional distress, but that is not an element
of battery either. (D) is not right because as long as the Store intended to cause the
chemical to spray anybody shaking the door handle, it will be liable. And Store
must have intended to cause the contact because the whole point of the mechanism
was to spray anybody who did shake the handle. The fact that Customer shook the
door handle would not change the Store’s intent to cause the harmful contact.

19. The answer is D. False imprisonment requires that defendant intentionally and
unlawfully confined the plaintiff within boundaries. (D) is right because it points
out the missing element: intent. For Store’s employees to intend to confine
Customer, they would have to (1) have locked Customer in the store on purpose or
(2) know that Customer was likely to be locked in. Here, Store’s employees didn’t
know that Customer was even there, so they could not have had the intent. (A) and
(B) are both true, but not relevant. Customer could not get out, and he had no
reasonable avenue of escape, but he can’t win unless he can show intent. (C) is
wrong because injury is not required if a person is aware of the confinement.

272
20. The answer is A. This is the only choice that gives Carr any chance to win. Assault
is an act that creates an apprehension of imminent battery, and the defendant must
act with the intent to cause that apprehension. So the defendant must act either with
the purpose of causing apprehension, or he must act knowing that apprehension will
result. Here, the students knew that the subjects of this experiment sometimes
reacted with shock and fear, which could be enough to show intent. Furthermore,
Carr said that he thought he was “under attack,” so the students’ actions caused him
to believe that he could be harmed. (B) is wrong because Carr suffered no contact.
(C) is wrong because conversion requires that the plaintiff’s property be taken or
harmed so severely that the defendant should have to pay for it. Here, Carr’s car
was not harmed, and it was not taken away. (D) is wrong because “restraint”
requires that the plaintiff believed that there was no reasonable avenue of escape.
Here, Carr knew that there was a way to escape because he thought about driving
away. The students would be no match for a moving car.

21. The answer is B. (A) is wrong because damage is not an element of trespass to land.
(C) is wrong because there don’t have to be “No Trespassing” signs for a defendant
to be liable. The intent required is the intent to enter onto the land, so it doesn’t
matter whether Husband and Wife knew that they weren’t supposed to be there.
Also, “No Trespassing” signs would tell a person that he didn’t have permission,
but a lack of signs would not give the person permission. (D) is wrong because
trespass to land does not require that a person know whose land it was. The answer
is (B) because it is the only answer that accurately describes an element of trespass
to land.

22. The answer is A. (A) is correct because it uses the concept of transferred intent.
Transferred intent is available for five torts, and both battery and assault are in that
category. Because Grower wanted to frighten Wife, that intent will enough to prove
intent for battery as long as the rest of the elements are met. (B) is wrong because
even though harmful or offensive contact is an element, “severe” injury is not.
Besides, the central issue is the intent because Grower did not have the intent
usually required for battery – knowing that a harmful or offensive contact will
result. (C) is wrong because it has nothing to do with battery. Assault requires
apprehension, but battery does not. (D) is wrong because confinement has nothing
to do with battery. It only has to do with false imprisonment. Furthermore, (D) is
not true – grower did not mean to confine Wife inside the fence. He wanted her to
leave.

23. The answer is C. The “if” in (C) tells you that you must take it as true that Husband
saw the dog coming at him. That refers to apprehension, which is an element of
assault, and the rest of the elements are present in the fact pattern. (A) is wrong
because biting would fit a battery claim (harmful or offensive contact), but it isn’t
necessary for assault. (B) is wrong because assault does not require actual fear. It
is enough if Husband was aware that the dog was coming toward him. (D) is wrong
because Grower does not have to intend for the dog to touch Husband. He intended
to “frighten” Husband and Wife, which means that he intended for them to think

273
that they were about to suffer a harmful or offensive contact. That is the intent
needed for assault.

24. The answer is C. (C) is the best answer because Dan wanted to cause a contact with
Pauline that he had to know would be offensive. That kind of intimate contact with
a stranger would violate the person’s right to bodily integrity because women have
to right not to be touched in private places by strangers. (A) is wrong because there
is no indication that Dan wanted the touching to happen or knew that it would – it
sounds like an accident. Furthermore, in a crowded elevator, people tend to bump
up against each other, and we don’t sue for it because it is such a commonplace,
everyday occurrence. (B) is wrong because in that context, Dan would not know
that such a socially acceptable touching would be harmful or offensive. (D) is
wrong because Dan probably would not know that patting his wife would offend
her because they are in an intimate relationship.

25. The answer is B. (B) is most likely to be an assault because the facts seem to fit all
the elements. A knew that B would feel that he was about to suffer a battery
because he raised his fist and walked toward B, and that is a very threatening pose.
The battery was imminent because A was only four feet away and was closing in on
B. And B was aware of what A was doing because B was looking at A. (A) is
wrong because the element of apprehension is missing – B wasn’t looking at A, so
B didn’t know what A was doing. (C) is wrong because the threat is not imminent.
A isn’t telling B that A is going to hit B now. Instead, A is telling B that A will hit
B some time in the future, and only if B does something first. (D) is wrong because
there is no threat – only words.

26. The answer is D. An element of IIED is that plaintiff must know that his actions
would cause plaintiff severe emotional distress. Defendant did not know that
Robby was plaintiff’s brother, so defendant would have no way to know that
assaulting Robby would cause plaintiff not only emotional distress, but severe
emotional distress. (A) is wrong because this is a good claim for trespass to land.
The intent required is the intent to cause entry onto land. Even though defendant
did not know that the porch was hanging over plaintiff’s land, defendant did know
that he had put it there. (B) is wrong because the first element of trespass to
chattels is intent, which means knowing that serious harm to the chattel will result.
Defendant had to know that hitting a tiny dog with a hard object like an umbrella is
likely to cause serious harm. Also, defendant did substantially interfere with the
value and condition of the chattel. That element is proved by the gash. (C) is
wrong because it does state a claim for conversion. The intent required is knowing
that the act will prevent the owner from being able to exercise control over the
chattel. This intent does not mean that defendant has to know that the chattel
belongs to someone else.

27. The answer is B. (A) is wrong because there is no apprehension of imminent


battery. Joe said that if he weren’t in such a good mood, he would flatten
Salesman’s face. He did not say that he was going to flatten Salesman’s face. (C)

274
is wrong because stores are open to the public. Stores want people to come in, and
Joe had not yet overstayed his welcome. (D) is wrong because Joe could not have
known that severe emotional harm would result, nor did it result. However, all the
elements of battery are arguably present (intent, contact, harmful or offensive), so
(B) is the right answer.

28. The answer is D. (A) is wrong because plaintiff will lose. Defendant is walking
away from plaintiff, so plaintiff can’t have an apprehension of imminent battery.
(B) is wrong because defendant does not have intent. He doesn’t know that his act
will put plaintiff in apprehension of imminent battery because he doesn’t even seem
to know that plaintiff is there – defendant is looking at his girlfriend, not at plaintiff.
(C) is wrong because defendant does not have apparent ability to carry out a
harmful or offensive contact – he is too far away to possibly touch her, and he is
confined behind bars. (D) is the best answer. Defendant had to know that he would
cause plaintiff to apprehend an imminent battery when he ran at plaintiff with his
fist raised. He had apparent ability to commit the battery because he was running
toward plaintiff with his fist all ready to hit plaintiff. It was also imminent because
it was happening right then.

29. The answer is A. The boundaries of land extend upward, but not infinitely upward.
The object entering the land must be within a reasonable distance from the surface
of the land. Therefore, to win, Pauline must show that the helicopter was within a
reasonable distance. (B) is wrong because damage is inferred in trespass cases, so
Pauline does not have to prove damage. (C) is wrong because expectation of
privacy is not an element of trespass. (D) is wrong because it contains the wrong
definition of the intent required for trespass. The intent required is to enter the land
or to send something onto the land; it does not matter whether the defendant knows
whose land it is.

30. The answer is D. (D) is the answer that implies the correct definition. If Dan knew
that accusing an average person of having SARS would cause that person severe
emotional distress, then Dan’s purpose was to cause severe emotional distress.
None of the other answers contain an accurate definition of the intent required.

31. The answer is B. (B) is the answer that implies the correct definition. If Donald
knew that Pat thought she might fall, then he knew that she had an apprehension of
an imminent harmful or offensive contact. (D) is not a good answer because
transferred intent does not apply to intentional infliction of emotional distress.

32. The answer is A. If Donald knew that Pat would suffer extreme emotional distress,
then the intent would be proved. (A) states that Donald had no intent to inflict any
emotional distress. But he had to know that Pat was suffering emotional distress
because (1) she was very young, and therefore vulnerable; (2) she told him that she
was scared to death, so he knew she was upset to start with; (3) Donald was sending
her out on a steel beam 400 feet above the ground, where any normal person would
be absolutely terrified; and (4) at the end of all of this, he tells her there’s no job and

275
it’s just a joke. That might devastate anyone. So it’s not clear that Donald did not
have the required intent, making (A) the most wrong answer. (B) is a true statement
because it is possible to recover for IIED without physical injury. It may be more
difficult, but it is possible. (C) is true – these actions might be extreme and
outrageous. Finally, (D) is true. If the person playing the joke knew that severe
emotional distress would result from it, an IIED claim is possible.

33. The answer is B. False imprisonment requires that the person be confined. In this
case, that would mean that Patsy’s will to leave must have been overcome. But
Dick did nothing to prevent Patsy from leaving. (A) is wrong because damage is
not an essential element of false imprisonment. Although Dick’s conduct might
have prevented Patsy from reaching her son, Patsy has to prove that she was
confined. Therefore, (C) and (D) are wrong.

34. The answer is C. Mistake is no defense for intentional trespass. Donald


erroneously believed the channel was a public waterway when, in fact, it was
owned by Penelope. Choice C is the correct answer because it addresses Donald’s
intentional entry into the channel. Note that choice D is wrong because there was
no easement created by necessity because there was no conveyance of land
presented in the facts.

35. The answer is A. In answering this question the student is challenged to know that
elephants are wild animals. The Restatement of Torts, 2d, Section 507 states “A
possessor of a wild animal is subject to (strict) liability for trespass to another for
harm done” even though the possessor has exercised the utmost care to confine
the animal. As a result, choice (A) is the best answer.

36. The answer is C. Here, (A) and (B) are clearly incorrect because the cases thus far
decided have found liability for emotional distress only where the defendant’s
conduct has been extreme and outrageous. Certainly Coach Dan’s conduct was
not so extreme and outrageous to a degree or of a character to go beyond all
possible bounds of decency to be regarded as atrocious. Therefore the choices are
ether (C) or (D). Clearly both can be correct, but one is more correct than the
other. (D) may be a logical pick because the conduct here may be extreme and
outrageous because of the actor’s knowledge that the victim is peculiarity
susceptible to emotional distress by reason of some physical or mental condition
or peculiarity. It may be cruel or heartless but it must be emphasized that major
outrage is essential to the tort, and the mere fact that the actor knows that the
victim will regard the conduct as insulting or will have his feelings hurt is not
enough. Consequently the correct answer is (C).

37. The answer is C. Under the “rescue doctrine” efforts to protect the personal safety
of another have been held not to supercede the liability for the original negligence
that has endangered it. As Judge Cardozo stated in Wagner v. International R.
Co., 232 N.Y. 176, 133 N.E. 437 (1921), “The Risk of rescue, if only it be not
wanton, is born of the occasion. The emergency begets the man.” There is thus an

276
independent duty of care owed to the rescuer himself, which arises even when the
defendant endangers no one’s safety but his own. Remember that whether the
rescuer injures himself, the person rescued or a third party, the original wrongdoer
is still liable.

38. The answer is B. Self defense in available if a person reasonably believes he is in


danger of being attacked, even if later he learns that there was no danger. Students
should be aware that there are both subjective and objective components to the
self-defense doctrine. The defendant must honestly believe he is about to be
attacked and the belief must be reasonable. Both (C) and (D) are incorrect because
Dave acted honestly and reasonably so his attack on Crooner is privileged as self-
defense. (A) is wrong because if Dave honestly but unreasonably believed
Crooner was about to attack him, then Dave’s actions would not be privileged.

39. The answer is C. Pious will not recover if Drugdealer’s conduct did not threaten
immediate physical harm. Assault requires that the defendant create apprehension
of an immediate battery. Threats of future harm without some act in furtherance
of the threat are not enough to establish assault. Intent to place someone in
immediate fear of physical harm is not enough, so (A) is wrong. Extreme and
outrageous conduct is not an element of assault, so (B) is wrong. Any action to
eliminate the threat by the plaintiff is irrelevant and do not vitiate any threat of
harm.

40. The answer is D. When the bullet came down in Penny’s yard it entered her
airspace. While Dante did not purposely shoot the bullet into her airspace, he
would know with substantial certainty that the bullet would enter into someone’s
airspace. (A) is incorrect because it is irrelevant whether Dante meant to cause
apprehension about leaving the pool. Apprehension in an assault involves an
imminent battery, not fear of leaving a dog alone. (B) is wrong because the
statement that Penny can only recover if the dog dies is incorrect. The tort of
trespass to chattels is a possibility if the dog lives and fully recovers; the tort of
trespass to land is also represented in the correct answer. (C) is incorrect, because
even if the act was extreme and outrageous, the facts do not establish any other
element of Intentional Infliction of Emotional Distress.

41. The answer is A. No contract arose on February 2nd because Shaftum’s offer
expired on February 1st, when Shaftum did not receive Martin’s acceptance. If a
period of acceptance is stated in an offer, the offeree must accept within that
period to create a contract. Failure to timely accept terminates the power of
acceptance in the offeree (i.e., a late acceptance will not be effective and will not
create a contract). Under the mailbox rule, an acceptance generally is effective
upon dispatch (i.e., the acceptance creates a contract at the moment it is mailed or
given to the telegraph company). However, the mailbox rule does not apply
where the offer states that acceptance will not be effective until received. In the
latter case, acceptance is effective only upon receipt. Here, Shaftum’s offer
specifically stated that the acceptance must be received by February 1st to be

277
effective. Thus, Shaftum opted out of the mailbox rule, and no contract was
created by delivery of the telegram on February 2nd. Note that Martin will not be
able to successfully argue that the acceptance was valid since the late delivery
was the telegraph company’s fault. This would be a valid argument if the
mailbox rule applied here because the acceptance would have been effective on
February 1st, when the message was given to the telegraph company. However,
by opting out of the mailbox rule, Shaftum put the burden of any negligence in
delivery on Martin. Thus, there was no valid acceptance. (B) is incorrect because
of the requirement that acceptance be received by February 1st. This requirement
obviates that general “mailbox rule,” so that mere mailing of a letter (or sending
of a telegram) does not operate as an effective acceptance. (C) is incorrect
because Shaftum was not obligated to respond in any way to the telegram
received on February 2nd. Once the specified time passed without receipt of
acceptance, the offer (as well as Martin’s power of acceptance) was terminated.
Thus, receipt of the telegram on February 2nd created neither a contract nor an
obligation on the part of Shaftum to respond to the telegram. (D) is incorrect
because no contract, voidable or otherwise, arose on February 1st. Furthermore,
the facts do not indicate circumstances under which a contract is usually held to
be voidable. A voidable contract is a contract that one or both parties may elect to
avoid or to ratify (e.g., contracts of infants). The facts of this question provide no
basis for concluding that any contract that might have arisen between these parties
would be voidable.

42. The answer is A. A contract was formed here because there was mutual assent
and valid consideration. For there to be mutual assent, there must be a valid offer
and an unequivocal acceptance before the offer is either rejected by the offeree or
revoked by the offeror. Here, Fox offered to sell his house and lot at 337 Green
Street for $100,000. This was an offer to Sack. The issue here is whether either
of Sack’s replies constitutes an unequivocal acceptance of this offer. A
counteroffer serves as a rejection of the original offer as well as a new offer.
However, a mere inquiry about additional terms or matters is not a counteroffer.
The test of whether the reply is a counteroffer or inquiry is whether a reasonable
person would believe that the offer was being rejected. Here, Sack’s February 8th
communication was a mere inquiry, rather than a counteroffer. His statements do
not show an outright rejection if his terms are not agreed to; he merely states what
he would prefer and then asks Fox to consider his proposal. Most likely this
would be considered to be an inquiry. The February 11th communication is an
acceptance. Sack clearly states that he accepts the “offer of February 3rd”
(showing that he is agreeing to Fox’s original terms). The additional language in
Sack’s telegram (about the deed) is not an alteration of the original terms because
implicit in a sale-of-land contract is that a deed will be conveyed. Statements by
the offeree that make implicit terms explicit do not prevent acceptance of the
offer. Thus, Sack’s second communication unequivocally accepts Fox’s offer.
Since promises were exchanged (a promise to sell the property for a promise to
pay $100,000), valid consideration existed, and a contract was formed. (B) is
wrong because, as explained above, Sack’s February 8th communication did not

278
constitute a counteroffer, which would act as a rejection of the offer. Since the
communication was a mere inquiry, the offer was still viable, and Sack could
accept it. (C) is wrong because the first communication only inquired about
altering the deal, and the second merely expressed terms already implicit in the
offer. Neither contained alterations of the offer’s terms. (D) is wrong because a
contract for the sale of land need only identify the land and contain a price term.
Here, the offer adequately described the property and stated the price. It was a
sufficiently definite offer.

43. The answer is C. At common law, an acceptance is effective upon dispatch (e.g.,
upon mailing a properly addressed and stamped letter) under the “mailbox rule.”
There is no mailbox rule, however, for revocations – revocations are effective
only upon receipt. Receipt does not require knowledge of the revocation, but
merely possession of it. [See Restatement (Second) of Contracts §68.] The facts
here present a close question as to whether the acceptance was sent before the
revocation was received. The outcome of this question will depend on the court’s
determination as to which came first (the posting of the acceptance or receipt of
the revocation). This will decide the existence or nonexistence of the contract.
(A) is incorrect because, as indicated above, revocation is effective only upon
receipt, not mailing. (B) is incorrect because whether the acceptance is effective
depends on whether the revocation was received before the acceptance was sent,
and whether the revocation was received first is not dependent on whether Sack
know of its contents; instead, it depends on whether Sack had possession of it.
(D) is incorrect because the mailbox rule makes acceptances effective upon
posting, and there is no reason to hold that handing a properly addressed, stamped
letter to a mail carrier is not a valid posting.

44. The answer is C. Since Professor had stated the nature of the performance that he
was requesting and had specified the terms, Professor would be held to have made
an offer, creating a power of acceptance in the student who received the highest
grade. Thus, (A) and (D) are incorrect. (B) is incorrect because Professor’s
motive was to induce a detriment (intensive study), and gratification from
influencing the mind of another may be sufficient to establish bargained-for
consideration. Thus, this was not to be a gift.

45. The answer is B. Since Professor’s offer could only be accepted by completion of
performance (that is, obtaining the highest grade in his class), when Stillwell
received the highest grade, a unilateral contract was formed. (A), (C), and (D) are
therefore incorrect.

46. The answer is D. A unilateral contract is revocable until performance is begun.


Here, Stillwell’s purchase of every Constitutional Law outline is not the
beginning of performance, but rather mere preparation to perform. Nevertheless,
the purchase constitutes foreseeable detrimental reliance sufficient to make the
promise binding on Professor. Thus, the offer is irrevocable as to Stillwell. (A) is
wrong because the offer was valid. (B) is wrong as to Stillwell, although it would

279
be correct as to anyone who had not detrimentally relied on the offer. (C) is
wrong because a revocation need only be published in the same manner as the
offer; it need not actually reach everyone who knew of the offer.

280

You might also like